ORTHOPEDIC MCQS OB 20 TRAUMA1
ORTHOPEDIC MCQS OB 20 TRAUMA1
- What percentage of patients will complain of knee pain at the time of union of a tibial shaft fracture treated with a reamed intramedullary nail?
1. <10%
2. 10-33%
3. 33-50%
4. 50-75%
5. >75%
CORRECT ANSWER: 4
Anterior knee pain is the most common complication after intramedullary nailing of the tibia. Dissection of the patellar tendon and its sheath during transtendinous nailing was thought to be a contributing cause of chronic anterior knee pain.
The referenced paper by Toivanen et al. compared two different nail-insertion techniques in 50 patients who were randomized to treatment with paratendinous or transtendinous nailing. Sixty-seven percent of the transtendinous and seventy-one percent of the paratendinous approaches resulted in patients with postoperative anterior knee pain. The same authors published an 8 year follow-up which showed that the percentage dropped down to 29%, but there was still no advantage of paratendinous over the transtendinous approach.
In the more recent study by Lefaivre with an average patient follow up of 14 years, knee pain was present in greater than 70% of the respondents.
- A 28-year-old man is thrown from his motorcycle and sustains the closed injury seen in Figure A. The limb remains neurovascularly intact. What is the most appropriate initial treatment of this injury?
- Bulky compressive splint
- Open reduction and internal fixation
- Closed intramedullary nailing
- Spanning external fixation
- Hinged spanning external fixation
- CORRECT ANSWER: 4
Figure A shows a significantly displaced, high-energy proximal tibia fracture with intra-articular extension. Appropriate initial treatment includes application of a spanning external fixation device with fasciotomy if needed.
The referenced article by Egol et al noted a low rate of wound infection, improved access to soft tissues, prevention of further articular damage, and osseous stabilization. They reported the downside being residual knee stiffness.
- The mangled extremity severity score (MESS) utilizes all of the following variables EXCEPT:
- Limb ischemia
- Shock
- Patient age
- Skeletal and soft tissue injury
- Time from admission to surgery
- CORRECT ANSWER: 5
The MESS is a tool utilized to help predict limb salvage success versus primary amputation at the time of presentation. As a screening tool for amputation, this scoring system has a high specificity but low sensitivity, as scores lower than 7 may also ultimately need amputation. All of the variables except choice
#5 are part of the scoring system.
The scoring system is as follows: 1. Skeletal / soft-tissue injury: Low energy = 1; Medium energy = 2; High energy = 3; very high energy = 4; 2. Limb ischemia: Pulse reduced or absent but perfusion normal = 1; Pulseless = 2; Cool, paralyzed, insensate = 3; 3. Shock: normotensive = 0; transient hypotension = 1; persistent hypotension = 2; 4. Age: < 30 = 0; 30-50 = 1;
>50 = 2. Limb category scores are doubled for ischemia > 6 hours. The system's original designers reported a cutoff of 7 as predicting amputation.
The referenced study by Ly et al found that the scoring system did not predict functional outcomes at 6 or 24 months. They also found that the Limb Salvage Index; the Predictive Salvage Index; the Nerve Injury, Ischemia, Soft-Tissue Injury, Skeletal Injury, Shock, and Age of Patient Score; and the Hannover Fracture Scale-98 all did not predict outcomes at 6 or 24 months.
- A 62-year-old man slips on ice and sustains an elbow dislocation. Post-reduction imaging reveals a highly comminuted radial head fracture and coronoid fracture through its base. What is the most appropriate treatment?
- Early passive range-of-motion in a hinged elbow brace
- Application of a static spanning external fixator for 6 weeks
- Radial head excision, coronoid excision, and repair of the lateral ulnar collateral ligament and medial collateral as needed
- Radial head excision, open reduction internal fixation of the coronoid, and repair of the lateral ulnar collateral ligament and medial collateral as needed
- Radial head replacement, open reduction internal fixation of the coronoid,
and repair of the lateral ulnar collateral ligament and medial collateral as needed
CORRECT ANSWER: 5
The results of elbow dislocations with associated radial head and coronoid fractures are often poor because of recurrent instability and/or stiffness from prolonged immobilization. Therefore radial head replacement and open reduction internal fixation of the coronoid is the most appropriate treatment.
Pugh et al reported their experiences with this difficult population. Their protocol consisted of ORIF or replacement of the radial head, ORIF of the coronoid fracture, repair of the LCL and capsule, and repair of the MCL and/or hinged external fixation. Of the 36 cases, the outcome was graded as 28 excellent to good, 7 fair, and 1 poor. 8 cases required re-operation. The authors concluded that their surgical protocol restored sufficient elbow stability to allow early motion post-op, thereby enhancing the functional outcome. In fracture dislocation of the elbow with radial head and coronoid fracture, the radial head must be fixed or replaced to restore stability. The ORIF of coronoid fracture and radial head restores some valgus stability therefore MCL repair may not be needed. However, the varus stability must be restored by LCL repair.
- Which of the following has been shown to have similar biochemical and clinical characteristics as iliac crest autograft?
- BMP-2
- BMP-7 with collagen matrix carrier
- Hydroxyapatite cement
- Platelet rich plasma with allograft cancellous bone carrier
- Femoral intramedullary reaming contents
- CORRECT ANSWER: 5
In multiple studies, femoral intramedullary reaming debris has been shown to have similar biochemical characteristics as iliac crest autograft. Intramedullary reaming products have osteogenic potential with viable cells while BMP's are osteoinductive cytokines.
Hoegel et al found that the reamings had alkaline phospatase activity, indicating living osteoblasts. The amount of activity was independent of the reamer sizes and reamer design.
Frolke et al concluded that reaming debris supports callus building (healing) as much as conventional iliac crest bone grafting in an animal fracture gap model.
The video shows a retrograde femoral autograft harvest using the RIA system.
- After open reduction and internal fixation of long bone fractures, at what time period should C-reactive protein start to decrease?
- 24 hours
- 48 hours
- 96 hours
- 7 days
- 12 days CORRECT ANSWER: 2
C-reactive protein (CRP) should peak by 48 hours after surgical fixation of bony orthopedic injuries, and decrease thereafter. This is important to recognize, as an increasing CRP after 48 hours is predictive for postoperative infection, and is more predictive in the first postoperative week than local erythema, persistent serous drainage, and increasing serial ESR.
The first referenced study by Waleczek et al noted that CRP was the earliest sign of developing infection and that clinical diagnosis, ultrasound, and WBC counts all lagged significantly behind CRP as a diagnostic tool. They report that latency to the clinical diagnosis based on clinical signs, ultrasound, WBC in blood and wound drainage was up to 14 days, while there was no patient with CRP increasing after day 2 without an infection.
The second referenced article by de Zwart et al noted an increased sensitivity and specificity of CRP as compared to ESR in the scenario of a clinically suspected infection. They advocate for determination of two CRP-levels with a short interval to screen for a clinically suspected infection.
Furthermore, Mok et al found that CRP showed an exponential decrease with a half-life of 2.6 days in postoperative spine patients. They found that CRP is a reliable test in the early postoperative period compared with ESR.
- Which of the following is the most appropriate clinical scenario to utilize locking plate and screw technology?
- Intra-articular fracture
- Oblique ulnar diaphyseal fracture
- Osteoporotic periprosthetic distal femur fracture
- Transverse tibial diaphyseal fracture
- Spiral humeral diaphyseal-metaphyseal fracture CORRECT ANSWER: 3
Conventional plating provides stable internal fixation when fractures are anatomically reduced. Stability of this type of fixation relies on the plate/bone interface and the friction that develops between this interface. Locked plates rely on the plate/screw interface, and each provides not only axial stability but also angular stability; each screw acts as a fixed angle device. Indications for locked plating for indirect reduction include: 1. metaphyseal/diaphyseal fractures 2. comminuted diaphyseal fractures 3. comminuted metaphyseal fractures. 4. short segment fixation. Locked plates are not indicated for displaced articular fractures unless anatomic rigid fixation of the articular surface is done first (locking technology cannot reduce fractures/lag segments together).
The referenced article by Gardner et al reviews locking technology and reminds us that compression technology using non-locking screws and plates is still needed for many fractures and is even required for proper treatment of some fractures.
The referenced article by Wagner is an instructional paper on how to use hybrid plating technology and reviews concepts such as the necessity of lag screw fixation before locking.
The referenced study by Egol et al is a review paper that notes that locked plates and conventional plates rely on completely different mechanical principles to provide fracture fixation and in so doing they provide different biological environments for healing. They report that locked plates are indicated for: indirect fracture reduction, diaphyseal/metaphyseal fractures in osteoporotic bone, and with bridging severely comminuted fractures.
- Which muscles cause the fracture displacement of the proximal fragment shown in figure A?
- gluteus maximus and adductors
- gluteus maximus and rectus femoris
- gluteus medius and hamstrings
- gluteus medius and iliopsoas
- rectus femoris and hamstrings CORRECT ANSWER: 4
The gluteus medius attaches to the greater trochanter, leading to abduction, while the iliopsoas attaches to the lesser trochanter, leading to flexion. French et al evaluated forty-five Russell-Taylor Type 1B subtrochanteric femoral fractures which were stabilized using an interlocked cephalomedullary nail. The intraoperative complication rate was 13.5%; and the most frequent complication was a varus malreduction. The primary reason for this was failure to counteract the muscle forces acting on the proximal fragment combined with the adducted position of the distal femur during portal creation. This problem can be avoided if the position of the proximal fragment is evaluated carefully and reduced before guidewire insertion.
- Which of the following is true of a knee disarticulation as compared to a transtibial amputation?
- Faster self-selected walking speeds
- Improved performance on the Sickness Impact Profile (SIP) questionnaire
- Physicians were more satisfied with the cosmetic appearance
- Decreased use of a prosthetic
- Decreased dependence with patient transfers CORRECT ANSWER: 4
Knee disarticulation level is associated with the worst functional result 2 years after injury (compared to transmetatarsal, Symes, AKA, or BKA). The prosthetic use is decreased with a knee disarticulation as compared to a transtibial amputation.
The cohort study by MacKenzie et al prospectively followed 161 patients that were part of the Lower Extremity Assessment Project (LEAP). These patients underwent an above-the-ankle amputation at a trauma center within 3 months following the injury and followed for 2 years. This study revealed that through- the-knee amputations had significantly worse scores for the objective performance measures of self-selected walking speed, independence in transfers, walking, and stair-climbing. Through-the-knee amputees also had worse SIP scores than AKA and BKA patients. Physicians were also less satisfied with both the clinical and the cosmetic recovery of the patients with a through-the-knee amputation. It should be noted that patients with a BKA had a faster walking speed than those with an AKA. Despite the worse SIP scores for through-the-knee amputations, patients actually reported less pain than those with an AKA or BKA, though this wasn't statistically significant.
- During open reduction and internal fixation of a both bone forearm fracture, restoration of the radial bow has been most associated with which of the following?
- Improvement in wrist extension strength
- Improvement in wrist flexion strength
- Restoration of forearm rotation
- Restoration of elbow range of motion
- Decreased incidence of synostosis CORRECT ANSWER: 3
Restoration of the anatomy of the radial bow directly correlates with the range of motion postoperatively (pronation-supination).
The referenced study by Schemitsch et al found that restoration of the normal radial bow was related to the functional outcome. A good functional result
(more than 80 percent of normal rotation of the forearm) was associated with restoration of the normal amount and location of the radial bow. Similarly, the recovery of grip strength was associated with restoration of the location of the radial bow toward normal.
- In a pilon fracture, the Chaput fragment typically maintains soft tissue attachment via which of the following structures?
- Interosseous ligament
- Anterior inferior tibiofibular ligament
- Posterior inferior tibiofibular ligament
- Deltoid ligament
- Tibiotalar ligament CORRECT ANSWER: 2
The Chaput fragment, highlighted by the arrow in Illustration A, is the anterolateral fragment of the distal tibia. This section of bone attaches to the anterior inferior tibiofibular ligament and is often hinged off this structure due to the fracture. A pilon fracture is often split into three main fragments at the joint level (Illustration B): Chaput fragment (anterolateral), Volkmann fragment (posterolateral), and a medial fragment. The Volkmann fragment is the attachment site of the posterior inferior tibiofibular ligament. The Wagstaff fragment is the fibular corollary to the Chaput fragment, and serves as the other attachment of the anterior inferior tibiofibular ligament.
- A 55-year-old male is involved in a motor vehicle accident and sustains the injury seen in Figure A. What is the most appropriate treatment for this type of injury?
- Total hip arthroplasty
- Bipolar hemi-arthroplasty
- Sliding hip screw
- Percutaneous screw fixation
- Cephalomedullary nail fixation
- CORRECT ANSWER: 5
The radiographs demonstrate a reverse obliquity intertrochanteric femur fracture. Compared to the more stable intertrochanteric femur fracture, a reverse oblique intertrochanteric hip fracture is not optimally treated with a sliding hip screw. Compression along a sliding hip screw is designed to create compression along the plane of the fracture, however in a reverse obliquity fracture pattern as seen here, shear force is created causing medial displacement of the femoral shaft and screw cutout.
Haidukewych et al showed in their retrospective review of 55 consecutively treated reverse obliquity intertrochanteric fractures, that patients treated with a sliding hip screw had nearly a 56% failure rate (9/16). The failure rate of patients treated with a blade plate was only 13%.
Sadowski et al showed in their prospective randomized trial in patients with a reverse obliquity or transverse intertrochanteric fracture who were randomized to either a 95 degree screw-plate or cephalomedullary nail a much higher failure rate for the plate-screw implant. Implant failure was seen in 7/19 patients treated with the 95 degree screw plate and only 1/30 in the intramedullary nail group. Both articles support the use of a blade plate or cephalomedullary nail for reverse obliquity fractures.
An example of screw cutout and medial displacement is seen in Illustration A.
- A 42-year-old male sustains the closed injury shown in Figure A. Which of the following factors is associated with improved outcomes with open reduction and internal fixation?
- Age > 40
- Smoking
- Male sex
- No worker's compensation involvement
- Career as construction worker CORRECT ANSWER: 4
The clinical and radiographic presentation is consistent with a closed, displaced, comminuted calcaneus fracture. Non-worker's compensation patients have improved outcomes with operative treatment.
Buckley et al performed a multicenter prospective randomized study of over 300 displaced calcaneus fractures comparing nonoperative vs. operative treatment. They looked at patient satisfaction, SF-36 scores, and Bohler’s angle. They showed improved outcome in at least one measure after an operation vs. no operation for intraarticular calcaneal fractures in: 1) Women
2) Age <30 3) Non Smokers 4) Light work laborers 5) Non worker’s compensation patients.
Sanders et al reviews calcaneus fractures, including treatment techniques. They recommend getting two-dimensional computed tomographic scans in both the coronal and the transverse plane. They report with operative treatment, the goal is restoring not only articular congruency, but also the shape and alignment of the calcaneus.
- A 36-year-old male is brought to the trauma center following a motor vehicle accident. Physical exam shows a deformed left lower extremity with a 1-cm open wound over the anterolateral aspect of his leg. Radiographs are provided in Figures A and B. Which of the following interventions has been shown in the literature to decrease the occurrence of infection at the fracture site?
- Operative debridement within 6 hours of injury
- Immediate prophylactic antibiotic administration
- Immediate stabilization with internal fixation after debridement
- Irrigating with a saline solution that is mixed with an antibiotic
- Irrigating with high pressure pulsatile lavage following surgical debridement CORRECT ANSWER: 2
The clinical scenario and radiographs are consistent with a Gustilo and Anderson type 3A open tibia fracture.
Melvin et al review the evidenced-based literature and make recommendations for the initial evaluation and management of open tibial shaft fractures. The time elapsed before antibiotic administration and adequate surgical debridement of all contamination are the only factors definitively shown to reduce infection and improve outcome. Traditional recommendations have suggested surgical debridement of open fractures occur within 6 hours of injury. However, there is no literature to support this time window. Certainly, open fractures should be addressed with urgency, but there is no evidence reporting a definitive time window. There is insufficient data to recommend gram negative coverage with gentamicin for all open fractures although this is a common practice. The addition of antibiotics to the irrigation solution has been shown to decrease bacterial load, but it has also demonstrated host tissue necrosis and delayed wound healing. There is not sufficient data to support its use over a castile soap solution or normal saline. Similarly, high pressure pulsatile lavage decreases bacterial load, but also seeds bacteria deeper within the soft tissues and harms host tissues. There is no evidence to support pulsatile lavage over gravity flow.
- A 34-year-old female is involved in a motorcycle crash. She sustains a talus fracture with associated dislocation of the subtalar joint and maintained congruence of the tibiotalar and talonavicular joints as shown in Figure A. The fracture has healed and she now has symptomatic impingement of the dorsal surface of the talus on the distal tibia and restriction of ankle dorsiflexion. What is the most likely deformity causing these symptoms?
- Combined varus and plantar malunion
- Isolated varus malunion
- Isolated valgus malunion
- Isolated dorsal malunion
- Isolated plantar malunion CORRECT ANSWER: 4
Figure A displays a Hawkins Type 2 talar neck fracture. (Hawkins classification shown in Illustration A). Malunion after inaccurate reduction of talar neck fractures has a reported incidence as high as 32%, with varus malunion occurring most frequently. Dorsal malunion can occur when the body is not properly derotated during reduction and the head fragment remains dorsal to the body. Dorsal malunion can lead to symptomatic impingement of the dorsal surface of the talus on the distal tibia and restriction of ankle dorsiflexion.
Canale found that 3 of the 4 patients with dorsal malunion improved following dorsal beak resection of the talar neck. Patients with varus malunion have decreased subtalar range of motion(especially eversion), walk with the foot internally rotated, and often complain of excessive weight bearing on the lateral border of the foot.
Level 4 evidence from Canale and Kelly found that varus malunion occurred most frequently in Hawkins type 2 fractures that had been treated in a closed manner.
- What is the Injury Severity Score (ISS) for a patient with an open chest wound (Abbreviated Injury Scale, AIS=4), colon transection (AIS=4), femoral fracture (AIS=3), shoulder dislocation (AIS=2), and a thyroid gland contusion (AIS=1)
1. 11
2. 13
3. 41
4. 45
5. 46
CORRECT ANSWER: 3
Injury Severity Score (ISS) scores are used to define injury severity for research purposes. The score is based on anatomic and severity indicies. Injury severity is based upon the AIS (abbreviated injury scale). AIS scores range from 1-6 where 1 is a minor laceration or contusion and 6 is a unsurvivable severe injury. An example of a 6 is a crushed head or brain whereas a 5 is a crushed larynx. Open pelvic fracture and femoral shaft fracture come in at 3 and large joint dislocations are a level 2 injury. ISS is the sum of the squares for the highest AIS grades in the three most severely injured ISS body regions. An ISS greater than 18 reflects multiply injured patients and that a transfer to a trauma center is indicated. So in this case, it would be (4x4)+(4x4)+(3x3)= 16+16+9=41. The AIS table can be found in Miller Review on page 699.
Recently, the New Injury Severity Score (NISS) has been developed and found by some authors (Lavoie et al & Balogh et al) to be more reliable indicator of
length of stay and ICU stay. The NISS differs from the ISS in that the NISS sums the squares of the 3 most significant injuries (even if they occur in the same anatomic area). The ISS sums the 3 most significant injuries in 3 separate anatomic areas.
- A 42-year-old male sustains a closed, isolated ulna shaft fracture with 2mm displacement and 3 degrees valgus angulation. He is treated conservatively with early range of motion but presents at one year with a painful atrophic nonunion. What treatment is indicated at this time?
- Dynamic splinting
- Open autogenous cancellous bone grafting
- Open reduction internal fixation with autogenous bone grafting
- Closed reduction and percutaneous pinning
- Use of an implantable ultrasound device
- CORRECT ANSWER: 3
Appropriate treatment of an atrophic nonunion of the ulna includes open reduction and internal fixation with autogenous bone grafting. The atrophic nature of the nonunion reveals that biology, and not necessarily stability, is the major issue of the nonunion. The referenced article by Ring et al reviews a case series of these patients and found that even in the face of significant preoperative bone resorption, good clinical outcomes and union rate is possible with open plating and grafting. The article by Street reviews intramedullary nailing/pinning of the forearm, and found a 7% nonunion rate with this technique.
- A 62-year-old man falls on his porch and sustains an elbow injury. A radiograph is provided in Figure A. Which of the following is the best treatment?
- Closed reduction and long arm casting
- Early motion with a hinged elbow brace
- Open reduction internal fixation with a tension band construct
- Open reduction internal fixation with a plate
- Fragment excision and advancement of the triceps tendon CORRECT ANSWER: 4
The radiograph shows an olecranon fracture with articular comminution and depression of a large intra-articular fragment. This pattern is best treated with plate fixation to support the articular reduction.
Bailey et al reviewed 25 cases of olecranon fractures (simple and comminuted fracture patterns) treated with plate fixation. All 25 went on to union. There were no major complications reported. Twenty percent of patients underwent hardware removal at a later date for prominence.
Hak et al review the treatment options available for olecranon fractures. Simple intra-articular fractures without comminution are suitable for tension band fixation. Comminution of the articular surface is an indication for plate fixation and may benefit from bone graft to support depressed articular segments. Osteoporotic patients or fractures with severe comminution may do better with fragment excision and advancement of the triceps.
- When viewing pelvic injury radiographs, which of the following describes the findings diagnostic of an isolated transverse acetabular fracture?
- Fracture line crossing the acetabulum with disruption of the iliopectineal and ilioischial lines
- Disruption of the iliopectineal and ilioischial lines, with extension into the iliac wing and obturator ring
- Disruption of the iliopectineal and ilioischial lines, with extension into the obturator ring
- Isolated disruption of the iliopectineal line, with an intact ilioischial ine
- Isolated disruption of the ilioischial line, with an intact iliopectineal ine CORRECT ANSWER: 1
Transverse acetabular fractures separate the innominate bone into two fragments, the superior iliac and the inferior ischiopubic, by a single fracture line that crosses the acetabulum horizontally. The iliopectineal and ilioischial lines are disrupted on the AP pelvis radiograph. Axial CT scan of this fracture pattern at the level of the dome will show a vertical anterior to posterior fracture line. Illustrations A-C show AP and Judet pelvic radiographs of a transverse fracture. Illustration D demonstrates the axial CT appearance of this fracture type. Answer choice 2 is describing a both column injury or anterior column posterior hemitransverse, and answer choice 3 describes a T-type fracture pattern. Answer choices 4 and 5 describe an anterior column and posterior column injury respectively. Judet et al provide one of the first comprehensive reviews on acetabular surgical approaches, fracture types, and radiographic anatomy. Illustration E demonstrates the acetabular classification scheme developed by Judet.
- A patient undergoes the treatment seen in Figure A for a displaced intertrochanteric femoral fracture. With use of this construct, a starting point 3 mm anterior to the center of the piriformis fossa has which of the following benefits?
- Improved placement of screws through the nail into the femoral head
- Decreased risk of varus alignment
- Decreased risk of joint penetration
- Decreased risk of avascular necrosis of femoral head
- Decreased risk of iatrogenic proximal femur fracture CORRECT ANSWER: 1
Figure A shows an intertrochanteric fracture treated with a cephalomedullary device. A starting point slightly anterior to the piriformis fossa (starting point for standard antegrade femoral nail) has the benefit of improved placement of screws through the nail and into the femoral head. This is due to the fact that the hip is anteverted and the femoral neck arises from the anterior portion of the proximal femur. Therefore, by moving the nail anterior, that will increase the distance between the head screw and the posterior cortex of the neck and lead to a “straight” shot into the center of the femoral head.
Johnson et al investigated the effect of starting hole position, fracture component length, reamed diameter, and nail type on the potential for femoral bursting and fracture instability. They found the most significant factor in the proximal femoral component was found to be the position of the starting hole.
They found excessive anterior displacement greater than 6 mm from the neutral axis of the medullary canal consistently caused high hoop stresses at the level of the fracture, which can increase the possibility of iatrogenic fracture. Posterior starting points increase the risk of possible distal femur anterior cortex impingement/fracture.
Ostrum et al showed that lateral starting points should be avoided in order to avoid varus reduction when using a trochanteric antegrade nail in subtrochanteric fractures. They recommended a slightly medial starting point.
- An 11-year-old child has a tibia-fibula fracture following a fall from a swing. The fracture is reduced and placed in a long leg splint in the emergency room. What is considered the earliest sign or symptom of a developing compartment syndrome of the leg?
- pain out of proportion to injury
- pale appearance of the foot
- loss of the ability to move the toes
- decreased sensation in the foot
- decreased pulses in the foot CORRECT ANSWER: 1
The Willis reference states “the single most important symptom of impending compartment syndrome is pain out of proportion to the injury." This symptom requires a conscious patient. Most children requiring a reduction for a displaced upper or lower extremity fracture will become comfortable soon after the reduction has been completed. Children requiring frequent analgesia or complaining loudly about pain should be examined very carefully for possible compartment syndrome.” The key wording in this question is “earliest indicator”. Pulselessness, paralysis, pallor, and parasthesias are all late indicators.
The Willis article also lists the most reliable signs of a developing compartment syndrome as severe pain with passive stretching of the involved compartment, pain with palpation of the involved compartment, sensory disturbances
- When discussing treatment options with a 35-year-old healthy male with an isolated, closed tibial shaft fracture, the surgeon should inform him that in comparison to closed treatment, the advantages of intramedullary nail fixation include all of the following EXCEPT?
- Quicker time to union
- Decreased risk of malunion
- Decreased risk of compartment syndrome
- Decreased risk of shortening
- Quicker return to work CORRECT ANSWER: 3
All of the answer choices are correct except #3. Intramedullary nailing can increase the risk of compartment syndrome.
In a study of 94 tibial fractures, Finkemeier reported 10 (11%) had compartment syndromes. Three of the 10 patients developed the compartment syndrome postoperatively.
In comparing IM nailing to non-op, Bone et al showed that IM nailing had a shorter time to union (mean, 18 vs 26 weeks; p = 0.02), lower non-union rate (2% vs 10%), decreased incidence of shortening (2% vs 27%), and quicker return to work (mean, 4 vs 6.5 months), but no difference in compartment syndrome (0% in both groups).
The classic article cited by Sarmiento el al. reported that closed treatment with use of a prefabricated functional below-the-knee brace was effective in a study of 1000 closed diaphyseal fractures of the tibia with an incidence of nonunion of only 1.1%. However, those authors had very strict criteria for use of the fracture-brace (exclusion criteria included intact fibula and tibial shortening
>2cm).
- A 58-year-old man underwent distal radius ORIF with a volar locking plate yesterday. Preoperatively, he reported some mild sensory disturbances in the volar thumb and index finger, but had 2-point discrimination of 6mm in each finger. Now, he complains of worsening hand pain and sensory disturbances in his volar thumb and index finger. Two-point discrimination is now >10mm in these fingers. Radiographs show a well-fixed fracture in good alignment. What is the most appropriate treatment at this time?
- Strict elevation
- Removal of hardware
- Immediate carpal tunnel release
- Carpal tunnel release if no resolution at 6-12 weeks
- Trial of night splinting CORRECT ANSWER: 3
This patient had mild median parasthesias preoperatively that have significantly worsened postoperatively. Immediate carpal tunnel release is the most appropriate next step in treatment.
Mack et al reported on ten cases of acute carpal tunnel syndrome (ACTS) and six cases of nerve contusion in patients with acute median neuropathy associated with blunt wrist trauma. The patients with ACTS initially had normal sensation and subsequently developed objective sensory loss (2-point discrimination greater than 15 mm) in the median nerve distribution associated with severe wrist pain. In contrast, patients with nerve contusion injuries had immediate sensory loss and symptoms were nonprogressive. Four of five patients with ACTS who underwent carpal tunnel release within 40 hours of the onset of numbness had normal 2-point discrimination within 96 hours.
Neuropathy, secondary to nerve contusion without coexisting ACTS, may be treated initially by observation.
Ford et al reported of five cases of ACTS. Four with delayed treatment had poor outcomes while the one patient with early CTR had full recovery. All patients with ACTS had increasing and severe pain in the wrist with parasthesia and impaired sensation in the median distribuation. These symptoms initially weren’t present after wrist trauma, but developed rapidly in the next few hours.
- A 20-year-old man falls from his bicycle. He is going to be scheduled for open reduction internal fixation. What best describes the injury shown in Figure A and B?
- Coronoid fracture
- Capitellum fracture with extension into the trochlea
- Radial head and capitellum fracture
- Isolated capitellum fracture
- Trochlea fracture CORRECT ANSWER: 2
The radiographs shows a coronal shear fracture of the capitellum with extension into the trochlea, which would be classified as a Type IV fracture under the Bryan and Morrey classification system which was modified by McKee to include this specific injury. The lateral radiograph in Figure B and Illustration A is an example of the "double arc" sign representing an injury to both the trochlea and capitellum. The treatment of choice for a displaced Type IV fracture is open reduction internal fixation.
Dushuttle et al demonstrated that absence of the capitellum did not lead to valgus instability unless the medial collateral ligament was injured, suggesting that excision of highly comminuted fractures could be performed.
The reference by Grantham et al looked at a series of capitellum fractures and recommended the choice of treatment should be selective and individualized
depending on age, character of the bone, and type of fracture.
McKee et al in their case review described this coronal injury pattern and their results for ORIF of these fractures.
- An 85-year-old woman falls and injures her elbow in her non- dominant arm. Radiographs are shown in Figure A and B. She also suffers from severe osteoporosis, lives independently, and is a low- level community ambulator. Which of the following is the most appropriate treatment?
- Hinged elbow brace
- Olecranon osteotomy, articular ORIF, locked lateral plating
- Triceps-splitting approach with double plate fixation
- Total elbow arthroplasty
- Casting for 4 weeks then ROM CORRECT ANSWER: 4
Total elbow arthroplasty (TEA) is ideal for treating comminuted osteoporotic fractures of the distal humerus in low demand elderly patients. Outcomes are good to excellent with quick return of stability and functional motion but with carrying weight restriction of 5 lbs. ORIF would be the best choice for younger individuals with better bone quality.
Cobb described the outcomes of 21 total elbow arthroplasties in elderly patients all of which had good or excellent results without evidence of component loosening. The mean motion was 25 to 130 degrees. Complications included fracture of the ulnar component in one patient after another fall, ulnar neurapraxia in three, and reflex sympathetic dystrophy in one.
McKee et al. performed a randomized controlled study of TEA versus fixation and found that TEA for the treatment of comminuted intra-articular distal humeral fractures resulted in more predictable and improved 2-year functional outcomes compared with ORIF. They also found that although elderly patients with this injury have an increased baseline DASH score, they appear to accommodate to objective limitations in function with time.
Frankle et al. retrospectively compared TEA to plate fixation for distal humerus fractures in the elderly and found a significant improvement in outcomes and revision rates with TEA as compared to plate fixation. The differences were seen most in women with associated comorbidities, such as rheumatoid arthritis, osteoporosis, and conditions requiring the use of systemic steroids.
- Coupled with reduction of the syndesmosis, which of the following interventions is most important when surgically addressing the ankle malunion shown in Figure A?
- Placement of an osteochondral allograft
- Fibular lengthening osteotomy
- Calcaneofibular ligament release
- Medial malleolar shortening osteotomy
- Deltoid ligament imbrication CORRECT ANSWER: 2
Late correction with a corrective osteotomy of a fibular malunion associated with diastasis of the ankle mortise (Illustrations A and B) is an effective means of salvaging function in a joint otherwise destined to be stiff and painful.
The referenced study by Offierski et al reports that the factors that determined the success of the revision were the duration of the malunion, the quality of the reduction achieved, and the condition of the articular cartilage at the time of revision.
The referenced study by Chao et al reported that the fibular lengthening osteotomy was crucial in regaining the anatomy and stability of the ankle mortise.
The referenced study by Weber et al is a review of the technique of such an osteotomy, with commentary regarding its clinical success even if mild degenerative changes are seen. They also note that no differences are seen in outcomes between oblique and step-cut osteotomies.
The referenced study by Weber and Simpson is a case series of corrective
lengthening osteotomies after malunited ankle fractures. They report that a lengthening and/or rotational osteotomy of a malunited fibula is successful in preventing further ankle arthrosis if no more than minimal degenerative radiographic changes are seen.
- All of the following implants offer adequate fracture fixation of the injury shown in Figure A EXCEPT:
- Trochanteric entry point cephalomedullary nail
- Piriformis fossa entry point cephalomedullary nail
- Dynamic hip screw
- Fixed angle blade plate
- 95 degree dynamic condylar screw CORRECT ANSWER: 3
Currently, cephalomedullary nails are used widely for reverse obliquity fractures because they limit medialization of the shaft fragment unlike sliding hip screws.
The Haidukewych et al study quoted demonstrated the superiority of fixed angle devices such as blade-plates or dynamic condylar screws over the sliding (or dynamic) hip screws. Reverse obliquity intertrochanteric fractures of the femur are recognized as biomechanically different from standard intertrochanteric fractures. The rate of failure of internal fixation for this fracture pattern was higher than the rates in most reports of internal fixation of intertrochanteric fractures devices.
- A 27-year-old woman gives birth by normal spontaneous vaginal delivery. Two weeks after delivery she reports anterior pelvic pain and a radiograph is obtained (Figure A). What is the next step in management?
- Pelvic external fixator
- Open reduction and reconstruction plating of the symphysis
- Protected weightbearing and binder as needed and observation
- Open reduction and wiring of the symphysis
- Symphysiotomy CORRECT ANSWER: 3
The clinical presentation and radiograph is consistent with an open-book type parturition-induced pelvic dislocation.
The case series by Kharrazi et al reports four patients treated with open-book type parturition-induced pelvic dislocations. The authors advocate nonoperative treatment with bedrest and a properly positioned pelvic binder in the acute setting for patients with a postpartum symphyseal diastasis less than 4.0 cm.
All four patients had significant symptoms and radiographic widening (anterior splaying) of the sacroiliac joints. The three patients who had presented acutely were treated with closed reduction and application of a pelvic binder, while two had closed reduction of their pelvic dislocation while anesthetized with a general anesthetic. At latest follow-up the diastasis at the pubic symphysis reduced to an average of 1.7 cm (range: 1.5-2.0) The authors advocate nonoperative treatment with bedrest and a properly positioned pelvic binder in the acute setting for patients with a symphyseal diastasis of 4.0 cm of less and operative treatment for diastasis greater than 4cm.
- A 30-year-old patient sustains a comminuted tibia fracture and is treated with minimally invasive plating, shown in Figure A. The patient returns to the office 2 weeks after the surgery and reports persistent numbness over most of the dorsum of the foot, but motor exam is normal. What is the most likely explanation?
- unrecognized compartment syndrome
- common peroneal nerve injury
- superficial peroneal nerve injury
- sural nerve injury
- tibial nerve injury CORRECT ANSWER: 3
Superficial peroneal nerve (SPN) injury is a known complication of percutaneous plating of proximal tibial fractures with the LISS system as seen in Figure A.
The Less Invasive Stabilization System (LISS) is a minimally invasive implant that uses indirect fracture reduction techniques. When using the LISS system, percutaneous screw placement increases the risk of injury to nearby structures because they are not necessarily visualized. The superficial peroneal nerve exits the superficial fascia of the leg approximately 8 cm above the tip of the
lateral malleolus. The nerve then travels from posterior to anterior in the vicinity of the distal aspect of the 13-hole proximal tibia LISS plate (near holes 11-13). In a patient of shorter stature, the nerve could cross the distal portion of a 9-hole plate.
Deangelis et al. performed a cadaveric study using Less Invasive Stabilization System (LISS) plates and found that the average distance from the SPN to the center of holes 11, 12, and 13 was 10.0 mm, 6.8 mm, and 2.7 mm respectively. They recommended using a larger incision and careful dissection down to the plate in this region to minimize the risk of damage to the nerve.
Cole et al. retrospectively reviewed 77 tibia fractures treated with LISS and found that 91% healed without complication. In their cohort, there were no superficial peroneal nerve palsies and one deep peroneal nerve palsy.
Figure A demonstrates AP and lateral x-rays of a tibial shaft fracture treated with a LISS plate.
Incorrect Answers:
A: compartment syndrome would have demonstrated pain out of proportion which the patient never complains of
B, D, and E are all less likely to be injured with LISS plate application than the superficial peroneal nerve.
- A 20-year-old male is involved in a motorcycle accident and presents with the injuries shown in Figures A-F. The left ankle injury is open medially, with a clean 3cm laceration, and the right femur and tibia are closed. He has no visceral or head injury, and is hemodynamically stable. He is cleared to go to the operating room. Without taking into account order of fixation, how should his injuries be treated?
- Retrograde nailing of the femur, intramedullary nailing of the tibia, ankle debridement and casting
- External fixation of the femur, intramedullary nailing of the tibia, ankle debridement and ORIF
- Antegrade nailing of the femur, external fixation of the tibia and ankle after debridement
- Retrograde nailing of the femur, intramedullary nailing of the tibia, ankle debridement and ORIF
- External fixation of the tibia and femur, and ankle debridement and external fixation
CORRECT ANSWER: 4
The patient is hemodynamically stable, has no other injuries, and is medically cleared for the operating room. Therefore, there is no need for damage control fixation.
Ostrum et al conducted a review of 20 patients treated by percutaneous stabilization for ipsilateral fractures of the femur and tibial shafts. All patients were treated with a retrograde femoral intramedullary nail and a small diameter tibial intramedullary nail through a 4-cm medial parapatellar tendon incision. Six of the tibial shaft fractures required revision surgery, and no patients reported signs or symptoms of knee pain. Ostrum concluded that although this is an excellent treatment option for patients with ipsilateral femoral and tibial shaft fractures, the tibial fracture complication rates remain high.
Franklin et al reviewed 38 cases of open ankle fractures that had been treated with immediate splinting, antibiotics, debridement, and internal fixation. They found that all of the fractures united, but three patients required subsequent ankle fusion because of cartilage damage noted at the initial operation. Of the thirty-five ankles with complete follow-up, the functional result was excellent in twenty-six and fair or poor in nine.
- A 33-year-old male is involved in a motor vehicle accident and suffers a right pilon fracture. Which of the bone fragments labeled on the distal tibia in the axial CT scan shown in Figure A is attached to the posterior inferior tibiofibular ligament?
- A
- B
- C and B
- D
- A and D CORRECT ANSWER: 4
Figure A is an axial CT scan slice of an intra-articular distal tibia fracture. The bands of the posterior tibiofibular ligament pass obliquely from the fibula to the posterolateral aspect of the distal tibia. The ligaments of the ankle often remain intact after a pilon fracture producing the major fracture segments consisting of posterolateral or Volkmann's fragment (labeled D), the anterolateral or Chaput fragment (labeled B), and the medial fragment (labeled C). The fibula is labeled A. Any surgical approach taken to treat this injuries should respect these attachments.
Michelson reviews the important role of ankle ligamentous anatomy in his study on rotational ankle fractures.
Hermans et al review the anatomy of the ankle syndesmosis and state that stress on the posterior inferior tibiofibular ligament results more often in a posterior malleolus avulsion fracture than in a rupture of the ligament. They go on to state that with direct reduction of the posterior malleolus avulsion fracture, the syndesmosis can often be stabilized.
Illustration A shows the posterior inferior tibiofibular ligament highlighted in red on MRI imaging in a LEFT ankle (the CT image in the question is of a RIGHT ankle).
- What structure has been described as having a risk of injury with retractor placement on the sacrum during combined acetabular-pelvic ring surgery using the Stoppa approach with a lateral window?
- External iliac artery
- Pudendal nerve
- Corona mortis
- L5 nerve root
- Ilioinguinal nerve CORRECT ANSWER: 4
Care must be taken when placing a retractor on the anterior aspect of the sacrum, as the L4 and L5 nerve roots are both at risk.
Illustration A shows a diagram of the lumbosacral plexus, indicating the proximity of the L4 and L5 nerve roots to the anterior sacrum and SI joint.
The first referenced article by Atlihan et al reported on the anatomy of the anterior sacroiliac joint and reported that the L4 nerve root is within 1 cm of the joint at its inferior margin.
The second article by Ebraheim et al found that the L4 and L5 nerve roots are 10 mm medial to the sacroiliac joint at the pelvic brim.
- A 24-year-old man who sustained a gunshot wound to the abdomen ten hours earlier was brought to the emergency department. On physical examination he was found to have 4 of 5 weakness in his bilateral lower extremities. Radiographs are shown in Figure A. Computed tomography of the lumbar spine showed retained missile in the vertebral body and paraspinal soft tissues, but not within the spinal canal. His FAST was positive and he underwent an emergent exploratory laparotomy where an injury to the cecum was identified and treated. Management should now include which of the following?
- Bullet fragment removal from a transabdominal approach
- Bullet fragment removal from a retroperitoneal approach
- Broad-spectrum oral antibiotics for 3-5 days
- Broad-spectrum intravenous antibiotics for 7-14 days
- IV methylprednisolone at 5.4mg/kg/h for 48 hours CORRECT ANSWER: 4
The patient in the scenario has a GSW to the lumbar spine with neurologic deficits but without a retained missile in the spinal canal. In patients with visceral injury, the treatment is broad-spectrum antibiotic coverage for 7 days.
Kumar et al reviewed 33 patients with GSW to the spine and associated visceral injuries. They concluded that 7 days of antibiotic treatment targeted at colonic flora is the treatment of choice.
Roffi et al reviewed 51 low-velocity GSW that perforated the viscus prior to the spine. They concluded that broad spectrum antibiotics combined with bedrest significantly reduced the risk of spinal or paraspinal infections. Furthermore, bullet removal had no effect on infection rates.
Velmahos et al followed 153 GSW to the spine for 28 months. While rates of sepsis were higher in the lumbar spine than cervical and thoracic spine, they concluded that retained bullets do not increase the likelihood of septic complications.
Incorrect Answers:
Answer 1 & 2: Indications for surgery with a GSW to the lumbar spine include
1) spinal instability 2) a neurologic deficit is present that correlates with radiographic findings of neurologic compression by the missile. 3) Lead missile is in contact with the cerebrospinal fluid (CSF). This patient does not have any of these criteria.
Answer 5: GSWs are a contraindication for spinal dose steroids.
- Which of the following is indicative of a patient who has been successfully resuscitated following a trauma?
- Urine output of 0.25 mL/kg/hour
- Lactic acid of 1.9 mmol/L
- Base deficit of 5.5
- Gastric mucosal pH of 6.5
- Pulse pressure of 15
CORRECT ANSWER: 2
Rapid fluid resuscitation is the cornerstone of therapy for hypovolemic shock. Fluid should be infused at a rate sufficient to rapidly correct the deficit. If the estimated blood loss is greater than 30% of the total volume (class III), blood replacement is also indicated. In general, a favorable response to fluid replacement therapy includes increased urinary output (at least 0.5ml/kg/hr), improved level of consciousness, increased peripheral perfusion, and changes in vital signs (such as increased BP, increased pulse pressure, and decreased heart rate). Lab values that are important include lactic acid, which is increased if the shock is severe enough to cause anaerobic metabolism, and decreased serum bicarbonate which leads to a negative base deficit. Successful resuscitation in a shock patient will therefore lead to a falling lactate and normalizing pH. Successful resuscitation in a shock patient will therefore lead to a falling lactate (i.e. <2.0mmol/L) and a normalizing pH.
Incorrect Answers:
1: Urine output should be at least 0.5ml/kg/hr. 3: Base deficit should be less than 2.
4: Gastric mucosal pH is different than gastric fluid pH, and should be greater than 7.2.
5: Pulse pressure should be greater than 15mmHg.
- A 22-year-old female is struck by a truck and sustains the injury seen in figure A. What deformities are most commonly seen in treating this injury with an intramedullary nail?
- Apex anterior and varus
- Apex anterior and valgus
- Apex posterior and varus
- Apex posterior and valgus
- Rotational CORRECT ANSWER: 2
Proximal tibial shaft fractures treated with intramedullary nails are most commonly malreduced with apex anterior and valgus deformities. Several techniques are available to overcome this malalignment: proximal and lateral nail starting point, usage of a femoral distractor or temporary plating, suprapatellar nailing, and lateral parapatellar approaches.
A final technical trick is the usage of blocking (Poller) screws - the referenced article by Ricci et al had 100% correction and maintenance of reduction with usage of blocking screws without other adjunct techniques. They report a union rate of >90% in this small series.
- When treating a stable 2-part intertrochanteric hip fracture with a sliding hip screw construct, what is the minimum number of screw
holes that are needed in the side plate for successful fixation?
- One
- Two
- Three
- Four
- Five
CORRECT ANSWER: 2
A two part stable intertrochanteric femur fracture can be treated with a sliding hip screw, with good biomechanical and clinical results.
The referenced article by Bolhofner et al reviews a series of 69 patients with a sliding hip screw and two hole side plate and notes that they did not have any failure of the side plate construct.
The referenced article by McLoughlin et al is a biomechanical evaluation of 2 versus 4 hole plates and found that peak load in the failure test was not found to be statistically different between the two-hole and four-hole designs. In cyclic testing, the two-hole configuration exhibited statistically smaller fragment migration in both shear and distraction than the four-hole design.
- A 25-year-old male is involved in an high-speed motor vehicle collision and sustains a closed femoral shaft fracture. During further evaluation, a CT scan of the chest/abdomen/pelvis reveals a non- displaced ipsilateral femoral neck fracture. Which of the following treatment options will most likely achieve anatomic healing of the femoral neck and minimize the risk of complications?
- Retrograde femoral nail followed by compression hip screw
- Lag screw fixation followed by plating of the femoral shaft
- Antegrade femoral nail followed by lag screw fixation
- Lag screw fixation followed by retrograde femoral nail
- Proximal femoral locking plate CORRECT ANSWER: 4
An ipsilateral femoral neck fracture occurs in approximately 6% to 9% of all femoral shaft fractures. A comminuted midshaft femoral fracture secondary to axial loading should alert the treating physician to the possibility of an associated femoral neck fracture. As a result, trauma CT scans should be reviewed for non to minimally displaced femoral neck fractures during the
initial work up.
Watson et al did a retrospective review of 13 patients who had healing complications develop after their index surgical procedure for ipsilateral femoral shaft and neck fractures. Six of the eight (75%) femoral neck nonunions occurred after the use of a second generation, reconstruction-type intramedullary nail. Factors contributing to nonunion of the femoral shaft were the presence of an open fracture, use of an unreamed, small diameter intramedullary nail, and prolonged delay to weightbearing. Lag screw fixation of the femoral neck fracture and reamed intramedullary nailing for shaft fracture stabilization were associated with the fewest complications.
Peljovich et al discuss that several treatment options are described in the literature, but no clear consensus exists regarding the optimal treatment of neck/shaft fractures. Due to the the potentially devastating complications of the femoral neck fracture in young patients (avascular necrosis, nonunion, and malunion), the neck fracture should be treated first followed by the shaft.
Current recommendations involve treating the neck with a sliding hip screw versus cannulated screws followed by intramedullary nailing of the femoral shaft.
- A through-knee disarticulation has been shown to have what advantage over a traditional above-knee (transfemoral) amputation?
- Decreased rate of prosthesis adjustment
- Less postoperative time to final prosthesis fitting
- Decreased neuroma formation
- Decreased rate of revision
- Less energy expenditure with ambulation CORRECT ANSWER: 5
A through-knee disarticulation has been shown to have decreased energy expenditure with ambulation, improved limb proprioception, improved sitting capabilities, decreased hip joint flexion contracture incidence, and improved lever arm for mobilization.
Knee disarticulation is also recommended in children to prevent overgrowth of the distal femur which may be seen in transfemoral amputations (if the physis remains open). No difference in prosthesis fitting has been shown between transfemoral amputation and through-knee disarticulation. The referenced
paper by Pinzur et al is a excellent review of knee disarticulation, from technique to outcomes.
- A 65-year-old female presents with the injury seen in Figures A and B after a motor vehicle collision. She is hemodynamically unstable and undergoes emergent pelvic supra-acetabular external fixation followed by laparotomy. She is now hemodynamically stable and cleared for surgery. She has no evidence of neurologic deficit on examination. Which of the following factors is a relative contraindication to open reduction and plating of her posterior pelvic injury from an anterior approach?
- Sacral fracture
- Prior laparotomy
- Supra-acetabular external fixtator
- Parasymphyseal fractures
- Ipsilateral proximal femur fracture CORRECT ANSWER: 1
An anterior approach to the sacroiliac (SI) joint is indicated with displaced SI joint dislocations that cannot be reduced with closed or percutaneous techniques. One contraindication to anterior exposure of the SI joint is comminuted sacral fracture patterns.
Posterior pelvic ring injuries that are unable to be reduced by closed techniques may require open reduction via anterior or posterior approaches. Relative contraindications to anterior approach include comminuted sacral fractures, morbid obesity, iliac wing external fixation, and ipsilateral diverting colostomy. In the presence of a comminuted sacral fracture, aggressive medial dissection would be required and would place the L5 nerve root at risk.
Simpson et al describe their initial results with open reduction and internal fixation of the SI joint via an anterior exposure in a series of 16 patients. They note that sacral alar comminution is a contraindication to the anterior approach
Jones provides an overview of the operative treatment of posterior pelvic ring injuries. He demonstrates reduction and fixation techniques via both anterior and posterior exposures.
Incorrect Answers:
Answer 2: Prior laparotomy is not a contraindication to open anterior approach if the bowel is in continuity and there is no evidence of wound infection Answer 3: Supracetabular external fixation does not interfere with anterior approach to the SI joint
Answer 4: Anterior pelvic ring injuries such as parasymphyseal fractures do not affect the choice of approach
Answer 5: An ipsilateral proximal femur fracture does not affect the choice of approach
- A 45-year-old homeless hemophiliac male presents with chronic tibial osteomyelitis. Which of the following factors has been shown to predict a better prognosis?
- Polymicrobial infection
- Use of external fixation
- Infection with Methicillin-resistant Staphylococcus aureus
- Metaphyseal infection
- Contralateral lower extremity amputation CORRECT ANSWER: 4
Success in the treatment of chronic tibial osteomyelitis is dependant on various factors including patient factors (immunocompetency of patient, nutritional status), injury factors (severity of injury as demonstrated by segmental bone loss), and infection factors (the extent and location of infection – metaphyseal infections heal better than mid-diaphyseal infections).
Cierny’s article states that factors affecting prognosis and treatment include: residual foreign materials and/or ischemic and necrotic tissues, host compromise, inappropriate antibiotic coverage, and the lack of patient cooperation or desire.
The second referenced article by Cierny reviews the significant increase in success over the last 20 years in treating infected tibial nonunions, due to pharmacological and technological advances. He reports an increase in limb salvage from 78% to 93% with modern protocols.
Incorrect Answers:
Answer 1: Polymicrobial infection portends a worse prognosis than a single organism.
Answer 2: External fixation has not been shown to improve outcomes in chronic osteomyelitis.
Answer 3: MRSA infections are a risk factor for poor outcomes.
Answer 5: Contralateral extremity amputation increases the risk of poor outcomes.
- A 30-year-old male sustains the injury seen in Figure A after a motor vehicle collision. Which of the following is the most likely complication at 2-year follow-up?
- Avascular necrosis
- Hip instability
- Malunion
- Chondrolysis
- Ipsilateral medial knee degenerative changes CORRECT ANSWER: 1
Figure A shows a displaced femoral neck fracture. Avascular necrosis (AVN) and nonunion are the two most common complications after femoral neck fractures in the young adult.
Dedrick et al found nonunion of the fracture site was observed in 20% and avascular necrosis in 36% of young patients with femoral neck fractures. In addition, they reported that of patients with subcapital fractures, 83% developed nonunion or avascular necrosis, compared to 21% with a more distal femoral neck fracture.
- A 40-year-old male laborer sustained a fall from height and has isolated pelvic pain. He is otherwise hemodynamically stable. A radiograph is shown in Figure A. A stress examination under anesthesia does not show any further anterior diastasis or posterior pelvic ring displacement. Computed tomography reveals no asymmetry of the sacroiliac joints. What is the most appropriate management of this injury?
- protected weight-bearing and pain control
- open reduction and internal fixation
- skeletal traction followed by open reduction and internal fixation
- pelvic external fixation
- pelvic external fixation followed by sacroiliac screws CORRECT ANSWER: 1
This patient sustained an open-book pelvic fracture with a pubic symphysis diastasis of less than 2.5cm. From the Young and Burgess classification, he has anteroposterior compression (AP) type 1 injury. Treatment of this is protected weight-bearing and symptomatic treatment. Stress examination can be utilized in order to ensure that the injury is, in fact, a APC-1 injury, and not a more severe posterior injury that would require operative intervention.
- A 40-year-old woman is involved in motorcycle accident 2 hours ago and sustains an isolated right leg injury shown in Figure A. She has dopplerable posterior tibial and dorsalis pedis artery signals with less than 2 second capillary refill as shown in Figure B. Sensation is intact in the distribution of the tibial nerve but decreased in the distribution of the peroneal nerve. She is cleared by the general surgery trauma team to go to the operating room for treatment of her leg. What is the most appropriate Gustilo classification and initial treatment for her injury?
- Gustilo 3A with spanning external fixation and delayed definitive fixation with soft tissue coverage
- Gustilo 3A with immediate medial and lateral plating followed by delayed soft tissue coverage
- Gustilo 3B with spanning external fixation and delayed definitive fixation with soft tissue coverage
- Gustilo 3B with immediate medial and lateral plating followed by delayed soft tissue coverage
- Gustilo 3C with spanning external fixation and delayed definitive fixation with soft tissue coverage
CORRECT ANSWER: 3
The injury described above is a Type IIIB injury as per the Gustilo and Anderson classification. Type I injuries are low energy and have small soft- tissue wounds (usually <1 cm in length) with minimal contamination. Type II injures have a wound >1 cm in length, but do not have extensive soft-tissue
damage, flaps, or avulsions. Type IIIA injuries are associated with soft-tissue damage secondary to high-energy trauma but have adequate soft-tissue coverage. Type IIIB injures exhibit severe periosteal stripping and bone exposure, often associated with massive contamination. These often require treatment with soft-tissue coverage procedures. Type IIIC fractures require vascular repair. The risk for infection in this scenario is as high as 44%, so placing definitive plate fixation is contraindicated when future debridement and soft tissue coverage procedures will be needed. External fixation provides excellent stability, provisional skeletal alignment, and minimal additional soft tissue injury.
- A 35-year-old male laborer falls off a ladder and sustains the injury shown in Figures A and B. He has a 2 cm laceration over the medial ankle with exposed bone and a normal neurovascular exam. What is the recommended initial treatment?
- Immediate open reduction and internal fixation
- Closed reduction and casting
- Irrigation and debridement and external fixation
- Irrigation and debridement and splinting
- Amputation CORRECT ANSWER: 3
Severe pilon fractures are generally the result of high energy trauma leading to bony comminution of the articular and metaphyseal bone. They are usually associated with significant soft tissue injury which prevents immediate definitive open reduction and internal fixation. In this situation, due to the soft tissue injury and open fracture, initial treatment should consist of irrigation and debridement and stabilization with external fixation. Definitive management such as open reduction and internal fixation is performed once the soft tissue swelling has improved and there is no evidence of infection. Sirkin et al published their results of a staged protocol for complex pilon injuries. Their data suggests the historically high rates of infection associated with ORIF of pilon fractures are likely due to attempts at immediate fixation through swollen, compromised soft tissues.
- A 27-year-old female sustains injuries to the left femur and ipsilateral tibia shown in Figures A and B following an ATV accident. Her injury severity score (ISS) is 27 for her musculoskeletal and abdominal injuries. Her left limb is neurovascularly intact and there are no signs of compartment syndrome. What is the most appropriate definitive management?
- Intramedullary nailing of the tibia and femur
- External fixation of the tibia and femur
- Balanced skeletal traction
- Circular external fixation of the tibia and intramedullary nailing of the femur
- Uniplanar external fixation of the tibia and intramedullary nailing of the femur
CORRECT ANSWER: 1
Polytrauma patients with ipsilateral femoral and tibial fractures (floating knee injuries) often require aggressive hemodynamic resuscitation and immediate stabilization via external fixation following tenets of damage-control orthopaedics. However, goals for definitive management of these fractures include obtaining anatomic alignment, early joint range of motion, and early weightbearing. If the floating knee injury is an isolated injury and the patient is hemodynamically stable then immediate intramedullary nailing of the tibia and femur is acceptable. Of the choices listed, intramedullary nailing of both the femoral and the tibial fracture is the optimal form of fixation for these transverse fractures. The technique of antegrade intramedullary nailing of both the femur and the tibia has been well described. Retrograde femoral nails and antegrade tibial nails can be advantageous because it allows simultaneous surgical setup for both the femoral and the tibial fracture.
- All of the following are true statements regarding compartment syndrome in the pediatric patient EXCEPT:
- Increasing analgesic requirement is an important indicator for the diagnosis of compartment syndrome in children
- Duration of compartment syndrome prior to treatment is the most important variable in determining the outcome
- Mechanism of injury is the best predictor of compartment syndrome development
- Traditional hallmarks of adult compartment syndrome may be more challenging to detect in pediatric compartment syndrome
- Careful patient positioning and the use of prophylactic fasciotomy are methods of preventing compartment syndrome
CORRECT ANSWER: 3
Compartment syndrome can often be difficult to diagnosis in the pediatric patient. Mechanism of injury is not the best predictor of compartment syndrome development or diagnosis in pediatric patients. It is important to note that functional outcome following compartment syndrome in patients is inversely related to the duration of elevated tissue pressures before surgical fasciotomy.
Level 4 evidence by Bae et al reviewed 33 children with compartment syndrome. They found that all 10 compartment syndrome patients that had access to nurse or patient controlled analgesia (PCAs), during their initial evaluation, demonstrated an increasing requirement for pain medication.
Matsen et al reviewed 24 children with compartment syndrome with the most common causes being fracture, vascular injury, and tibial osteotomy. The study concluded that is imperative that a compartment syndrome be identified and treated as promptly as possible.
- A 27-year-old male is involved in a motor vehicle accident and sustains the injury shown in Figures A through E. The articular surface is depressed 2 mm while there is 3 mm of condylar widening. Valgus instability of the knee is noted. Which of the following is most important to long-term success in surgical treatment of this case?
- Restoration of joint stability
- Repair of associated meniscal pathology
- Surgical fixation within 48 hours of injury
- Correction of the articular depression
- Tibial condylar diastasis < 3 mm CORRECT ANSWER: 1
The clinical presentation and imaging studies are consistent with a tibial plateau fracture. Restoration of joint stability has been shown to be the strongest predictor of long term outcomes.
Honkonen reviewed 131 tibial condyle fractures and determined that articular stepoff <3mm and tibial widening <5mm did not negatively effect outcomes. In contrast, 70% of knees with moderate to severe malalignment went on to functionally unacceptable outcomes. They suggested operative fixation for all medial uni and bicondylar fractures, any lateral fractures with >5 degrees of
valgus tilt, >3mm of articular depression, >5mm of condylar widening, or >5 degrees of valgus malalignment.
In the Marsh et al JAAOS symposium review, the authors noted that fractures with up to 10mm of articular depression and joint stability obtained acceptable functional outcomes. They also cited a 20 year follow-up which indicated that articular step-off alone was not a predictor of poor long-term results. More importantly, when instability is present with other factors, including step-off and central depression, poor results followed.
Illustrations A and B show the intraoperative films. Illustration C reviews the Schatzker classification system.
- A 45-year-old male is involved in a motor vehicle accident and sustains the injury shown in Figures A-D. Which of the following is the most appropriate approach for surgical fixation of this fracture?
- Ilioinguinal
- Kocher-Langenbeck
- Stoppa
- Stoppa with lateral window
- Extended iliofemoral CORRECT ANSWER: 2
The images demonstrate a posterior column acetabular fracture. These are best surgically treated with a Kocher-Langenbeck approach, which allows access to the posterior column and posterior wall. Figure A shows disruption of the ilioischial line with an intact iliopectineal line which is diagnostic of this fracture pattern. The CT image in Figure D shows the characteristic horizontal (coronal) orientation of the column fracture when viewed on an axial CT. Illustration A shows the radiographic landmarks used in diagnosing acetabular fractures. Illustrations B and C show the orientation of column and wall fractures respectively. Ilioinguinal and Stoppa approaches allow access for
anterior column fixation and symphysis fixation respectively. The extended iliofemoral approach can be used to treat both column injuries, but has high rates of post-operative heterotopic ossification.
- A 33-year-old male presents 9 months after a fall from 15 feet. He complains of continued pain over his left arm and you elicit pain and gross movement with palpation of his humerus. Infectious workup is negative and a radiograph is shown in Figure A. What is the most appropriate next step in his management?
- Reassurance and appropriate followup
- Sarmiento bracing
- Use of a bone stimulator
- Exchange humeral nailing
- IM nail removal, open reduction internal fixation with bone grafting CORRECT ANSWER: 5
This patient has developed a hypertrophic non-union of his left humerus following IM nailing. This will not go on to union without surgical intervention. McKee et al. reviewed 21 cases of humeral nonunion after failed intramedullary humeral nails. Although technically difficult, open reduction internal fixation with plating and bone grafting was more successful in union in 9/9 cases, vs exchanged humeral nailing which was only successful in 4/10 cases. Seven of the nonunions were atrophic, 2 were hypertrophic in the ORIF group. The authors conclude that the extent of humeral bone loss after failure of primary humeral nailing makes open reduction internal fixation with compression and bone grafting the most acceptable method of treating this problem.
It should be noted, however, that the use of bone grafting in the presence of a hypertrophic nonunion is controversial and has not been definitively proven in the literature to increase healing rates.
- A 47-year-old male sustains an isolated posterior wall acetabulum fracture after a motor vehicle collision and undergoes open reduction and internal fixation. Post-operative radiographs are shown in Figure
- Which of the following has been shown to correlate most closely with good outcomes following ORIF of posterior wall fractures?
- Degree of displacement seen on preoperative AP pelvis view
- Degree of displacement seen on preoperative Judet views
- Degree of displacement seen on preoperative pelvic CT scan
- Degree of displacement seen on postoperative Judet views
- Degree of displacement seen on postoperative pelvic CT scan CORRECT ANSWER: 5
Moed et al performed a study to determine the clinical outcome in patients in whom a displaced fracture of the posterior wall of the acetabulum had been treated by open reduction and internal fixation. They were able to show good to excellent clinical results for patients who underwent anatomic reduction and internal fixation of posterior wall acetabulum fractures as assessed using radiographs. Fractures in elderly patients and patients who sustained extensive comminution were more likely to have worse clinical result.
In a separate study, Moed et al. evaluated the results of 67 patients who underwent ORIF of a posterior wall fractures by assessing the accuracy of postoperative AP pelvis, obturator oblique films, iliac oblique films, and CT scans. They found that postoperative pelvic CT scan was the most accurate way to judge final fracture reduction and was able to pick up residual fracture displacements that were not seen on postoperative plain radiographs. They concluded that the accuracy of reduction as assessed on postoperative CT scan was the most reliable indicator of clinical outcomes.
- A 68-year-old male sustains the humeral shaft fracture shown in Figures A and B. Upon presentation, he is unable to extend his thumb, fingers, and wrist. After 4 months of non-operative management, the fracture has healed, but his physical exam is unchanged. What is the next most appropriate step in management?
- EMG and nerve conduction tests followed by possible surgical exploration
- Continued observation
- Immediate surgical exploration
- Shoulder MRI
- CT scan of the humerus CORRECT ANSWER: 1
The clinical presentation is consistent for a residual radial nerve palsy 4 months after a humeral shaft fracture. An EMG is indicated at this time to evaluate the status of the nerve recovery.
A radial nerve injury which occurs during a humeral shaft fracture or after bracing is not an indication for immediate exploration. Most often, the nerve function returns without surgical intervention. An EMG should be performed at 3-5 months to evaluate the status of the nerve recovery. If fasciculations are present, then this represents recovery, and observation should be continued. If fibrillations are present, this represents denervation, and surgical exploration should be considered.
Pollock et al followed 24 humeral-shaft fractures with associated radial-nerve injuries, 2 of which required open exploration and all recovered. They recommend careful observation for return of nerve function and exploration at 3.5-4 months after injury if there is still no clinical or EMG evidence of recovery.
Bostman et al reviewed 59 immediate and 16 secondary radial nerve palsies and no support emerged for routine early exploration in either group.
Figures A and B show an oblique fracture at the junction of the middle and distal 1/3 of the humeral shaft.
Illustration A shows the relative close position of the radial nerve to the humerus at the midlevel of humerus, and why it is at risk with a humerus shaft fracture.
- A 56-year-old male sustains a Type IIIB open, comminuted tibial shaft fracture distal to a well-fixed total knee arthroplasty that is definitively treated with a free flap and external fixation. Nine months after fixator removal, he presents with a painful oligotrophic nonunion. Laboratory workup for infection is negative. Passive knee range of motion is limited to 15 degrees. What is the most appropriate treatment for his nonunion?
- Knee manipulation under anesthesia
- Cast immobilization and use of a bone stimulator
- Unilateral external fixation
- Intramedullary nailing
- Compression plating CORRECT ANSWER: 5
At 9 months, observation is no longer an option, as the fracture is not healing and is adjacent to a arthrofibrotic joint. Plate osteosynthesis has been shown to be an effective method of treatment for patients who have had an open fracture of the tibia that has failed to unite after external fixation and/or immobilization in a cast.
Wiss et al reported a series of fifty tibial non-unions with a similar clinical scenario. He reported that, with compression plating, 92% of the nonunions healed without further intervention. In their study, 39/50 patients, had autogenous bone grafting in addition to compression plating.
- What muscles are responsible for the most common deformity after antegrade intramedullary nailing for a subtrochanteric femur fracture?
- Hip abductors and iliopsoas muscle
- Hip internal rotators and iliopsoas muscle
- Quadriceps and iliopsoas muscle
- Hamstring and iliopsoas muscle
- Quadriceps and hip adductors CORRECT ANSWER: 1
The most common deformity after antegrade nailing of a subtrochanteric femur fracture is varus and procurvatum (or flexion). This is caused by the hip abductors and iliopsoas pulling the proximal fragment into abduction and flexion, while the distal fragment is pulled into adduction from the adductors.
The reference by French et al is a review on 45 patients with subtrochanteric fractures treated with cephalomedullary interlocked nailing. Based on femoral neck-shaft angle, 61% of the fractures were reduced in at least 5º varus. The authors attributed this malalignment to failure to counteract muscle forces acting on the proximal fragment, combined with the adducted position of the distal femur during portal creation.
The reference by Ricci et al is a report of 403 femoral shaft fractures treated with intramedullary nailing. Patients with proximal femoral shaft fractures were found to have the highest incidence of malalignment. The most common deformity in this group was varus, followed by procurvatum (or flexion).
- A 34-year-old male has persistent anterolateral ankle pain after a snowboarding injury 1 week ago and is unable to bear weight. Three good quality radiographic views of the ankle are negative for fracture or other abnormalities. What is the next best step in management?
- Short leg cast application
- Bone scan
- MRI of ankle
- Diagnostic injection
- Repeat radiographs CORRECT ANSWER: 3
Fractures of the lateral process of the talus are frequently overlooked and should always be considered in the differential diagnosis of ankle pain in snowboarders. The common mechanism for fracture is dorsiflexion of the ankle and eversion of the hindfoot.
The reference by Vlahovich et al is a case report of a talus fracture in a snowboarder and argues the importance of CT scans in evaluating these injuries as radiographs may fail to show the injury and amount of displacement and comminution of the fracture.
The reference by Tucker et al is a review of the literature which emphasized the importance of early diagnosis to avoid long term complications. They recommend short leg casting for nondisplaced fractures and surgery for displaced or comminuted fractures.
- A 40-year-old male who sustained an open pilon fracture 2 weeks ago is scheduled for a below-the-knee amputation (BKA). What laboratory value is the best predictor for wound healing?
- serum albumin level
- total protein level
- calcium levels
- C-reactive protein
- ESR
CORRECT ANSWER: 1
Based on the choices above, the most important predictor of wound healing is the serum albumin level.
Wound healing is based on several factors, which include the vascular status, the immune status, and the nutritional status of the patient. Some important clinical findings include an ankle brachial index (ABI) > 0.45, a total lymphocyte count > 1500/mm3 and a serum albumin > 3.0 g/dL.
Kay et al. discuss the importance of the nutritional status in wound healing after lower extremity amputation procedures. They found eleven of 25 patients who were malnourished sustained either local or systemic complications postoperatively. They recommend that patients should undergo nutritional screening prior to elective lower extremity amputations, to help optimize their wound healing.
Incorrect Answers
Answer 2: While total protein is a marker of nutritional status, it is not as sensitive as the serum albumin for wound healing potential.
Answers 3, 4, 5: Calcium levels, C-reactive protein and ESR are not markers of wound healing
- A 30-year-old male sustains the injury shown in figure A and undergoes successful open reduction and internal fixation. Which of the following radiographic features is a good prognostic factor for this injury?
- Talar dome subchondral lucency
- Talar dome subchondral sclerosis
- Diffuse osteopenia
- Associated medial malleolus fracture
- Talar lateral process fracture CORRECT ANSWER: 1
Figure A demonstrates a talar neck fracture. A subchondral talar lucency at approximately 6 weeks postoperatively indicates revascularization of the talus and is a good prognostic factor for this injury (aka Hawkins' Sign and is exhibited by the arrows in Illustration A).
The talar neck blood supply is tenuous and is susceptible to avascular necrosis. The reference by Hawkins classified talar neck fractures and correlated the incidence of avascular necrosis with the degree of displacement and severity of the fracture: Type I = Nondisplaced vertical fractures (AVN 10%). Type II = Displaced with subtalar dislocation/subluxation (AVN > 40%). Type III = Displaced with talar body dislocation (AVN >90%). Type IV = Displaced with talar head subluxation and body extrusion (AVN 100%).
The reference by Canale et al reviewed long term outcomes after ORIF of talar neck fractures, and they found that good or excellent results were seen in only 59%. Salvage procedures such as triple arthrodesis, tibiocalcaneal fusion, and dorsal beak resection of the talar neck all resulted in a high percentage of satisfactory results, but talectomy did not.
- A 45-year-old male undergoes open reduction internal fixation for a displaced olecranon fracture as shown in Figure A. What is the most common complication for this type of fixation?
- anterior interosseous nerve palsy
- osteomyelitis
- implant failure
- symptomatic implants
- avascular necrosis CORRECT ANSWER: 4
The most common complication of an olecranon fracture treated with tension band wiring is symptomatic implants. This is largely related to the
subcutaneous nature of the olecranon.
Macko and Szabo encountered a high incidence of complications related to the technique of tension-band wire fixation of displaced fractures of the olecranon in a five-year retrospective study of twenty patients. They reported that the most frequent complication of this construct is symptomatic prominence of the hardware which is usually due to improper seating at the time of surgery.
Other complications reported include loss of fixation, skin breakdown, and infection.
Hume and Wiss reported on 41 patients randomized to tension band wiring and plate fixation. They note that there was no difference in regards to elbow motion postoperatively, but plating required more operative time. Plating of these fractures resulted in a significantly increased rate of good clinical and radiographic results.
Figure A shows an AP and lateral radiograph of an olecranon fracture treated with tension-band wiring.
Incorrect Answers:
Answer 1: This is a possibility with overpenetration of the wires through the anterior ulnar cortex, but not a common finding.
Answer 2: This is not a common finding with this treatment method. Answer 3: Although implant failure is a possibility with this technique, this is not the most common complication.
Answer 5: Avascular necrosis of the proximal ulna is not a common complication of this injury or treatment method.
- Which of the following percentages of normal circulating blood loss would first result in a patient with tachycardia and a narrowed pulse pressure?
1. 5%
2. 10%
3. 25%
4. 40%
5. 50%
CORRECT ANSWER: 3
Hemorrhagic shock is divided into four classes - class I is <15% loss and shows compensation for the blood loss (no tachycardia/hypotension) and is
treated with crystalloid replacement as necessary. Class II is a loss of 15-30% and is the target of this question. In this class, vasoconstriction leads to maintenance of perfusion pressure and tachycardia helps maintain cardiac output in the face of a decreased overall volume. The vasoconstriction leads to an elevated diastolic pressure, which is the cause of the narrowed pulse pressure (the difference between systolic and diastolic). Treatment remains control of ongoing bleeding and crystalloid replacement. Class III is a 30-40% loss, and is the first stage where hypotension is present. Signs of end organ hypoperfusion, such as confusion and decreased urine output, is seen.
Treatment is crystalloid replacement and blood product replacement. Class IV is a loss of >40% and is often fatal.
Hak reviews the ATLS classification of hemorrhage in his review article on pelvic fractures and bleeding.
Illustration A shows the hemorrhagic shock class table.
- A 47-year-old male sustains the closed injury seen in Figures A and B after failing to land a motorcycle jump. A post-reduction radiograph is seen in Figure C. Which of the following is the most appropriate treatment at this time?
- Definitive closed treatment
- Addition of percutaneous pins
- Open reduction and internal fixation
- Tibiotalocalcaneal arthrodesis
- Primary subtalar arthrodesis CORRECT ANSWER: 3
The clinical presentation and imaging studies are consistent with Hawkins II talar neck fracture, which by definition is a displaced talar neck fracture with subtalar dislocation/subluxation. Despite achieving an adequate reduction initially (shown in Figure C), there is no role for closed treatment of these unstable injuries, and the treatment of choice is open reduction and internal fixation.
The referenced article by Bibbo et al describes these injuries: 32% of subtalar joints are irreducible to closed means (half with soft tissue block, half with bony block to reduction), 88% have co-existing injuries of the ipsilateral foot, 89% have radiographic subtalar arthrosis at 5 years (62% symptomatic).
- A 40-year-old male suffers the isolated injury shown in figure A with no associated fractures. What joint is dislocated in this radiograph?
- Tibiotalar
- Talonavicular
- Calcaneocuboid
- First metatarsophalangeal
- First tarsometatarsal CORRECT ANSWER: 2
The radiograph shows a subtalar (talocalcaneal) dislocation with a talonavicular dislocation as well. If subtalar dislocations also involve dislocation of the articulations at both the talonavicular and ankle (tibiotalar) joint, a talar extrusion is seen. Subtalar dislocations are associated with high energy, open (25%), and irreducible (33%) fractures. Medial dislocations account for 65%, and reduction is blocked by the extensor digitorum brevis (EDB). Lateral dislocations that are irreducible are blocked by the posterior tibialis, FHL, and FDL tendons. These dislocations often require emergent open reductions, tendon relocation, and stabilization.
Bibbo et al reported clinical and radiographic outcome on 25 patients and the majority of these patients had radiographic degenerative changes at 5 years follow up.
The review reference by Bohay and Manoli covers subtalar joint dislocations and notes the importance of anatomic reduction to achieve optimal outcomes.
- Increased hip intracapsular pressures can lead to diminished femoral head perfusion. Which of the following limb positions has been shown to create the lowest intracapsular hip pressures after a femoral neck fracture?
- flexion and internal rotation
- extension and external rotation
- flexion, abduction, and external rotation
- extension, adduction, and internal rotation
- there are no differences in hip pressures with any position CORRECT ANSWER: 3
After a femoral neck fracture, patients often present with their injured hip in a flexed, abducted, and externally rotated position due to decreased pain from minimization of the capsular distension from fracture hematoma (if the capsule isn't disrupted).
In the referenced study by Bonnaire et al, extension and internal rotation had the highest intracapsular pressure. They found that the greatest decrease in pressure was found with flexion, abduction and external rotation. This is a possible etiology to the common position in which these patients will present to the emergency room. Traction was shown to increase pressure in the joint capsule.
Incorrect Answers: 1,2,4,5: All of these positions have increased pressure as compared to the position of flexion, abduction, and external rotation.
- Which of the following elbow injuries as found in Figures A-E best characterizes the radiographic "double-arc" sign?
- Figure A
- Figure B
- Figure C
- Figure D
- Figure E CORRECT ANSWER: 3
Figure C and Illustration A (below) demonstrate the radiographic "double-arc" finding.
McKee et al described a unique "shear fracture of the distal articular surface of the humerus" which involved coronal fractures of the capitellum and a portion of the trochlea. He described the characteristic radiographic finding as the "double-arc sign" which represents the subchondral bone of the displaced capitellum and lateral trochlea ridge.
Incorrect Answers:
Figure A shows a radial head fracture. Figure B shows an elbow dislocation.
Figure D shows a pediatric lateral condyle fracture.
Figure E shows a pediatric medial epicondyle apophyseal avulsion fracture.
- A 31-year-old male sustains an ipsilateral displaced transverse acetabular fracture, pubic rami fractures, and a sacroiliac joint dislocation. What structure should be reduced and stabilized first?
- Pubic rami
- Posterior column
- Anterior column
- Sacroiliac joint
- Quadrilateral plate CORRECT ANSWER: 4
In an ipsilateral unstable pelvic ring and acetabular fractures, the pelvic ring injury must be initially stabilized in order to reduce the acetabular fracture to a stable base.
The referenced article by Matta reviewed 259 patients with acetabular fractures treated within 21 days of injury and found that the hip joint can be preserved and post-traumatic osteoarthrosis can be avoided if an anatomical reduction is achieved.
- A 35-year-old patient sustains a left calcaneus fracture. Which of the following fractures has the highest risk of post-traumatic arthritis?
- Male patient, Sanders Type III fracture, treated with ORIF
- Male patient, Sanders Type II fracture, treated with ORIF and bone graft
- Female patient, workers compensation, Sanders Type I fracture, treated non-operatively
- Female patient, Sanders Type II fracture, treated non-operatively
- Female patient, workers compensation, Sanders Type II fracture, treated with ORIF
CORRECT ANSWER: 1
The patient most likely to require late subtalar fusion for post-traumatic arthritis is a male patient with a Sanders Type III fracture treated with ORIF.
The Sanders classification system seems to remain prognostic for injury severity. It has been shown that greater articular injury is associated with greater risk of arthritis, irrespective of patient gender and occupation. In current reviews, type III fractures are 6.5 times more likely to develop PTA and 4 times more likely to require an ST fusion than Sanders type II fractures. Post-traumatic arthritis of the subtalar joint is a common complication.
Successful salvage can be achieved with a subtalar arthrodesis.
Howard et al reported on complications following management of displaced intra-articular calcaneal fractures. 469 patients were randomized into ORIF and non-operative groups. ORIF patients were more likely to develop complications (overall 25% vs 18% of non-op) with fractures of increasing severity (Sanders
IV) developing more complications regardless of management strategy.
Sanders et al. reviewed the operative treatment of displaced intra-articular calcaneal fractures. Based on the results of this comparative analysis, the Sanders classification remains prognostic; after a minimum of 10 years, type III fractures were 4 times more likely to need a fusion than type II fractures.
Illustration A shows the Sanders classification. Incorrect Answers:
Answers 1,3-5: Functional outcomes and arthritis will be determined by the
amount of cartilage damage. This is directly correlated with initial injury, fracture pattern (Sanders) and reduction techniques. Worker compensation are associated with worse functional outcome scores, not post-traumatic arthritis.
- A 34-year-old male sustains the injury seen in Figure A after being struck by a truck while crossing the street. Upon arrival in the trauma bay, he is initially tachycardic and hypotensive, but after application of a pelvic sheet and administration of intravenous fluids, his vitals normalize. Radiographs of his neck, chest, and pelvis are then obtained after pelvic sheeting; his new pelvis radiograph is shown in Figure B. Which of the following is the most appropriate next step?
- CT scan of chest, abdomen, pelvis
- Immediate sheet removal in exchange for a pelvic binder for added stability
- Immediate external fixator placement in the emergency room
- Pelvic arterial embolization
- Definitive open reduction internal fixation CORRECT ANSWER: 1
The clinical presentation is consistent for an anterior-posterior compression (APC) pelvic injury. The patient has been hemodynamically stabilized and the next step in treatment is a CT scan of chest, abdomen, pelvis to rule out other life threatening injuries.
APC injuries are highly unstable and can produce high mortality rates by pelvic exsanguination. Pelvic venous bleeding is far more common than arterial
bleeding. The initial treatment intervention should be to reduce the pelvic volume by any means possible (sheet, binder, ex-fix) to reduce further intrapelvic bleeding. After application of the pelvic sheet in the above scenario, the patient has become stabilized and can undergo further workup to rule out other life threatening injuries.
Illustration A is a useful algorithm for treating trauma patients with pelvic injuries.
Incorrect Answers:
Answer 2: If the patient is hemodynamically stable there is no need for immediate pelvic fixation.
Answer 3: If the patient is hemodynamically stable there is no need to change to a pelvic binder.
Answer 4: If pelvic volume reduction failed to stabilize the patient, and other sources of hemorrhage, such as chest or abdomen were ruled out with radiographs or FAST exam, pelvic angiography and embolization would be the next appropriate step to treat potential pelvic arterial injury.
Answer 4: There is no role for acute definitive open reduction internal fixation.
- A 30-year-old male undergoes successful surgical fixation of a displaced talar neck fracture. Which of the following is the most likely long-term complication even after anatomic reduction and stable fixation is achieved?
- Tibiotalar and/or subtalar arthritis
- Loss of forefoot supination
- Osteonecrosis
- Nonunion
- Infection CORRECT ANSWER: 1
It is important to counsel patients regarding these devastating injuries and their poor prognosis, as osteonecrosis, subtalar and tibiotalar joint degeneration, and talar collapse are not infrequent.
In a series by Lindvall et al, subtalar joint arthritis was reported as more common than osteonecrosis of the talus at 4 year follow-up after fixation. Osteonecrosis of the talus was the next most common complication following surgery.
The referenced article by Vallier et al reported on mid- to long-term follow-up of this patient population. Ten of the twenty-six patients had development of osteonecrosis of the talar body, but only half experienced collapse of the talar dome. All patients with a history of both an open fracture and osteonecrosis experienced collapse. Seventeen of twenty-six patients had posttraumatic arthritis of the tibiotalar joint, and nine of twenty-six had posttraumatic arthritis of the subtalar joint.
Illustrations A and B show a displaced talar neck fracture.
- Valgus intertrochanteric osteotomy with blade plate fixation is the most appropriate treatment for which of the following figures?
- Figure A
- Figure B
- Figure C
- Figure D
- Figure E CORRECT ANSWER: 3
Figure C shows a femoral neck fracture that underwent closed reduction and percutaneous pinning with three screws, that has gone on to femoral neck nonunion. Figure A shows slipped capital femoral epiphysis (SCFE). Figure B shows a subtrochanteric fracture status post intramedullary nailing. Figure D shows a hip labral tear. Figure E shows hip avascular necrosis.
In the case of femoral neck nonunion, a valgus intertrochanteric osteotomy helps to convert any shear force of a vertical fracture line (especially with a high Pauwel's angle) into a horizontal compressive force (Illustration A.)
In their series of 50 patients with femoral neck nonunions, Marti et al treated all patients with Pauwels valgus intertrochanteric osteotomies. At an average followup of seven years, they reported that only 7 patients needed prosthetic replacement. However, nearly half of the patients showed radiographic signs of avascular necrosis (only 3/22 were symptomatic). They conclude that valgus intertrochanteric osteotomy yields good results for young patients with femoral neck nonunions.
- Placing the starting point for an antegrade femoral nail too anterior to the axis of the medullary canal can most commonly lead to what intraoperative complication?
- Nail incarceration
- Loss of locking screw trajectory into the lesser trochanter
- Creation of a recurvatum deformity
- Iatrogenic fracture of the proximal fragment
- Decrease in hoop stresses CORRECT ANSWER: 4
Usage of an anterior starting point that is too anterior leads to creation of significant hoop stresses in the proximal segment, potentially leading to iatrogenic fracture of the proximal segment. The referenced study by Johnson et al reviews the topic of femoral bursting and he notes that even shifting 6mm too far anteriorly can lead to proximal femoral fracture creation. He also reported that overreaming the canal by at least 0.5mm diameter is necessary to decrease hoop stresses throughout the femur, likely due to a mismatch in the radius of curvature of the femur and intramedullary nail.
- A 32-year-old male sustains the closed injury shown in Figure A. He undergoes reamed intramedullary nailing 4 hours after his injury. Postoperative images are shown in Figures B and C. Compared to unreamed nailing, reamed nailing of this injury has been associated with which of the following?
- Decreased infection rate
- Increased need for additional surgeries to obtain union
- Increased infection rates
- Decreased time to union
- Increased compartment syndrome rate CORRECT ANSWER: 4
Reamed nailing of closed tibial shaft fractures has been shown to lead to an earlier time to union without an increased rate of complications when compared to unreamed nailing.
The referenced study by Finkenmeier et al is a randomized controlled study of reamed vs. unreamed nails in open and closed tibia fractures (excluding Grades IIIB and IIIC). They found that the use of reamed insertion of IM nails for the treatment of closed tibia fractures lead to an earlier time to union without increased complications. The authors reported no differences in infection rate, compartment syndrome rate, or percent needing additional surgeries to obtain union. More secondary procedures were needed with unreamed nails in closed fractures only. Increased rates of interlocking screw failure were seen if smaller screws were needed for smaller, unreamed nails.
The referenced study by Keating et al reported that reaming for open tibia fractures is safe, with time to union and nonunion rate increasing with increased soft tissue injury as classified by the Gustilo-Anderson classification.
Bhandari et al conducted a multicenter, blinded randomized trial of 1319 adults in whom a tibial shaft fracture was treated with either reamed or unreamed intramedullary nailing. When comparing outcomes in open and closed injuries at twelve months, they found a benefit for reamed intramedullary nailing in patients with closed fractures, but found no difference between approaches in patients with open fractures.
Figure A shows a tibial shaft fracture, with intramedullary fixation shown in Figures B and C.
- Which of the following tibial injuries is most commonly treated with staged open reduction and internal fixation with free flap soft tissue reconstruction?
- Type IIIB intra-articular distal tibia fracture
- Type IIIB segmental midshaft tibia fracture
- Type IIIB transverse midshaft tibia fracture
- Type IIIB Schatzker I proximal tibia fracture
- Type IIIC Schatzker IV proximal tibia fracture CORRECT ANSWER: 1
By definition, with Type IIIB injuries, there is exposed bone after debridement which will require a local or a free flap for coverage. Distal third IIIB tibial shaft fracture are unique in that they usually require a free flap or reverse sural rotational flap to obtain adequate coverage. As stated in Skeletal Trauma, "As local donor muscles in the distal third of the tibia are almost non-existent, closure of an open plafond fracture, or any extensive Type IIIB injury in this area will usually require free tissue transfer. The primary options are latissimus dorsi or rectus abdominus for large defects, and gracilis for smaller wounds." In addition to the flaps mentioned here, others, including fasciocutaneous flaps and radial forearm flaps, are also utilized with success in this area.
Typically, treatment of Type IIIB tibial shaft fractures should be staged. Initially tetanus prophylaxis, antibiotics with gram negative and positive coverage, and application of an external fixator with repeat I&D’s are employed for immediate fracture care. Plating is usually required in the presence of significant intra-articular fracture involvement.
Incorrect Answers: Typically, proximal third tibia fractures requiring soft tissue coverage can be treated with a gastrocnemius rotation flap and middle third tibia fractures with soft tissue defects can be reliably covered with a soleus
rotation flap. Therefore, a free flap is less commonly indicated in the proximal and middle tibia.
- Which of the following variables has not been shown to be increased in patients who sustain bilateral femoral shaft fractures as compared to patients with unilateral femoral shaft fractures?
- Hypotension upon initial evaluation
- Mortality
- Rib fractures
- Open skull fractures
- Pelvic fractures CORRECT ANSWER: 3
Bilateral femur fractures have not been shown to have increased rates of thoracic/chest wall injury. They have been shown to have increased rates of initial hypotension, mortality, open skull fractures, and pelvic fractures.
Due to their high-energy nature, bilateral femur fractures have increased rates of initial hypotension or hemodynamic instability, mortality, head injuries, abdominal injuries, pulmonary injuries, and other orthopaedic injuries.
Copeland et al. performed a retrospective analysis using their trauma registry data on consecutive blunt trauma patients with unilateral (800 patients) or bilateral (85 patients) femoral shaft fractures. Patients with bilateral femoral fractures had a significantly higher Injury Severity Score (30.2 versus 24.5, p
< 0.05) and higher mortality rate (25.9 vs 11.7%, p < 0.014) than patients with unilateral femoral fractures. Bilateral fracture patients also had significantly more closed head injuries, open skull fractures, intra-abdominal injuries requiring surgical intervention, and pelvic fractures. Regression analysis of variables evident on admission revealed a significant correlation between bilateral femoral fractures and death; however, other factors (shock, closed head injury, and thoracic injury) had much stronger correlations with mortality. No increase in risk of thoracic injury was seen with bilateral injuries as compared to unilateral injuries.
Kobbe et al. also reviewed their trauma registry data on 776 patients with unilateral and 118 patients with bilateral femoral shaft fractures. They found that bilateral femur fracture patients has a higher ISS score, higher incidence of delayed pulmonary failure and multiple organ failure, and higher mortality. They also noted that patients with bilateral femoral shaft fractures have
significantly more often severe abdominal injuries as well as severe blood loss which may account for the increased mortality rate.
Incorrect Answers:
Answer 1: Increased rates of hypotension upon admission are seen in the bilateral group.
Answer 2: Increased mortality rates have been reported in bilateral femur patients.
Answer 4: Increased rates of open and closed head injuries are noted in bilateral femur patients.
Answer 5: Increased rates of pelvic and other orthopaedic injuries are reported in the bilateral group.
- A 45-year-old male laborer falls off a 15 foot retaining wall 6 hours ago and sustains an open fracture shown in Figures A through C. He has a normal neurovascular exam. Coronal and sagittal CT scan images are shown in Figures D and E. What is the MOST appropriate next step in management in addition to operative irrigation and debridement?
- ORIF with standard plating of the tibia and fibula
- ORIF with locked plating of the tibia and fibula
- ORIF with standard plating of the tibia and fibula and immediate bone grafting of tibia defect
- External fixation of the tibia, ORIF of the fibula with standard plating, and immediate bone grafting of tibia defect
- External fixation of the tibia CORRECT ANSWER: 5
External fixation of the tibia is the MOST appropriate initial treatment. Some surgeons prefer to defer the fibular fixation until definitive ORIF of the pilon but the common theme is that pilon fractures are treated with external fixation and staged ORIF. If bone grafting of the tibia is necessary it would be performed during the staged ORIF of the tibia.
Sirkin et al reports Level 4 evidence of a staged protocol of immediate (within twenty-four hours) open reduction and internal fixation of the fibula, using a one-third tubular or 3.5-mm DC plate and application of an external fixator spanning the ankle joint. They followed both open and closed pilon fractures that were treated with external fixation until the soft tissue swelling resolved and then underwent ORIF. There was less than an 11% rate of major wound complication or infection in both groups. Their conclusions suggested the historically high rates of wound complication and infection associated with ORIF
of pilon fractures are likely due to attempts at immediate fixation through swollen, compromised soft tissues.
- A 52-year-old diabetic male sustained minor blunt trauma to his left thigh 10 hours prior to presentation. He initially complained of extreme thigh pain with erythema and swelling but rapidly developed bullae and worsening erythema over the affected area along with fever and tachycardia. A clinical photo is shown in Figure A. What clinical factor has been shown to reduce mortality when treating this pathology?
- Presence of MRI findings
- Administration of pressors
- Decreasing time from admission to surgery
- Immediate identification of causative organism
- Location of injury CORRECT ANSWER: 3
The clinical presentation and image shown in Figure A are consistent with necrotizing fasciitis. The most common pathogen is group A Streptococcus, but polymicrobial infection with Gram-positive, Gram-negative, aerobic, and anaerobic bacteria is not uncommon. Necrotizing fasciitis is a surgical emergency and prompt aggressive débridement of all necrotic tissue is critical for survival.
Wong et al demonstrated that early operative debridement (<24 hours) improved the survival rate. The mortality associated with this condition has remained high, with a reported cumulative mortality rate of about 20% in this particular study.
Fontes et al suggest in their review article that early diagnosis and treatment are imperative because necrotizing infections typically spread rapidly and can result in multiple-organ failure, adult respiratory distress syndrome, and death.
- An 82-year-old female sustains a valgus-impacted subcapital femoral neck fracture and undergoes cannulated screw fixation as shown in Figure A. She returns for her first follow-up visit one week later following another fall and now complains of severe hip pain. She is unable to bear weight on the limb, and a new radiograph reveals varus displacement of her fracture. She subsequently undergoes revision fixation but during this procedure, the femoral neck fracture displaces and becomes comminuted. Which is the most appropriate next step in management?
- Skeletal traction
- Revision fixation of the femoral neck fracture
- Hardware removal and placement of a sliding hip screw device
- Hardware removal and hip arthroplasty
- Resection hip arthroplasty CORRECT ANSWER: 4
In the scenario of an elderly patient with questionable fixation into the femoral head and a non-healed femoral neck fracture, proper treatment is arthroplasty. In a physiologically younger patient, reduction and fixation of the fractures (femoral neck and subtrochanteric, if present) with methods such as a valgus producing osteotomy at the level of the subtrochanteric fracture are
recommended.
Figure A shows cannulated screw fixation of a right femoral neck fracture.
The referenced study by Oakey et al evaluated strength of proximal femurs after cannulated hip screw placement and found that placement of an inverted triangle had a higher ultimate load to failure than placement in a standard triangle format (two screws distal).
- A 45-year-old diabetic woman with a gangrenous foot undegoes a Chopart amputation without tendon transfer or lengthening. Which type of deformity is the most likely complication of this procedure?
- Valgus deformity
- Varus deformity
- Equinus deformity
- Cavus deformity
- Planus deformity
CORRECT ANSWER: 3
The Chopart amputation is an amputation of the foot at the level of the calcaneocuboid and talonavicular level. Historically, its use has been criticized because an amputation at this level results in a muscular imbalance with flexor predominance and equinus deformity that eventually leads to stump breakdown. To prevent this complication it should be coupled with Achilles tenotomy (vs. lengthening) as well as transfer of the tibialis anterior insertion to the talar neck.
Advantages of the Chopart amputation include increased limb length and maintenance of heel proprioception that cannot be preserved with more proximal amputations.
Lieberman et al argue in patients with peripheral vascular disease, it is important to preserve as much tissue as possible to preserve maximum function. They recommend that with appropriate care, an amputation at the Chopart (calcaneocuboid-talonavicular) level can give a good functional result.
Figure A shows a lateral radiograph of a Chopart amputation, while Illustration A is a diagram showing this amputation.
- A patient sustained a transverse humeral shaft fracture 6 months ago and presently complains of pain and instability at the area of injury. A plain radiograph is shown in Figure A and on exam there is gross motion at the fracture site. What is the most appropriate definitive treatment?
- ultrasound therapy to nonunion site
- oral bisphosphonates
- open reduction internal fixation with autologous bone graft
- antegrade intramedullary nail
- retrograde intramedullary nail CORRECT ANSWER: 3
Figure A shows an atrophic nonunion of a humeral shaft fracture.
Humeral shaft nonunion is defined as failure of healing after 6 months. The nonunion can be hypertrophic, oligotrophic, or atrophic. The risk of atrophic nonunion increases with bone loss, open fractures, and infection among other factors.
As Jupiter has described, the application of a compression plate to stabilize the fracture fragments with autogenous cancellous bone graft has been successful as long as there is a well vascularized envelope of muscle.
- A 32-year-old ballet dancer sustains a distal radius fracture, and is subsequently closed reduced and casted. She presents 11 months later with the radiograph seen in Figure A, complaining of significant wrist pain. What is the appropriate surgical treatment at this time?
- Distal radius corrective osteotomy
- Total wrist arthrodesis
- Proximal row carpectomy
- Scaphoid excision and four corner fusion
- Interposition arthroplasty CORRECT ANSWER: 1
The lateral wrist radiograph in Figure A shows significant dorsal angulation of the distal radius with a maintained joint space. The most appropriate surgical treatment for this patient would include corrective osteotomy of the distal radius, as there is no evidence of degenerative changes. If degenerative changes would be present, a salvage procedure such as total wrist arthrodesis would become an option.
The referenced article by Fernandez recommended distal radius corrective osteotomy with bone grafting and internal fixation for the following indications:
1) manually active patients who had a symptomatic extra-articular malunion of the distal end of the radius causing angulation of more than 25 to 30 degrees in either the frontal or sagittal plane without significant degenerative changes in the wrist joint (such as narrowing of the joint space, intra-articular incongruency, subchondral sclerosis, and osteophytic reaction) and in whom it
was thought that the result of either a Darrach procedure or shortening osteotomy of the ulna would be uncertain because the deformity of the radius would not be corrected, and 2) patients who wished to have the deformity corrected even though they had adequate function of the wrist.
- A 25-year-old male presents to the emergency department with a mangled lower extremity that is not salvageable. He undergoes transfemoral amputation. Three months later the patient presents to the office with the limb sitting in an abducted position. What important step was forgotten during the amputation?
- Beveling the distal femur
- Saving the patella
- Allowing the sciatic nerve to retract deep into the soft tissue
- Myodesis of the adductors
- Timely fitting of orthosis CORRECT ANSWER: 4
Prior to the late 80’s, techniques for transfemoral amputation sacrificed the hip adductor muscles resulting in unopposed abductor forces. Amputation with an abducted femur leads to an increase in side lurch and higher energy consumption. Gottschalk in ’99 showed that myodesis of the adductor magnus through drill holes in the lateral femur preserved maximum muscle force and provided a mechanical advantage for the adductors of the thigh. This resulted in maintenance of the normal anatomic alignment of the femur and a balance between the abductor and adductor mechanisms of the hip, thus providing patients with improved control and easier prosthesis fit.
- What is the most common complication of the fracture seen in figure A, if operatively treated as seen in figure B?
- Decreased elbow range of motion
- Wound healing complications
- Iatrogenic ulnar nerve injury
- Inadvertent intra-articular hardware penetration
- Nonunion of the distal humerus fracture CORRECT ANSWER: 1
Decreased elbow range of motion is the most common complication after open reduction internal fixation of distal humerus fractures. Per Galano et al, this loss of motion can arise from "a variety of causes, including articular incongruity or adhesions, capsular contractures, loose bodies, heterotopic ossification, and prominent hardware."
Other complications include nonunion of the distal humerus (2-10%) and ulnar neuropathy (0-12%).
- A 32-year-old female sustained a bimalleolar ankle fracture and was treated with open reduction and internal fixation four months ago. A radiograph of her ankle is shown in Figure A. Recommended management should consist of?
- Physical therapy for ambulation assistance and proprioception training
- Short leg bracing
- Revision open reduction and internal fixation with open syndesmosis reduction
- Addition of syndesmosis screw from fibula to tibia
- Open medial ankle ligament reconstruction CORRECT ANSWER: 3
This patient requires revision open reduction and internal fixation of her syndesmosis as post-operative radiographs demonstrate a severely malaligned ankle with obvious syndesmosis widening and fibular shortening.
Malalignment following ankle fracture fixation can alter the anatomical axis of the joint, articular congruency, and normal load distribution. This predisposes the patient to the development of chronic pain, functional impairment, and finally early post-traumatic ankle arthritis.
Marti et al. retrospectively reviewed the outcomes of 31 patients with malunited ankle fractures who underwent reconstructive osteotomies. The authors found that reconstruction resulted in good or excellent results in the majority of patients. They also note that minor post-traumatic arthritis was not a contraindication to reconstruction.
Ramsey et al. evaluated 23 cadaveric ankles using a carbon black transference technique to determine the contact area in the dissected tibiotalar articulations, with the talus in neutral position and displaced one, two, four, and six millimeters laterally. They found that 1 mm of lateral talar displacement resulted in a 42% decrease in tibiotalar contact area.
Figure A is a x-ray demonstrating severe malalignment of a bimalleolar ankle fracture following fixation of the fibula and medial malleolus. There is obvious shortening of the fibula and lateral shift and valgus tilt of the talus associated with a disrupted syndesmosis.
Incorrect Answers:
Answer 1 & 2: Physical therapy and short leg bracing are not indicated at this point as the anatomical malalignment needs to first be addressed via revision surgery.
Answer 4: The addition of a syndesmosis screw will not successfully reduce the syndesmosis as it has been chronically malreduced and will require open reduction and debridement prior to syndesmosis screw fixation.
Answer 5: Open medial ankle ligament reconstruction is insufficient in isolation to provide mechanical stability to the ankle fractures with syndesmosis disruption.
- A 32-year-old taxi driver sustains a displaced supination external rotation ankle injury after slipping off of a curb. He subsequently undergoes surgical fixation, and a post-operative radiograph is shown in Figure A. At the eight-week postoperative visit, you are asked to fill out a return to work form. How long from today’s visit will his braking time be expected to return to normal?
- Two weeks ago
- One week from now
- Three weeks from now
- Six weeks from now
- Eight weeks from now CORRECT ANSWER: 2
Patients recover the ability to safely operate the brakes of an automobile 9 weeks following operative repair of an ankle fracture. Because this patient is currently 8 weeks out from surgery, his braking time will be expected to return to normal one week from now.
Egol et al studied the time braking ability returns to normal in patients with operatively treated ankle fractures. Patients were studied at 6, 9, and 12 weeks postoperatively and compared to healthy controls. It was determined that total braking time returned to normal by 9 weeks.
- A 45-year-old IV drug abuser has sternoclavicular (SC) joint pain for the past 2 weeks. He is afebrile and physical exam findings include point tenderness and swelling. He most likely has septic arthritis of the
sternoclavicular joint. If so, what is the most likely infecting organism?
- Streptococcus pneumoniae
- Staphylococcus aureus
- Pseudomonas aeruginosa
- Staphylococcus epidermis
- Propionibacterium acnes CORRECT ANSWER: 2
Risk factors for sternoclavicular septic arthritis include IV drug abuse, diabetes, and HIV.
According to the reference by Ross et al, Staphylococcus aureus accounts for 49% of infections. SC joint arthritis accounts for 1% of septic arthritis in the general population but 17% in the IV drug abuse population. Symptoms include spontaneous swelling with the appearance of joint subluxation and localized pain. These patients are not uncommonly afebrile.
- A 42-year-old male sustains a left leg injury as the result of a high- speed motor vehicle collision. Physical exam reveals a grossly deformed left leg with a 1 centimeter open wound over the anterolateral aspect of his tibia; no gross neurovascular deficits are noted. Injury radiographs are shown in Figures A and B. He undergoes immediate tibial nailing with debridement and primary closure of his traumatic wound. Which of the following is the Gustilo-Anderson classification for his fracture?
- I
- II
- IIIA
- IIIB
- IIIC
CORRECT ANSWER: 3
Figures A and B show a significantly comminuted, segmental tibial fracture. In this scenario, the fracture is appropriately classified as a Grade III because there is a highly comminuted, segmental fracture which is always associated with significant periosteal stripping. The patient's leg was able to be closed primarily, which means that it should be classified as a IIIA.
Gustilo et al reviews 18 years of open fracture treatment and outcomes. They reported that débridement and copious irrigation, with primary closure for type I and II fractures and secondary closure for type III fractures resulted in a 5% infection rate (9% for type III fractures). Initial wound cultures were positive in 70.3% despite an infection rate of that patient group of only 2.5%.
Incorrect Answers:
Answer 1: Gustilo Type I wounds have a clean skin lesion that are < 1 cm, and a simple fracture with minimal comminution.
Answer 2: Gustilo Type II wounds have a skin lesion > 1 cm, no extensive soft tissue damage, minimal crushing, and moderate comminution and contamination.
Answer 4: Gustilo Type IIIB are defined as wounds that require a flap for salvage.
Answer 5: Gustilo Type IIIC have an exposed fracture with arterial damage that requires repair.
- Which of the following statements is true regarding the superomedial fragment of an intra-articular calcaneus fracture?
- Fragment typically does not move due to its attachment to the Achilles tendon
- Fragment has the flexor hallucis longus wrap inferiorly around it
- Fragment typically does not move due to its attachment to the navicular
- Fragment typically displaces superior and laterally
- Fragment has the tibialis posterior wrap inferiorly around it CORRECT ANSWER: 2
The superomedial fragment of an intra-articular calcaneus fracture is also referred to as the sustentacular fragment (Illustrations A and B). Additionally, the flexor hallucis longus (FHL) tendon wraps inferiorly around this piece within an osseus groove (illustration C).
In their CT analysis of acute calcaneal fractures, Gilmer et al describe that the superomedial piece typically does not move in these fractures due to its strong
ligamentous attachments to the talus. For this reason, it is commonly referred to as the "constant fragment." The superomedial fragment is important during ORIF of calcaneal fractures, because fixation from the lateral side is typically based off this "constant" fragment as a guide to provide anatomic fixation.
The reference by Berberian et al also reviews the CT findings of 100 consecutive calcaneus fractures, and reports that the sustentacular fragment was displaced in forty-two of the 100 fractures. This brings into question the nature of the "constant fragment" possibly being less of a constant finding as we previously thought.
Illustration D shows the classification and position of these fractures. Incorrect Answers:
Answer 1: This fragment does not typically move due to the Achilles tendon
pull.
Answer 3: The fragment does not typically stay in place due to navicular attachments.
Answer 4: This fragment does not typically displace in these directions. Answer 5: The posterior tibial tendon does not wrap directly inferiorly to this piece of bone.
- A 56-year-old diabetic male presents to the emergency department by ambulance after developing high-grade fevers, malaise, and altered mental status. Upon presentation, he is found to be hypotensive and initial labs show an elevated WBC with a profound left shift. Figure A shows skin manifestations confined to the foot at initial presentation. He is started on broad spectrum antibiotics. Upon follow- up exam 3 hours later his clinical condition deteriorates (Figure B) and he is taken to the operating room for surgical debridement. In a bacterial culture, what would be the most common single isolate for this condition?
- Staphylococcus aureus
- Staphylococcus epidermidis
- Group A streptococcus
- Enterobacteriaceae
- Pseudomonas CORRECT ANSWER: 3
The above clinical vignette is describing necrotizing fasciitis. Necrotizing fasciitis is a rare and often fatal soft-tissue infection that requires high clinical suspicion and prompt administration of broad-spectrum antibiotics and aggressive surgical debridement (illustrations A). Fontes et al found that although polymicrobial infections including gram-positive, gram-negative, aerobic, and anaerobic bacteria were found most commonly in necrotizing fasciitis, Group A streptococcus was the most common bacterial isolate. Wong et al also found the most isolated organism to be group A streptococcus. In their study, the highest associated medical comorbidity was diabetes mellitus (71%). They found that delay in surgery of more than 24 hours was correlated with increased risk of death.
- A 74-year-old female trips over the curb in a parking lot and sustains the shoulder injury shown in Figures A and B. An open reduction and humeral hemiarthroplasty is performed. A postoperative radiograph is provided in Figure C. This patient is most at risk for which of the following complications?
- Shoulder dislocation
- Pulmonary embolus
- Loss of sensation over the lateral shoulder
- Reduced shoulder elevation and abduction
- Ulnar nerve palsy CORRECT ANSWER: 4
The radiographs demonstrate a 3-part proximal humerus fracture with an intra-articular split. The postoperative radiograph shows a humeral hemiarthroplasty with the humeral head resting just below the top of the greater tuberosity. This puts the shoulder at risk of impingement and loss of abduction and elevation. The remaining answer choices are all possible, but much less likely complications.
Zuckerman et al review 26 hemiarthroplasties performed for 3 and 4 part proximal humerus fractures. The procedure reliably produced pain free shoulders, but the function was much less predictable with up to 25% demonstrating some loss of daily function.
In an Instructional Course Lecture, Bigliani et al review techniques for humeral arthroplasty and soft tissue preservation. Of note, the humeral head should rest above the level of the greater tuberosity to prevent impingement. Closure
of the rotator interval is not necessary during this procedure and can over- tighten the anterior soft tissue restraints causing loss of external rotation.
- A 37-year-old man presents to the emergency room with the left lower extremity injury shown in Figure A. A radiograph is shown in Figure B. Which of the following has the most impact on the decision to attempt limb salvage versus amputation?
- Quality of initial fracture reduction
- History of tobacco use
- Insurance status
- Extent of soft tissue injury
- Operative debridement and irrigation within 1 hour of injury CORRECT ANSWER: 4
Extent of soft tissue injury has been shown in Level 2 evidence as having the highest impact on the decision to undergo limb salvage or amputation.
The referenced study by MacKenzie et al looked at 527 of the 601 patients initially enrolled in the Lower Extremity Assessment Project (LEAP) and looked at several variables which are thought to be predictors of amputation. Severe muscle injury had the highest impact on the decision to amputate the limb, likely related to the surgeon’s assessment that the salvaged limb would function poorly because of the risk of infection, nonunion, and poor function.
The absence of plantar sensation had the next most significant impact on surgical decision making. Factors that would influence proceeding with an amputation include an nonviable limb, irreparable vascular injury, warm ischemia time of more than 8 hours, or a severe crush injury with minimal remaining viable tissue. Amputation should also be considered when attempts at limb salvage leave the limb so severely damaged that function will be less satisfactory than that afforded by a prosthetic replacement, are a threat to the patient’s life, or would demand multiple surgical procedures and prolonged reconstruction time that is incompatible with the personal, sociologic, and economic consequences the patient is willing to undergo.
- A 34-year-old woman twists her right ankle stepping off the city bus. An AP ankle radiograph is provided in Figure A. Which of the following statements accurately describe this radiograph?
- The tibiofibular overlap is less than 3 mm
- The fibula demonstrates a Weber C fracture pattern
- The tibiofibular clear space is less than 4 mm
- The fracture is consistent with a Lauge-Hansen pronation-external rotation injury pattern
- The medial clear space is greater than 5 mm CORRECT ANSWER: 3
The AP radiograph demonstrates a stable, minimally displaced Weber B ankle fracture. It is consistent with a Lauge-Hansen supination-external rotation injury pattern.
Harper et al performed cadaveric measurements to define normal radiographic values for standard ankle imaging. The tibiofibular overlap is defined as the horizontal distance from the lateral border of the posterior tibial malleolus (the incisura fibularis) and the medial border of the fibula at the point where the posterior malleolus is widest on an AP radiograph should be great than 6 mm. Tibiofibular clear space is defined as the horizontal distance between the medial border of the fibula and the lateral border of the anterior tibial prominence on an AP radiograph, and should be <6mm. The medial clear space, defined as the distance between the lateral aspect of the medial malleolus and the medial border of the talus at the level of the talar dome on the mortise radiograph should be less than 4 mm.
Ostrum et al performed a radiographic study on human volunteers and noted gender differences. In this study, normal tibial clear space should be less than
5.2 mm in women and less than 6.5 mm in men. The tibiofibular overlap should be greater than 2.7 mm in women and greater than 5.7 mm in men.
Illustration A is an example of the proper measurement of the tib-fib clear space.
- A cadaveric study in 1990 established much of the orthopaedic literature on humeral head vascularity for two decades until recent experiments have provided new data. This original study in 1990 concluded that the anterolateral branch of the anterior circumflex artery supplies blood to what aspect of the proximal humerus?
- Anterior portion of humeral head
- Lesser tuberosity
- Entire humeral head except posteroinferior portion of lesser tuberosity and head
- Entire humeral head except posteroinferior portion of greater tuberosity and head
- Entire humeral head except entire greater tuberosity CORRECT ANSWER: 4
The anterolateral branch of the anterior circumflex artery, called the arcuate artery terminally, provides blood supply to the entire humeral head, lesser tuberosity and greater tuberosity except for a small posterior area. The posterior portion of the greater tuberosity and a small posteroinferior portion of the humeral head are supplied by the posterior circumflex artery.
Gerber et al performed an anatomical study of the arteries of the humeral head to determine their intraosseous distributions. They injected a radiopaque suspension into the anterior circumflex, posterior circumflex, suprascapular, thoracoacromial, or subscapular artery and then analyzed the specimens macroscopically and radiographically. The humeral head was shown to have been perfused by the anterolateral ascending branch of the anterior circumflex artery in all specimens. The posterior circumflex artery vascularized only the posterior portion of the greater tuberosity and a small posteroinferior part of the head.
While previous literature suggested that the anterior humeral circumflex artery provided the main blood supply to the humeral head, more current literature supports the posterior circumflex humeral artery as the predominant blood supply. Despite the anterior humeral circumflex artery being disrupted in approximately 80% of proximal humeral fractures, the occurrence of resultant osteonecrosis is still infrequent. This inconsistency also suggests a greater role for the posterior humeral circumflex artery.
Hettrich et al. performed a cadaveric study assessing the vascularity of the proximal part of the humerus. They injected gadolinium into the axillary artery proximally, and then either the anterior humeral circumflex artery or the posterior humeral circumflex artery was ligated. MRI was then performed and the specimens were dissected to determine the dominant blood supply. They found that the posterior humeral circumflex artery provided 64% of the blood supply to the humeral head, whereas the anterior humeral circumflex artery supplied 36%. The posterior humeral circumflex artery also provided significantly more of the blood supply in three of the four quadrants of the humeral head.
Illustration A depicts the humeral head vascular supply with #2 being the posterior circumflex, #3 being the anterior circumflex arteries, and #4 being the anterolateral humeral circumflex artery.
- A 21-year-old collegiate football player has been diagnosed with a left superior trunk brachial plexus injury following a tackle. Which of the following would most likely be normal on physical exam?
- Sensation over the lateral aspect of shoulder
- Biceps reflex
- Shoulder abduction
- Sensation over radial aspect of forearm
- 2nd and 5th finger abduction CORRECT ANSWER: 5
Examination of finger abduction would be normal in a patient with an isolated superior trunk brachial plexus injury. Finger abduction is performed by the ulnar nerve, which is supplied by the inferior trunk of the brachial plexus.
Superior trunk brachial plexus injuries are thought to occur secondary to traction when an athlete sustains a lateral flexion injury of the neck. Transient injuries are often referred to as "stingers" or a "burner." Symptoms of these injuries are referable to the motor and sensory functions of the axillary, musculocutaneous, and supra-scapular nerves.
Hershman reviewed the etiology of brachial plexus injuries. They showed that superior trunk brachial plexus injuries are usually transient, with 95% of people regaining full neurological recovery with conservative management.
Schenck showed that injuries to the superior trunk will mainly affect muscles supplied by C5 and C6, such as the deltoid, biceps brachii, brachialis, and brachioradialis muscles. Decreased sensation will occur over the lateral
shoulder, lateral aspect of the upper limb, as well as the radial half of the volar forearm with these injuries.
Illustration A demonstrates a lateral traction injury that can precipitate this type of superior brachial plexus injury. Illustration B shows a schematic of the brachial plexus. The nerves supplied by the superior trunk of the brachial plexus include the axillary, musculocutaneous, and supra-scapular nerves.
Incorrect Answers:
Answer A: Decreased sensation over the lateral aspect of shoulder = axillary nerve.
Answer B: Decreased biceps reflex = C5 +/- C6 reflex arc. Answer C: Weakness to shoulder abduction = axillary nerve.
Answer D: Decreased sensation over the radial aspect of the forearm = lateral antebrachial cutaneous nerve of the forearm (branch of musculocutaneous nerve).
- The greatest amount of iatrogenic injury to the piriformis tendon is associated with which of the following?
- Antegrade piriformis entry femoral nailing
- Antegrade greater trochanteric entry femoral nailing
- Retrograde femoral nailing
- External fixation of a femoral shaft fracture
- Open reduction and internal fixation of an intertrochanteric fracture CORRECT ANSWER: 1
There is an increased rate of injury to the piriformis tendon, medial femoral circumflex artery branches, gluteus minimus, and superior gluteal nerve branches are noted with the piriformis starting site. Increased injury to the gluteus medius is seen with a greater trochanteric starting point.
The referenced study by Dora et al noted increased injury to the piriformis tendon with a piriformis starting point (as compared to a more lateral insertion site).
The classic referenced article by Johnson et al notes that anterior placement of the starting point >6mm over the recommended start leads to increased hoop stresses and possible burst-type fractures.
The classic referenced study by Winquist et al reviewed their series of 520 femur fractures treated by antegrade nailing; they report a 99.1% union rate.
- Figure A shows a below the knee amputation performed in a diabetic patient with significant vascular disease. Removal of the "dog ears", indicated by the red arrows, could cause direct damage to what vasculature leading to flap necrosis?
- Anterior tibial artery
- Saphenous and sural arteries
- Posterior tibial artery
- Peroneal artery
- Lower popliteal artery CORRECT ANSWER: 2
"Dog ears" at the edge of a long posterior flap BKA incision are typically left intact because removal risks posterior flap blood supply.
Gray et al conducted an anatomic study to examine the BKA vascular anatomy and specifically the blood supply contribution of the soleus muscle. In their article, they describe the saphenous and sural arteries as being the main blood supply to the proximal posterior aspect of the calf. These arteries lie on the medial and lateral border of a long posterior flap, and can be at risk for transection when excising "dog ears". These arteries are particularly important in patients with severe vascular disease, as the popliteal artery and its immediate branches may be occluded, while the collateral smaller vessels (ie. saphenous and sural arteries) remain patent. Of note, the authors did conclude that the soleus muscle does not contribute blood supply to a long posterior flap, and it should be entirely excised.
Faltie-Jensen et al compared the rate of complications with long posterior flaps vs. equal sagittal flaps in diabetic and non-diabetic patients. They found that in diabetic patients, there was a higher incidence of infection and failure of wound healing in patients with the long posterior flap.
Manoli argues against the use of equal coronal flap, "fish mouth" incisions for below the knee amputations in patients with diabetes and peripheral vascular disease because of the increased risk of anterior flap necrosis.
- A 8-year-old girl sustained a Gustilo-Anderson grade III open tibia fracture 1 week ago and underwent two debridements with definitive fracture fixation. She now has a soft-tissue defect that measures 7 cm
× 7 cm on the distal third leg that is a 3 centimeters proximal to the ankle. There is exposed bone on the medial aspect of her leg. A Negative pressure wound therapy (NPWT) device was applied to her leg. All of the following are benefits of the NPWT EXCEPT:
- Decrease likelihood of complex secondary soft tissue reconstruction
- Permits outpatient management of complex wound
- Reduce edema to wound
- Stimulation of granulation tissue
- Decreases wound angiogenesis CORRECT ANSWER: 5
Due to the limited soft tissue coverage of the medial aspect of the distal third of the tibia, full-thickness wound in this region often requires free-flap coverage. However, NWPT is typically applied first, as this has been shown to contribute to all of the above benefits with the exception of decreased wound angiogenesis.
NPWT has a number or purported beneits, including stimulation of angiogenesis, reduction of local edema, increased blood flow at the wound bed, and increased granulation tissue in the wound. These affects accelerate wound healing and may reduce the need for complex wound coverage. NPT has become increasingly popular as a temporizing measure for complex wound management and can enable outpatient treatment.
Caniano et al. reviewed 51 pediatric patients who underwent NPWT using the Vacuum Assisted Closure (VAC) device to aid in soft tissue closure. Nine of these patients had extremity wounds, for which a VAC was applied as a bridge to either a skin graft or a free flap. The VAC was applied intraoperatively, and patients then followed up as an outpatient with dressing changes performed three times weekly. The authors found that NPWT was safe, cost-effective to complex wound care, and permitted outpatien management.
Mooney et al. reviewed 27 patients with complex extremity wounds managed with NPWT. They found that all wounds, whether acute or with prior failed soft tissue procedure, eventually healed with NPWT and without additional complex coverage procedures. The authors noted that patients developed robust granulation tissue, even over exposed bone, tendon, joint, and hardware, which could then be covered with a split thickness graft or allowed to heal by
secondary intention. They concluded that NPWT may decrease need for complex microvascular tissue transfer.
Incorrect Answers:
Answer 1: NPWT decreases the likelihood of complex secondary soft tissue reconstruction.
Answer 2: NPWT often permits outpatient management of complex wounds. Answer 3: NPWT reduces edema to the wound bed.
Answer 4. NPWT stimulates of granulation tissue and prepares the wound for STSG or free flap.
- In patients with ipsilateral femoral neck and shaft fractures, what percent of femoral neck fractures are diagnosed on a delayed basis if fine cut CT is not utilized?
1. 1%
2. 10%
3. 30%
4. 60%
5. 75%
CORRECT ANSWER: 3
Ipsilateral femoral neck and shaft fractures occur in high energy injuries, with a reported incidence of 2.5-9%. The diagnosis of neck fracture is delayed in 19%-31% of patients. The neck fracture line is almost vertical and nondisplaced, or minimally displaced in 26% to 59% of cases. Two major complications, AVN of the femoral head and non-union of the neck result from neck fracture; therefore, it takes precedence. Dedicated protocols of femoral neck fracture detection have been described and include: dedicated preoperative hip radiographs, pelvic/hip CT scan, and intraoperative fluoroscopic examination.
The referenced article by Bennet et al is a case series of 42 patients with ipsilateral femoral neck-shaft fractures; 31% of the neck fractures were initially missed but none developed femoral head AVN.
The referenced article by Wolinsky et al is an excellent review of this injury. They also report that evaluation of these fractures with plain radiographs can be nondiagnostic 19% to 50% of the time when the majority of the fractures are minimally or nondisplaced.
Illustration A depicts a radiograph of an ipsilateral femoral shaft and vertical, minimally displaced femoral neck fracture.
- A 22-year-old male sustains the injury shown in Figure A. When placing an antegrade intramedullary nail with manual traction in a supine position, which of the following is true when compared to placement of a nail using a fracture table?
- Increased operative time
- Decreased internal malrotation deformities
- Increased external malrotation deformities
- Increased pudendal nerve injury
- Increased need for revision CORRECT ANSWER: 2
Figure A shows a proximal (supraisthmal) femoral shaft fracture.
The referenced article by Stephen et al is a randomized controlled trial between manual traction and fracture-table traction for the reduction and nailing of femoral shaft fractures in terms of quality of the reduction, operative time, complications, and functional status of the patient in eighty-seven patients. Internal malrotation was significantly more common when the fracture table had been used: twelve (29%) of the forty-two femora were internally rotated by >10° compared with three (7%) of the forty-five treated with manual traction. Mean operative time was also less in the manual traction group.
The referenced study by Wolinsky et al also found that use of a traction table significantly increased the anesthesia time, total operating room time, prep and drape time, and overall surgical time as compared to manual traction.
- A 16-year-old male fell from a roof onto his right shoulder and presents with decreased pulses in his right upper extremity. Imaging reveals a posterior sternoclavicular dislocation. What is the best treatment at this time?
- Nonoperative treatment with a sling and swathe for six weeks
- Nonoperative treatment with immediate active range of motion of the shoulder
- Closed reduction in the emergency room
- Open reduction and pinning of his medial clavicular physeal injury
- Reduction in the operating room with thoracic surgery back-up CORRECT ANSWER: 5
Symptomatic acute posterior sternoclavicular dislocations in adolescents should undergo reduction with thoracic surgery back-up. If closed reduction is not successful (inability to reduce in up to 75% of cases), then open reduction is indicated. In patients younger than age 20-25, this is usually a physeal injury,
as the medial clavicular physis does not close until this age range. Chronic anterior dislocations are recommended to be treated conservatively, especially if not symptomatic.
The review article by Wirth and Rockwood notes the following complications with posterior dislocation: respiratory distress, venous congestion or arterial insufficiency, brachial plexus compression, and myocardial conduction abnormalities. They recommend reconstruction of the costoclavicular ligaments with resection of the medial clavicular head as needed for unstable injuries.
The referenced article by Waters et al noted 100% excellent short-term outcomes in adolescents with open reduction and reconstruction of the costoclavicular ligament in pure dislocations or with suture fixation of the medial physis in physeal injuries.
- A 25-year-old patient presents with a posterior wall/ posterior column acetabular fracture. She is scheduled for open reduction internal fixation through a posterior approach. What position of the leg exerts the least amount of intraneural pressure on the sciatic nerve?
- hip flexion, knee extension
- hip extension, knee extension
- hip flexion, knee flexion
- hip extension, knee flexion
- the pressure does not vary based on position CORRECT ANSWER: 4
In the cited study, researchers measured tissue fluid pressure within the sciatic nerve in cadaveric specimens using a pressure transducer. The hip and knee were taken through a combination of ranges and found that the clinically relevant increase in pressure happened with the hip flexed at 90 degrees and the knee fully extended. They concluded that increased intraneural pressure was related to excursion of the nerve as linear distance between the greater sciatic notch and the distal leg increase. Hence, according to the question stem, to avoid traction injury, the reverse position should be implemented (hip extension and knee flexion).
- Which of the following is most important to achieve a good outcome following a Syme amputation?
- trimming any dog ears
- a viable and stable heel pad
- achilles tendon lengthening
- preserving the malleoli
- tenodesing the extensor digitorum longus to the tibial shaft CORRECT ANSWER: 2
A Syme amputation is effectively a tibiotalar disarticulation, which provides an end-bearing stump that could potentially allow ambulation without a prosthesis over short distances. It works better for tumor and trauma, but the heel pad must be viable. The two most common problems are 1) skin sloughing from compromised vascular supply and 2) migration of the heel pad due to instability. A hypermobile heel pad can cause difficulty with prosthesis wear and damage to the soft tissues which can eventually lead to failure. Both malleoli are usually removed in the procedure, except in children or during the first stage procedure of a diabetic or infection case. The tibialis anterior is usually tenodesed to the anterior heel pad along with the EDL tendon to avoid posterior migration of the heel pad.
- A 21-year-old male presents to the emergency department after sustaining a gun shot wound to his abdomen. Subsequent radiographs reveal a bullet in the L2 vertebral body. Physical exam shows no neurologic deficits. He undergoes emergent laparotomy and is found to have a small bowel laceration. What would be the preferred treatment following his exploratory laparotomy and small bowel repair?
- Intravenous antibiotic coverage for Gram negative bacteria for 7 days
- Surgical decompression and bullet fragment removal
- Observation
- Broad-spectrum oral antibiotic coverage for 7 days
- Broad-spectrum intravenous antibiotic coverage for 7 days CORRECT ANSWER: 5
The clinical presentation is consistent with a GSW with bowel perforation and a retained bullet in the vertebral body. Because the patient is neurologically intact broad-spectrum intravenous antibiotic coverage for 7 days is the most appropriate treatment.
Gunshot wounds to the spine present relatively little risk of infection in most cases. When there has been an injury to the small bowel, the risk of infection can be minimized with a 7-day course of broad-spectrum antibiotics.
Indications for surgery include when a neurological deficit is present that correlates with imaging findings of neurological compression by the missile, or when the missile is in contact with the CSF posing a risk for metal toxicity.
Roffi et al performed a Level 4 study of 42 patients that sustained a gunshot wound that perforated the stomach or bowel and then entered the spinal column. They found that an extended regimen of broad spectrum antibiotics combined with bedrest appeared to significantly reduce the risk of spinal or paraspinal infection, whereas early bullet removal did not significantly prevent the occurrence of infection.
Velmahoos et al performed a Level 4 study including 24 patients that sustained a gunshot wound to the spine with associated colonic injury. They found that the incidence of sepsis was 8.4% (compared to 5% in non-bowel injuries) and concluded that retainment of the bullet did not increase the rate of sepsis.
Incorrect Answers:
Answer 1: The patient needs coverage for both Gram+ and Gram- organisms. Answer 2: There are no indications for surgery at this point.
Answer 3: Antibiotics are indicated to decrease the risk of infection.
Answer 4: Intravenous antibiotic has been found to be more effective than oral antiobiotics.
- A trauma patient presents with a major head injury and femoral shaft fracture. He undergoes early fixation of the femur fracture with a prolonged period of intraoperative hypotension. What is the most likely outcome to be expected post-operatively in this patient?
- Increased risk of post-operative bleeding
- Increased risk of pneumonia
- Decreased IV fluid administration
- Lower Glasgow Coma Scale scores at the time of discharge from hospital
- Improved central nervous system outcomes at the time of discharge from hospital
CORRECT ANSWER: 4
Prolonged periods of hypoxia and hypotension are associated with lower GCS scores in polytrauma patients with major head injuries.
The timing of fracture fixation in polytrauma patients with major head injuries has been controversial. Most studies support early fixation despite some literature reporting adverse affects on CNS outcomes. This is largely related to hypoxemia and hypotension intraoperatively, as well as greater fluid administration, which should be avoided. Hypoxia and hypotension are associated with lower GCS scores at the time of discharge.
In a study by Jaicks et al found a lower discharge GCS in the early fracture fixation group compared with the late group. However, they also found that this association was due to hypoxemia and hypotension.
The referenced study by Scalea et al reviewed 171 patients with pelvic or lower extremity fractures and head injuries; they showed no difference in CNS outcomes or mortality in patients who underwent early fixation.
Brundage et al showed improved outcomes (including high GCS scores at time of discharge) in those who had early fixation of femoral shaft fractures in the head-injured patient.
Incorrect Answers:
Answer 1: There is no direct correlation between intra-operative bleeding and post-operative bleeding. They is literature to support that hypotension my decrease bleeding at the time of surgery.
Answer 2: There is no direct correlation between early fixation rates and post- operative pneumonia.
Answer 3: Intraoperative hypotension is correlated with increased IV fluid administration.
Answer 4: Intraoperative hypotension is associated with decreased central nervous system outcomes.
- A 33-year-old female is diagnosed with spontaneous atraumatic subluxation of the sternoclavicular joint. She notes mild, intermittent pain and a small amount of prominence to that area. She is noted to have 6 points out of a possible 9 points on the Beighton-Horan scale. What is the most appropriate treatment at this time?
- Observation
- Figure of eight brace for 6 weeks followed by progressive physical therapy
- Resection arthroplasty of the sternoclavicular joint
- Sternoclavicular and costoclavicular ligament reconstruction
- Sternoclavicular arthrodesis CORRECT ANSWER: 1
Spontaneous atraumatic subluxaton of the sternoclavicular joint is a rare condition and is generally associated with ligamentous laxity. A score of 5 or more on 9-point Beighton-Horan scale defines joint hypermobility. The treatment for spontaneous atraumatic subluxaton of the sternoclavicular joint is observation.
Higginbotham et al reported that spontaneous atraumatic anterior subluxation of the sternoclavicular joint may occur during overhead elevation of the arm. The majority of cases are not painful, and the subluxation usually reduces with lowering the arm. Surgery is rarely indicated. Nonsurgical management, including patient education of the benign nature of the condition, is recommended.
Rockwood et al reviewed a series of 37 patients with this condition and noted that at an average follow-up of eight years, the twenty-nine patients who were treated non-operatively had excellent results, with no limitations of activity or changes in lifestyle. The eight patients who were treated operatively (group II) had numerous problems, including noticeable scars, persistent instability, pain, or limitation of activity that resulted in an alteration in lifestyle.
Illustration A is a chart that outlines the Beigton-Horan scale and Illustration B demonstrates the clinical images associated with the criteria. A score of 5 or more on 9-point Beighton-Horan scale defines joint hypermobility.
- A 46-year-old woman sustains an extra-articular fracture of the distal radius and undergoes open reduction and internal fixation with a volar plate and screw construct. During postoperative recovery from this injury, what benefit does formal physical therapy have as compared to a patient-guided home exercise program?
- Greater grip strength at 6 months
- Less wrist pain at 1 year
- Better hand dexterity at 1 year
- No difference in functional outcomes
- Quicker return to work CORRECT ANSWER: 4
There are no significant benefits demonstrated with formal physical therapy following distal radius fracture ORIF compared to a patient-guided home exercise program.
The reference by Wakefield and McQueen is a randomized controlled trial of 96 patients, comparing formal hand physiotherapy to a home exercise regimen.
There was no difference in grip strength, pronation/supination, radial/ulnar deviation, or hand function. The authors concluded that there were no significant benefits to formal physiotherapy.
The study by Souer et al is a level I study evaluating formal therapy and patient-guided exercise program for patients who underwent ORIF of a distal radius fracture with a volar plate and screw construct. This study showed a significant decrease in wrist ROM and grip strength with formal therapy. There were no differences in arm-specific disability (DASH score) at any time point.
- A 20-year-old man presents with erythema, swelling, and pain at the left sternoclavicular joint shown in Figure A. His temperature is
38.9 degress Celsius, serum WBC is 14,000, and his C-reactive protein is elevated. He reports that he uses IV heroin. A coronal 3D CT scan of the left clavicle is shown in Figure B. Joint aspiration shows many grams stain positive organisms. Which of the following organisms is the most likely pathogen?
- Propionibacterium acnes
- Staphylococcus aureus
- Group B streptococcus
- Neisseria gonorrhea
- Enterococcus coli CORRECT ANSWER: 2
This patient has sternoclavicular joint septic arthritis with gram positive organisms. Although there is an increased incidence of Pseudomonas aeruginosa infection in IV drug users, S. aureus is still the most common organism.
Ross et al states "Staphylococcus aureus is now the major cause of
sternoclavicular septic arthritis in intravenous drug users. Pseudomonas aeruginosa infection in injection drug users declined dramatically with the end of an epidemic of pentazocine abuse in the 1980s."
The referenced article by Goldin et al is from the New England Journal of Medicine reports that all of their cases of SC joint septic arthritis were in intravenous drug abusers and that P. aeruginosa grew out of 3 patients and S. aureus grew out of 1 patient.
A more recent article by Abu Arab et al reported that Staph aureus was most common even in IV drug users. The review article by Higginbotham and Kuhn note that risk factors for SC joint septic arthritis include hemodialysis, immunocompromise, alcoholism, and HIV. Neisseria gonorrhoeae, fungal, and candida present in HIV patients.
Treatment is I&D and appropriate antibiotics, although aspiration and abx have shown some success too. CT and MRI are useful in diagnosis, and open biopsy or aspiration is recommended for definitive diagnosis.
- A 30-year-old man presents with a distal third tibia fracture that has healed in 25 degrees of varus alignment. The patient is at greatest risk of developing which of the following conditions as a result of this malunion?
- Degenerative lumbar spine changes
- Ipsilateral ankle pain and stiffness
- Ipsilateral hip joint degenerative changes
- Contralateral hip joint degenerative changes
- Ipsilateral medial knee degenerative changes CORRECT ANSWER: 2
A significant malunion of the distal tibia has important consequences for patient outcome, including pain, gait changes, and cosmesis.
The first referenced article by Milner et al looked at long-term outcomes of tibial malunions and noted that varus malunion led to increased ankle/subtalar stiffness and pain regardless of the amount of radiographic degenerative changes.
The second referenced article by Puno et al reinforced the concept of decreased functional outcomes of the ankle with tibial malunions, and noted
that other lower extremity joints (ipsilateral and contralateral) do not have increased rates of degeneration from such a malunion.
- A 33-year-old man sustains a femur fracture in a motorcycle accident. AP and lateral radiographs are provided in Figure A. Prior to surgery, a CT scan of the knee is ordered for preoperative planning. Which of the following additional findings is most likely to be discovered?
- Tibial eminence fracture
- Sagittal plane fracture of the medial femoral condyle
- Schatzker I tibia plateau fracture
- Coronal plane fracture of the lateral femoral condyle
- Axial plane fracture through the medial femoral condyle CORRECT ANSWER: 4
The "Hoffa fracture" is a coronal plane fracture of the femoral condyle that is often missed on plain radiographs of supracondylar and intercondylar femur fractures. It involves the lateral condyle more frequently than the medial.
Identification is important as it may impact operative planning and likely require screw fixation in the anteroposterior plane.
Nork et al. reviewed 202 supracondylar-intercondylar distal femoral fractures and found a 38% prevalence of associated coronal plane fractures. The authors recommend CT scan imaging of all supracondylar and intercondylar fractures.
Ostermann et al reported on 24 unicondylar fractures of the distal femur treated with open reduction internal fixation with a screw construct. Twenty-
three patients acheived satisfactory results at 5 year follow-up. Illustrations A and B are another example of a supracondylar femur fracture with an associated Hoffa fracture identified on CT scan.
- A 35-year-old woman presents with an elbow injury which includes a coronoid fracture involving more than 50%, a comminuted
radial head fracture, and an elbow dislocation. What is the most appropriate treatment?
- closed reduction and early range of motion
- radial head resection and lateral collateral ligament reconstruction
- radial head resection and coronoid open reduction internal fixation
- radial head arthroplasty and coronoid open reduction internal fixation
- radial head arthroplasty, coronoid open reduction internal fixation, and lateral collateral ligament repair
CORRECT ANSWER: 5
A terrible triad of the elbow includes dislocation of the elbow with associated fractures of the radial head and the coronoid process. Ring et al. stressed that these injuries are prone to complications and advised against resection of the radial head due to instability, and instead recommended a radial head replacement if too comminuted for ORIF. Coronoid fractures compromise elbow stability as well and require open reduction and internal fixation as with the lateral collateral ligament. McKee et al. showed stable elbows in 34/36 with mean Mayo elbow score of 88 when the standard protocol of coronoid ORIF, radial head repair/replacement, and LCL repair were employed.
- The talocrural angle of an ankle mortise x-ray is formed between a line perpendicular to the tibial plafond and a line drawn:
- perpendicular to the medial clear space
- parallel to the talar body
- between the tips of the malleoli
- perpendicular to the shaft of the fibular
- parallel to the subtalar joint CORRECT ANSWER: 3
The talocrural angle is formed by the intersection of a line perpendicular to the plafond with a line drawn between the malleoli (average = 83+/-4deg). When the lateral malleolus is shortened secondary to fracture, this can lead to increased talocrural angle. This malunion leads to lateral tilt of the talus.
Phillips et al looked at 138 patients with a closed grade-4 supination-external rotation or pronation-external rotation ankle fracture. Although the conclusions were limited due to poor follow up, they found the difference in the talocrural angle between the injured and normal sides was a statistically significant radiographic indicator of a good prognosis.
Pettrone et al looked at a series of 146 displaced ankle fractures, and the effect of open or closed treatment, and internal fixation of one or both malleoli. They found open reduction proved superior to closed reduction, and in bimalleolar fractures open reduction of both malleoli was better than fixing only the medial side.
Illustrations A and B are demonstrations of the talocrural angle.
- A 33-year-old male sustains the injury shown in Figure A. He is initially treated with a spanning external fixator followed by definitive open reduction internal fixation of the tibia and fibula. His wounds healed without infection or other complications. Two years following surgery, which of the following parameters will most likely predict a poor clinical outcome and inability to return to work?
- Joint line restoration
- Degree of fracture displacement
- Time before definitive ORIF
- Open fracture
- Lower level of education CORRECT ANSWER: 5
Lower level of education is the parameter that correlated most closely with a poor clinical outcome and inability to return to work.
To determine what fracture- and patient-specific variables affect outcome, Williams et al evaluated 29 patients with 32 tibial plafond fractures at a minimum of 2 years from the time of injury. Outcome was assessed by four independent measures: a radiographic arthrosis score, a subjective ankle score, the Short Form-36 (SF-36), and the patient’s ability to return to work. The four outcome measures did not correlate with each other. Radiographic arthrosis was predicted best by severity of injury and accuracy of reduction. However, these variables did not show any significant relationship to the clinical ankle score, the SF-36, or return to work. These outcome measures were more influenced by patient-specific socioeconomic factors. Higher ankle
scores were seen in patients with college degrees and lower scores were seen in patients with a work-related injury. The ability to return to work was affected by the patient’s level of education.
Pollak et al performed a retrospective cohort analysis of pilon fractures. Patient, injury, and treatment characteristics were recorded. The primary outcomes that were measured included general health, walking ability, limitation of range of motion, pain, and stair-climbing ability. A secondary outcome measure was employment status. Multivariate analyses revealed that presence of two or more comorbidities, being married, having an annual personal income of less than $25,000, not having attained a high-school diploma, and having been treated with external fixation with or without limited internal fixation were significantly related to poorer results as reflected by at least two of the five primary outcome measures.
- What is the most appropriate treatment for a 17-year-old boy who sustained a gunshot wound to his forearm from a handgun with a muzzle-velocity of 1000 feet/second if he is neurovascularly intact and radiographs reveal no fracture?
- Irrigation and local wound care in the emergency department
- Emergent irrigation and debridement in the operating room with vacuum- assisted wound closure
- Emergent irrigation and debridement in the operating room with 7 days of intravenous antibiotics
- Wound closure in the emergency department with follow-up wound check in 1 week
- Exploration and removal of all bullet fragments in the emergency department and 10 day course of oral antibiotics
CORRECT ANSWER: 1
The question refers to appropriate management of a gunshot wound to the forearm. The first question that must be answered when evaluating gunshot injuries is whether the gunshot is low velocity or high velocity. Low-velocity wounds are less severe, are more common in the civilian population, and are typically attributed to bullets with muzzle velocities below 1,000 to 2,000 feet per second. Tissue damage is usually more substantial with higher-velocity (greater than 2,000 to 3,000 fps) military and hunting weapons. In this question, a muzzle velocity of 1,000 ft/sec is provided. Low velocity injuries with stable, non-operative fractures can be treated with local wound care.
The two referenced articles offer guidance for treating low-velocity gunshot injuries with stable, non-operative fracture patterns. The first article by Geissler et al is a retrospective study comparing 25 patients that prospectively received local irrigation and debridement, tetanus prophylaxis and a long acting cephalosporin intramuscularly to a random retrospective sample of 25 patients with similar ballistic-induced fractures and wounds managed by local debridement and 48h of intravenous antibiotics. One infection occurred in each group, requiring further therapy. It was concluded that patients with low- velocity gunshot induced fractures can be managed without the use of short- term intravenous antibiotics with no increased risk of infection.
In the second study, Dickey et al evaluated the efficacy of an outpatient management protocol for patients with a gunshot-induced fracture with a stable, non-operative configuration. 41 patients with a grade I or II open, non- operative fracture secondary to a low-velocity bullet were treated with 1gm of cefazolin administered in the emergency room and a 7-day course of oral cephalexin. No patient developed a deep infection. Thus, local I&D, tetanus, and oral antibiotics for 2-3 days is adequate for low velocity gunshot wounds.
- Which clinical sign is the most sensitive for the diagnosis of compartment syndrome in a child with a supracondylar humerus fracture?
- pulselessness
- pallor
- paresthesia
- paralysis
- increasing analgesia requirement CORRECT ANSWER: 5
Although pain, pallor, paresthesia, paralysis, and pulselessness are all possible signs and symptoms of compartment syndrome in children with fractures, studies have shown increasing analgesia requirement is more sensitive.
Bae et al reviewed thirty-six cases of compartment syndrome in 33 pediatric patients. Approximately 75% of these patients developed compartment syndrome in the setting of fracture. "They found pain, pallor, paresthesia, paralysis, and pulselessness were relatively unreliable signs and symptoms of compartment syndrome in these children. An increasing analgesia requirement in combination with other clinical signs, was a more sensitive indicator of
compartment syndrome."
Whitesides et al summarizes the diagnosis and treatment of acute compartment syndrome. They emphasize the need for early diagnosis, as "muscles tolerate 4 hours of ischemia well, but by 6 hours the result is uncertain; after 8 hours, the damage is irreversible." They recommend fasciotomy be performed when tissue pressure rises past 20 mm Hg below diastolic pressure.
- A 45-year-old man sustains the injury seen in Figures A and B following a motor vehicle accident. Postoperative radiographs are seen in Figures C and D. Which of the following is the most accurate when comparing outcomes between intramedullary nailing (IMN) and open reduction internal fixation (ORIF) for this injury?
- Union rates at one year are higher with ORIF
- Infection rates are higher with IMN
- Functional shoulder outcomes at one year are equivalent with IMN and ORIF
- Iatrogenic radial nerve injury rate is higher with ORIF
- Shoulder stiffness rates at one year are equivalent with IMN and ORIF CORRECT ANSWER: 3
Although shoulder pain and stiffness is increased following IMN compared to ORIF, functional outcome scores at one year have been shown to be equivalent
in both treatment groups.
Diaphyseal humeral shaft fractures outcomes following IMN and ORIF are under further investigation. Diaphyseal humeral shaft fractures have historically been treated with ORIF, however proponents for IMN cite benefits of less periosteal stripping and soft tissue dissection. Recent investigations have shown outcomes with regard to nonunion, infection, re-operation, and nerve palsy appear equivalent between both groups. Rates of shoulder stiffness and shoulder pain have been demonstrated to be higher in IMN compared to ORIF. American Shoulder and Elbow Scores (ASES) have shown no difference at one year post-operatively.
Bhandari et al. performed a meta-anaylsis of 3 prospective randomized trials. They found lower rates of re-operation and shoulder impingement with ORIF of humeral shaft fractures.
Wali et al. performed a prospective randomized study of IMN or ORIF on 50 patients with mid-diaphyseal humeral shaft fractures. They found IMN had shorter operative time, shorter hospital stay, and lower blood loss. They found no difference in union rates, complication, or shoulder functional outcomes scores. They conclude IMN to be an effective option for treating mid-diaphyseal humeral shaft fractures.
Heineman et al. have recently conducted an update on their meta-analysis to include more recent randomized studies. With the inclusion of newer studies the author found a statistically significant increase in total complication rate with the use of IM nailing compared with ORIF. The authors found no significant difference between the two treatment modalities for the secondary outcomes (nonunion, infection, nerve palsy, re-operation).
Figures A and B show a diaphyseal humeral shaft fracture. Figure C and D show postoperative radiographs following intramedullary nailing of a humeral shaft fracture.
Incorrect Answers:
Answer 1, 2, and 4: Union, infection, and radial nerve injury rates have been shown to be similar between ORIF and IMN.
Answer 5: Shoulder stiffness rates have been shown to be higher following IMN compared to ORIF.
- A 25-year-old female presents complaining of progressive anteromedial pain in her left ankle. She underwent operative fixation 5 months prior at an outside hospital. The operative report indicated that, due to anterior fracture blisters, a direct medial incision was utilized, centered over the posterior colliculus of the medial malleolus, without violation of the deltoid ligament. A radiograph and computed tomographic scan of her initial injury are shown in Figures A and B, respectively. On exam, she has well-healed incisions, exhibits no tenderness to palpation over her hardware, but does endorse pain with deep palpation along the anteromedial joint line. Figure C shows an anteroposterior left ankle radiograph taken today. Labs are obtained and reveal a white blood cell count of 9.0 k/uL (reference range 4.5-
11.0 k/uL) and a C-reactive protein value of 0.8 mg/dL (<0.9 mg/dL). What is the next best step in managing her problem?
- Syndesmotic fixation
- Intra-articular corticosteroid injection
- Referral to physical therapy
- Surgical correction of malunion
- Removal of hardware CORRECT ANSWER: 4
This patient sustained a supination-adduction (SAD) injury with a vertical shear fracture of her medial malleolus and a fibular avulsion fracture. She sustained a medial plafond articular impaction injury that was not addressed at the time of surgery.
In SAD injuries, supination of the foot is combined with inward rotation at the ankle, adduction of the hindfoot, and inversion of the forefoot. This results in the following sequence of events: 1. Talofibular sprain or distal fibular avulsion (equivalent to Weber A). 2. Vertical medial malleolus fracture as the talus strikes the tibia. Associated injuries may include osteochondral damage to the talus and marginal impaction of the medial plafond. It is important to evaluate the medial plafond for articular impaction. When present, an arthrotomy must be performed, typically utilizing an anteromedial incision, with direct visualization of the articular surface to restore the joint line appropriately.
Weber et al. provided a review article on corrective osteotomies for malleolar fracture malunions. They stress that malunions can lead to ankle instability, abnormal load transfer, and post-traumatic arthritis. They conclude that corrective osteotomies that restore anatomical alignment show good results in long-term follow-up.
Perera et al. provided additional commentary on the surgical reconstruction of malunited ankle fractures. The authors emphasize the link between malunion and poor outcomes. They state that successful salvage procedures involve a clear understanding of the deformity, careful preoperative planning, and a solid understanding of reconstructive techniques. They provide several instructive case examples in their review.
McConnell et al. provided a discussion on SAD ankle fractures at their institution and emphasized the importance of recognizing marginal impaction of the tibial plafond when treating these injuries. Of 800 ankle fractures identified over a 5-year period, 44 were SAD injuries, 19 of the 44 displayed a vertical shear fracture of the medial malleolus, and 8 of the 19 demonstrated marginal impaction of the tibial plafond. These 8 impaction injuries were treated with open reduction internal fixation with elevation of the articular impaction; all had good to excellent outcomes without arthritic changes at last
follow-up.
Figure A is an anteroposterior left ankle radiograph demonstrating a SAD injury with a vertical shear fracture of the medial malleolus, a fibular avulsion fracture, and articular impaction of the medial tibial plafond. Figure B is a coronal CT demonstrating articular impaction of the medial plafond. Illustration A is an intra-operative photograph with the medial malleolus retracted allowing inspection of the articular surface; mild anteromedial plafond impaction is present.
Incorrect Answers:
Answer 1: The syndesmosis should not be disrupted in a SAD injury. Answer 2: An intra-articular corticosteroid injection may provide temporary symptom relief, but does not constitute definitive management.
Answer 3: Physical therapy may provide some symptom relief but will not correct this patient's malunion and progression of post-traumatic arthritis. Answer 5: There is no indication in this vignette that the patient has symptomatic hardware.
- A 90-year-old female slips and falls at home. She is a community ambulator and has no medical problems. She reports right hip pain at this time. Injury radiographs are shown in Figures A & B. Delay of more than 48 hours may result in:
- Increased intraoperative time
- Increased 30-day mortality
- No impact on the rate of postoperative pneumonia
- Higher rates of blood transfusion
- Increased risk of post-operative infection CORRECT ANSWER: 2
Figures A & B demonstrate a right, unstable intertrochanteric femur fracture. Surgical stabilization within 48 hours improves short-term and 30-day mortality.
Hip fractures are common and mortality rates vary. In the elderly, mortality rates may reach 10% at 1-month, 20% at 4-months, and 30% at 1-year. Time to surgery has found to be a decisive factor. A pre-operative delay may lead to an increase in mortality and adversely influence other clinical outcomes.
Clinical guidelines recommend immediate operative stabilization, given the patient is medically fit for surgery.
Nyholm et al. performed a retrospective study of the Danish Fracture Database to investigate whether a surgical delay increases 30-day and 90-day mortality rates for patients with proximal femoral fractures. The 30-day and 90-day mortalities were 10.8% and 17.4%, respectively. The risk of 30-day mortality increased with increasing time intervals of more than 12 hours, 24 hours, and more than 48 hours. 90-day mortality increased with a surgical delay of more than 24 hours. They conclude that rapid surgical treatment should be performed by attending orthopaedic surgeons.
Moja et al. performed a meta-analysis and meta-regression to assess the relationship between surgical delay and mortality in elderly patients with a hip fracture. They analyzed 35 independent studies with 191,873 patients and 34,448 deaths. The majority of studies had a cut-off of 48 hours. They report that early hip surgery was associated with a lower risk of death and pressure sores. They conclude that early hip fracture surgery appears to provide a survival benefit compared to later intervention.
Rodriguez-Fernandez et al. performed a study examining 2 groups with hip fractures. The first group was studied retrospectively and had an average delay of surgical treatment of more than 1-week while the second group was studied prospectively, and had surgical treatment within 48 hours. They found a larger number of complications in the group with a delay in surgical treatment. They conclude that elderly patients with hip fractures should be treated as soon as their medical condition permits.
Figures A and B are the AP and lateral radiographs demonstrating a right, unstable intertrochanteric femur fracture. Illustration A is an intertrochanteric femur fracture, stabilized with a cephalomedullary nail.
Incorrect Answers:
Answer 1: A surgical delay of greater than 48 hours has not been found to increase intraoperative time when patients get to surgery.
Answer 3: A surgical delay of greater than 48 hours has been shown to increase the rate of postoperative pneumonia.
Answer 4: A surgical delay of greater than 48 hours has not been associated with higher rates of blood transfusion.
Answer 5: A surgical delay of greater than 48 hours has not been associated with an increased risk of post-operative infection.
- A 22-year-old healthy left hand dominant male presents to the ED with left shoulder pain after falling from an ATV. Figure A is the radiograph of his left clavicle. He is neurovascularly intact and there is no evidence of skin tenting or open fracture. Which of the following most predisposes this patient to nonunion?
- Diaphyseal fracture
- Fracture displacement
- Age
- Male Gender
- Injury involving the dominant extremity CORRECT ANSWER: 2
Displaced clavicle fractures are associated with higher rates of nonunion.
Nonunion occurs in roughly 5-6% of clavicle fractures and can result in slower functional return, poor cosmesis and muscle fatigability. Clavicle fractures can be sub-classified using the Allman classification into medial, diaphyseal, and lateral injuries (Illustration A). The Neer classification for diaphyseal injuries describes fractures as "nondisplaced" (less than 100% displacement) and "displaced" (greater than 100% displacement).
Robinson et al. performed a prospective cohort study to identify risk factors for nonunion after nonoperative management of clavicle fractures. The overall nonunion rate was 6.2% and was highest in lateral third fractures (11.5%).
Diaphyseal fractures had the lowest nonunion rate (4.5%). Additionally, the authors found that the risk for nonunion was increased by advancing age, female gender, fracture displacement, and comminution.
Jorgensen et al. performed a systemic review of the literature looking for predictors of non-union and malunion in mid shaft clavicle fractures treated non-operatively. They found fracture comminution, displacement, older age, female gender, and the presence of smoking to be his factors for non-union. Of these, displacement was the most likely factor that can be used to predict non- union.
Figure A demonstrates a displaced left clavicle diaphyseal fracture. Note that the medial fragment is displaced superiorly by the deforming force of the sternocleidomastoid. Illustration A represents the Allman classification.
Illustration B demonstrates the deforming forces acting on the clavicle.
Incorrect Answers:
Answer 1: Diaphyseal fractures were demonstrated to have the lowest rate of nonunion when compared to lateral third fractures and medial clavicle fractures.
Answer 3: Advancing age was found to be an independent predictor of nonunion.
Answer 4: Female gender was found to be an independent predictor of nonunion.
Answer 5: Injury to the dominant hand was not found to be associated with an increased risk of non-union.
- A 24-year-old male presents with ankle pain after being involved in a motor vehicle accident. His injury radiograph is shown in Figure A. Which of the following has been shown to contribute to the development of post-traumatic arthritis in this injury pattern?
- Initial superficial zone cartilage cell death via apoptosis at the fracture margins
- Initial superficial zone cartilage cell death via apoptosis remote from the fracture margins
- Initial superficial zone cartilage cell death via necrosis remote from the fracture margins
- Initial superficial zone cartilage cell death via necrosis at the fracture margins
- Delayed superficial zone cartilage cell death via necrosis at the fracture margins
CORRECT ANSWER: 4
Figure A demonstrates a tibial plafond fracture. Initial superficial zone cartilage cell death via necrosis at the fracture margins has been shown to contribute to post-traumatic arthritis.
Post-traumatic osteoarthritis typically occurs after an intra-articular fracture. Impacted chondrocytes die by either necrosis or apoptosis, which have both been implicated in post-traumatic osteoarthritis. Initial cell death in the superficial cartilage zones at the fracture margins occurs by necrosis. Apoptosis occurs in a delayed fashion and is mitigated by several bioactive agents.
Apoptosis also affects the superficial cartilage zones near the fracture margins. Deep cartilaginous zones and areas away from the fracture margins do not seem to be involved in these processes.
McKinley et al. performed a review of the basic science of intra-articular fractures and posttraumatic osteoarthritis. They report that initial damage to the cartilage in combination with the ensuing pathomechanical and pathobiologic response of the cartilage after a fracture contribute to post- traumatic arthritis. Chronic abnormal joint loading is also thought to contribute to this process as well. They conclude that the relative contribution of each is unknown.
Tochigi et al. performed a study to determine the distribution and progression of chondrocyte damage after intra-articular ankle fractures. They harvested 7 normal human ankles and subjected them to impaction. They found that immediate superficial zone chondrocyte death was greater in fracture-edge regions than on-fracture regions. Subsequent cell death over the next 48 hours was significantly higher in fracture-edge regions as well. They conclude that cartilage damage in intra-articular fractures was characterized by chondrocyte death at fracture margins.
Figure A is an ankle mortise radiograph demonstrating an intra-articular tibial plafond fracture.
Incorrect Answers:
Answer 1: Initial superficial zone chondrocyte cell death occurs by necrosis, not apoptosis.
Answer 2: Initial superficial zone chondrocyte cell death occurs by necrosis at the fracture margins.
Answer 3: Delayed superficial zone cartilage cell death occurs via apoptosis at the fracture margins.
Answer 5: Delayed superficial zone cartilage cell death occurs via apoptosis, not necrosis.
- A 35-year-old male presents with left knee pain after sustaining the injury seen in Figure A. He is neurovascularly intact and can perform a straight leg raise, but has pain with passive range of motion. Figures B and C show an anteroposterior and lateral radiograph of the left knee, respectively. 175 cc of saline is injected into the superolateral quadrant with no egress of fluid from the inferolateral
laceration. What percentage of traumatic arthrotomies would be detected with this test?
1. 50%
2. 75%
3. 90%
4. 95%
5. 99%
CORRECT ANSWER: 5
175 cc of fluid injected into the knee joint will predict 99% of inferolateral traumatic arthrotomies of the knee.
Accurately diagnosing traumatic arthrotomies of the knee directs prompt surgical intervention in the form of irrigation and debridement. Surgical intervention is indicated to decrease the risk of septic arthritis due to direct communication with the external environment with the joint. Traumatic arthrotomies may be difficult to diagnose on physical exam and imaging. In those circumstances, additional tests such as saline load testing or CT scan can provide evidence of traumatic arthrotomy and direct appropriate treatment.
Konda et al. retrospectively reviewed 50 consecutive patients who were evaluated for traumatic arthrotomy of the knee using saline load test (SLT). They found the saline load test had a sensitivity of 94% and a specificity of
91%. They conclude that in patients with a small periarticular wound, negative saline load test, and no other imaging studies to suggest traumatic arthrotomy of the knee, the rate of septic arthritis appears to be 0% with nonoperative management.
Nord et al. investigated the volume of saline needed to diagnose a traumatic knee arthrotomy. They found volumes needed to diagnose with 75%, 90%, 95%, and 99% sensitivity were 110, 145, 155, and 175cc, respectively. They concluded that in order to detect a 1-cm inferolateral traumatic arthrotomy with 95% sensitivity 155ml of saline need to be injected into the knee. They also concluded that inferomedial injections required significantly less saline when compared to superomedial injections.
Figure A shows a transverse laceration across the inferolateral portion of the knee joint. Figures B and C show normal AP and lateral left knee radiographs.
Incorrect Answers:
Answers 1, 2, 3 and 4: To detect 5%, 90%, 95%, and 99% of 1-cm inferolateral traumatic knee arthrotomies, 110, 145, 155, and 175cc of saline should be injected into the knee, respectively.
- A 55-year-old man presents with pain in his great toe that began 2 days ago. On physical exam, his first metatarsophalangeal (MTP) joint is red, warm, and tender to palpation (Figure A). His skin is intact with no evidence of ulceration. He reports pain with range of motion. He denies recent fevers and his admission temperature is 37.9°C. Labs are drawn and significant for a WBC of 15K (reference range [rr], 4500-11000 µL), ESR of 90 (rr, 0-20 mm/h), and CRP of 6.5 (rr, 0.08-
3.1 mg/L). A radiograph of his foot is shown in Figure B. Which of the following is the next best step?
- Joint aspiration
- Joint irrigation and debridement
- Outpatient oral antibiotics for 14 days
- Oral indomethacin for 3 to 5 days
- MRI of the foot CORRECT ANSWER: 1
This patient presents with a painful 1st MTP joint and elevated inflammatory markers concerning for septic arthritis vs. an inflammatory process such as gout. A joint aspiration to determine the etiology of his toe pain would be the next best step. Aspiration results for this patient confirmed septic arthritis of the 1st MTP joint.
Septic arthritis of the 1st MTP joint can arise from local inoculation or hematogenous spread. Patients typically present with an acutely swollen, warm, erythematous joint; inability to bear weight; and severe pain with micromotion. Radiographs may be normal or may reveal a subtle joint effusion. Inflammatory markers (ESR and CRP) are usually elevated and can, therefore, be used to rule out a diagnosis of septic arthritis. Joint aspiration, however, remains as the gold standard for diagnosis and is useful for differentiating septic arthritis from gout.
Anakwenze et al. reviewed the diagnosis and management of foot and ankle infections. They report that 3-7% of septic arthritis cases occur in the foot and ankle, with the tibiotalar joint and 1st MTP joint being the most commonly affected and with staph aureus being the most common causative organism.
They state that in cases with sufficient clinical suspicion for septic arthritis, joint aspiration is the diagnostic tool of choice.
Carpenter et al. performed a systematic review to describe the diagnostic characteristics of history, physical examination, and bedside laboratory tests for nongonococcal septic arthritis, and to quantify test and treatment thresholds. They found that, with the exception of joint surgery or skin infection overlying a prosthetic joint, history, physical examination, and serum tests do not significantly alter post-test probability. Serum inflammatory markers such as white blood cell (WBC) counts, erythrocyte sedimentation rate (ESR), and C-reactive protein (CRP) were also found to be not useful acutely.
However, extreme values of synovial WBC (> 50,000) can increase, but not decrease, the probability of septic arthritis.
Figure A is a clinical picture of a red, swollen 1st MTP joint. Figure B is an AP radiograph of a normal foot with no evidence of fracture, arthritis, or erosions.
Incorrect Answers:
Answer 2: Joint irrigation and debridement would be warranted only after a diagnosis of septic arthritis has been confirmed by joint aspiration.
Answer 3: Outpatient oral antibiotics would be appropriate to treat cellulitis, but only after a diagnosis of gout and septic arthritis has been eliminated.
Answer 4: Oral indomethacin is the treatment of choice for gout and would be appropriate after confirmation of the diagnosis via joint aspiration.
Answer 5: MRI is more effective than radiography for diagnosing sepsis with or with osteomyelitis. However, joint aspiration is the best next step for this patient.
- A 26-year old male arrives in the trauma bay following a roll-over motor vehicle accident with prolonged extrication. His heart rate is 130, his blood pressure is 70/50, and he exhibits confusion. A portable anteroposterior pelvic radiograph is shown is Figure A. Despite administering fluids, blood products in a 1:1:1 ratio, and applying a pelvic binder, his clinical status has not improved. A focused assessment with sonography for trauma (FAST) exam is negative.
What is this patients's class of hemorrhagic shock and the next best step in management?
- Class II; continue medical resuscitation in the intensive care unit
- Class II; emergent application of an external fixator
- Class III; emergent pelvic angiography
- Class III; emergent application of an external fixator
- Class IV; exploratory laparotomy CORRECT ANSWER: 3
This patient has an anteroposterior compression (APC) pelvic ring injury and, despite resuscitation with fluids, blood products, and the application of a pelvic binder, he has failed to improve clinically and remains in class III hemorrhagic shock (pulse rate 120-140, decreased blood pressure, anxiety/confusion). The next best step would be to proceed with emergent pelvic angiography.
Hemorrhage is the leading cause of death in pelvic ring injuries. In a patient exhibiting hemodynamic instability in the setting of a pelvic ring injury, a pelvic binder should be placed as soon as possible while the patient is being resuscitated with fluids and blood products. If the patient stabilizes, they can be treated with an external fixator or with open reduction and internal fixation. If the patient remains hemodynamically unstable, treatment options include pelvic angiography and possible embolization versus exploratory laparotomy and pelvic packing.
Verbeek et al. performed a retrospective study looking at "pelvic blush" on early computed tomographic scans in pelvic ring fracture patients and the
correlation with prognosis. They concluded that the finding of a pelvic blush does not obligate intervention and that treatment should be guided by patient clinical signs and hemodynamics.
Magnussen et al. performed a retrospective study to quantify transfusion requirements in patients with isolated acetabular or pelvic fractures and correlated these with fracture classification. They concluded that APC II or III, lateral compression (LC) III, vertical shear, and combined mechanism injuries had greater transfusion requirements than other pelvic ring injuries. In terms of acetabular fractures, T-type fractures and those involving the anterior column had higher transfusion requirements.
Cullinane et al. performed a systemic review and provided guidelines for managing blood loss in pelvic fractures. They addressed which patients with hemodynamically unstable pelvic fractures warranted emergent external fixation versus emergent angiography versus preperitoneal packing. They also addressed the workup of pelvic hemorrhage, radiologic findings, and the role of pelvic binding devices.
Figure A is a portable anteroposterior pelvic X-ray demonstrating diastasis of the pubic symphysis (APC-2). Illustration A provides an overview of the classification of hemorrhagic shock. Illustration B shows the arterial blood supply to the pelvis.
Incorrect Answers:
Answer 1: This patient is in class III hemorrhagic shock. Continued observation and medical resuscitation is not indicated, as the patient's continued source of bleeding must be addressed.
Answer 2: This patient is in class III hemorrhagic shock.
Answer 4: Emergent application of an external fixator may stabilize and close down the pelvic volume, however, a pelvic binder was previously applied and was unsuccessful.
Answer 5: This patient is in class III hemorrhagic shock. An exploratory laparotomy is an option in some institutions without angiography capabilities or when the patient is already undergoing an emergent laparotomy for other injuries.
- Which of the following radiographs demonstrate fracture fixation with buttress plating?
- Figure A
- Figure B
- Figure C
- Figure D
- Figure E CORRECT ANSWER: 1
Figure A is a lateral radiograph of the knee demonstrating a plate buttressing a posteromedial tibial plateau fracture.
Buttress plating is a construct that resists axial load by applying force at 90° to the axis of deformity. Compression plates function by compressing together the main fragments of a single fracture plane. They can result in absolute stability by eliminating interfragmentary movement. Neutralization plates protect fractures that are fixed with primary lag screw fixation. The lag screw exerts interfragmentary compression and the plate resists or neutralizes bending and rotational forces. Bridge plating is a technique which spans comminuted fractures and is attached to the main fragments. It is used to restore length, alignment, and rotation. It preserves the biology of the comminuted areas and allows for secondary bone healing and callus formation.
Zheng et al. performed a biomechanical study to examine four different fixation methods of posteromedial tibial plateau fractures. They stabilized 28 tibial models randomly with anteroposterior lag-screws, an anteromedial limited contact dynamic compression plate (LC-DCP), a lateral locking plate, or a posterior T-shaped buttress plate. They measured vertical subsidence and maximum load to failure. They found that the posterior T-shaped buttress plate allowed the least subsidence of the posteromedial fragment and produced the highest mean failure load.
Ratcliff et al. performed a study to compare the mechanical stability of a medial tibial plateau fracture model secured with a lateral locking periarticular plate versus a medial buttress plate in cyclic testing and load to failure. They created fractures in 6 matched pairs of fresh cadaveric tibias and randomly selected fixation. They found neither the mean maximum displacement during nor mean residual displacement after cyclic testing were statistically different, but the medial buttress plate constructs provided greater fixation strength with a greater load to failure. They conclude the medial buttress plate provides significantly greater stability in static loading.
Figure A is a lateral radiograph of the knee demonstrating a plate fixation of a posteromedial tibial plateau fracture. Figure B is an AP radiograph of the tibia and fibula demonstrating a tibia fracture stabilized with a plate. A fracture of the fibula is noted as well. Figure C is a lateral radiograph of the wrist demonstrating a dorsal spanning plate stabilizing a comminuted distal radius fracture. Figure D is an AP radiograph of the forearm demonstrating plate fixation of radial and ulnar shaft fractures. Figure E is a lateral of the wrist demonstrating lag screw and plate fixation of a radial shaft fracture.
Incorrect Answers:
Answer 2: The plate is stabilizing this fracture with a bridging technique which leads to relative stability and secondary bone healing.
Answer 3: The plate is stabilizing this fracture with a bridging technique which leads to relative stability and secondary bone healing.
Answer 4: These plates are functioning as compression plates which leads to absolute stability and primary bone healing.
Answer 5: Given the presence of a lag screw, this plate is functioning as a neutralization plate which leads to absolute stability and primary bone healing.
- A 57-year-old woman underwent open reduction internal fixation from a volar approach for a displaced distal radius fracture. Immediate
post-operative radiographs are seen in Figure A. The patient recovered well initially but presents after 6 months with grip weakness. What complication is most likely to occur in this patient?
- Inability to extend the index finger proximal interphalangeal joint.
- Inability to flex the index finger proximal interphalangeal joint.
- Inability to extend the thumb interphalangeal joint.
- Inability to flex the thumb interphalangeal joint.
- Inability to abduct the thumb.
CORRECT ANSWER: 4
A complication of very distal or prominent volar distal radius plate placement as seen in Figure A is rupture of the flexor pollicis longus (FPL) tendon. This would cause an inability to flex the thumb interphalangeal (IP) joint.
Flexor and extensor pollicis longus ruptures are known complications of distal radius fracture fixation. Volar plate placement distal to the watershed line, or prominence at the volar lip, can result in tendinopathy and eventual rupture of the FPL. With proper plate placement but screws protruding through the dorsal cortex, the extensor tendons are at risk, particularly extensor pollicis longus (EPL). Nonoperative treatment of distal radius fractures is also associated with EPL rupture.
Soong et al. compared flexor tendon ruptures between patients treated with 2 different volar distal radius plate designs. Most notably, they create and use a volar prominence grading system and demonstrate more tendon ruptures with 1 plate design that was more often found to be prominent and distal, suggesting position to contribute to tendon rupture.
Kitay et al. performed a case-control study assessing distal radius volar plate position in patients who did or did not develop flexor tendon ruptures. They found a significant association, reporting that plate position 3.0mm distal to the volar rim or prominence 2.0mm volar to the critical line (Soong grading system) each had an 88% sensitivity for flexor tendon rupture.
Figure A is a lateral radiograph of a distal radius fracture treated with a volar locking plate with distal prominence.
Illustration A shows the Soong classification of distal radius volar plate prominence, displaying the volar critical line as a line parallel to the volar radius shaft drawn tangentially from the most volar part of the distal volar rim (red line). Grade 0 is plate below this line, Grade 1 is plate crossing this line but proximal to volar rim, Grade 2 is plate crossing this line and distal to volar rim.
Incorrect Answers:
Answer 1: This would be rupture of extensor indices or central slip of index finger.
Answer 2: This would be rupture of flexor digitorum superficalis (FDS) to index finger.
Answer 3: This would be rupture of extensor pollicus longus (EPL). Answer 5: This would be rupture of either abductor pollicis brevis (APB) or longus (APL).
Rupture of FPL is more common with distal volar plate placement than the above tendons.
- An 80-year-old female sustained a twist and fall earlier at her home. She currently complains of left hip pain. On physical examination, she has hip pain with motion and is tender to palpation over her greater trochanter. Her injury radiographs are shown in Figure A. Which of the following should be performed next in this patient's care?
- Fixation with sliding hip screw
- Fixation with cephalomedullary nail
- Traction, internal rotation radiograph of the hip
- Computed tomography of the proximal femur
- Magnetic resonance imaging (MRI) of the proximal femur CORRECT ANSWER: 5
This patient sustained a greater trochanteric (GT) fracture. Evaluation with MRI is the next best step in treatment to evaluate for intertrochanteric (IT) extension.
Isolated fractures of the GT are uncommon. They may be diagnosed on radiography, but the extent of this injury is difficult to ascertain based on radiography alone. MRI has found a role in defining the extent of the fracture line. Though isolated GT fractures are often treated conservatively when they occur in isolation, operative fixation is necessary if there is IT extension. If the extent of the injury is not identified, it may lead to delayed rehabilitation and decreased long-term survival rates.
Kim et al. performed a systematic review to determine what proportion of GT fractures had IT extension and what are the treatment options of GT with occult IT extension. Patients included had an isolated GT fracture on radiographs and underwent MRI scans to determine IT extension. In 99/110 patients (90%), MRI revealed IT extension and surgical fixation followed in 61 patients. They conclude that MRI should be performed in patients presenting with an isolated GT fracture on plain radiograph.
Lee et al. examined the pattern and extent of an injury shown by MRI and radionuclide bone scan (RBS) in patients whose radiographs revealed fractures
limited to the GT. 25 patients were found sustaining a low-energy injury. They found that 3 patients had ITC extension that was not visualized with radiographs or RBS. 8 patients had fractures with MRI evidence of complete extension across the ITC region and 12 had incomplete extension. 11/12 patients with incomplete extension had extension more than 50% to the medial cortex. They recommend that all patients presenting with an isolated GT fracture on the plain radiographs should undergo MRI examination.
Collin et al. performed a study to evaluate the efficacy of CT in diagnosing occult hip fractures compared to MRI. They evaluated 44 consecutive elderly patients with trauma to the hip. All patients had negative CT scans while MRI changed the diagnoses in 27 cases. They conclude that MRI is a more reliable modality for hip fracture diagnosis in comparison to CT and negative CT finding cannot rule out a hip fracture in patients with a clinical suspicion for a hip fracture.
Figure A is an AP pelvis radiograph demonstrating a fracture of the left greater trochanter. Illustration A is a radiograph demonstrating a GT fracture and Illustration B is a T1-weighted MRI of the same patient demonstrating IT extension.
Incorrect Answers:
Answers 1 and 2: Treatment should not be performed until more information with an MRI is obtained.
Answer 3: Traction, internal rotation views are obtained in displaced intertrochanteric fractures to visualize the fracture fragments more accurately and to assess the efficacy of a closed-reduction maneuver.
Answer 4: CT scan of the proximal femur is not the best study to rule out IT extension of this fracture.
- A 45-year-old male with well-controlled diabetes and hypertension is involved in a high-speed motor vehicle collision. He is complaining of left knee pain only. On physical examination, his skin is intact and his neurovascular examination is normal. His injury films are seen in Figure A. Which of the following places this patient at an increased risk for postoperative infection after open reduction and internal fixation (ORIF)?
- Intraoperative time over 3 hours
- Age
- Fracture Pattern
- Medical comorbidities
- Mechanism of injury CORRECT ANSWER: 1
Intraoperative times approaching 3 hours have been associated with an increased risk of infection after undergoing ORIF of tibial plateau fractures.
The optimal treatment for displaced tibial plateau fractures is ORIF. The goals
of care are preservation of the soft tissues, restoration of the mechanical axis, and restoration of the articular surface. These injuries are associated with complications such as infections, arthrofibrosis, malunion/nonunion, and compartment syndromes. Infections have been associated with male gender, smoking, pulmonary disease, bicondylar fracture patterns, and intraoperative time over 3 hours. Modern techniques such as delay of definitive surgery, the use of temporary spanning external fixators, and dual incision approaches have improved the results of ORIF.
Basques et al. performed a database study to identify factors that are associated with short-term outcomes after ORIF of tibial plateau fractures. They examined adverse events (AAE), severe adverse events (SAEs), infectious complications, extended length of stay (LOS), and readmission within 30 days. They found that AAE was associated with increased ASA class and history of pulmonary disease. SAE was associated with male sex and increased ASA class. Infectious complications were associated with male sex, increased ASA class, smoking, pulmonary disease, and bicondylar fracture patterns.
Colman et al. performed a retrospective study to identify the relationship between surgical site infection and prolonged operative time in fractures of the tibial plateau. They found that mean operative time for patients who had an infection was 2.8 hours vs. 2.2 hours for patients without an infection. They also found that compartment syndromes that underwent fasciotomy had a higher infection rate than patients that did not develop this complication. Open fracture grade was also related to infection rate. They concluded that operative times approaching 3 hours and open fractures are related to an increased overall risk for surgical site infection.
Figure A is an AP radiograph of the knee demonstrating an intra-articular split of the lateral tibial plateau (Schatzker 2). Illustration A is an illustration of the Schatzker classification of tibial plateau fractures.
Incorrect Answers:
Answer 2: Age has not been associated with an increased risk of infection after ORIF of the tibial plateau.
Answer 3: An increased risk of infection after ORIF of the tibial plateau has been associated with bicondylar fracture patterns.
Answer 4: Well controlled diabetes and hypertension have not been associated with an increased risk of infection after ORIF of the tibial plateau.
Answer 5: Mechanism of injury has not been associated with an increased risk of infection after ORIF of the tibial plateau.
- A 33-year-old male sustains the injury seen in Figures A and B after a fall from a train station platform. Exam shows a closed, neurovascularly intact right lower extremity. The surgeon is taking the patient to the operating room for spanning external fixation for soft tissue rest followed by delayed open reduction internal fixation. Which of the following is true regarding outcomes with and without fibular fixation?
- Fixation of the fibula leads to lower rates of anatomic fracture reduction
- Fixation of the fibula leads to improved patient satisfaction scores
- Fixation of the fibula leads to better outcomes with regard to final alignment
- Fixation of the fibula leads to higher overall complication rates
- Fixation of the fibula leads to higher rates of radiographic arthrosis
CORRECT ANSWER: 4
Fixation of the fibula in tibial plafond fractures has been shown to lead to higher overall complication rates.
Tibial plafond fractures result from high energy axial load patterns and are associated with notoriously high complication rates. Management strategies often include temporary external fixation to allow for soft tissue rest followed by definitive open reduction internal fixation. Fixation of the fibula can be performed during initial external fixation or during delayed open reduction internal fixation. Instrumentation of the fibula may be done to help with reduction of the tibia, however, malreduction of the fibula may lead to difficulties in reduction of the tibia and articular surface. Fixing or not fixing the fibula in these scenarios remains controversial but it appears that fixation of the fibula may lead to higher rates of complication and hardware removal.
Kurylo et al. reviewed 93 patients treated with surgical fixation for tibial plafond fractures. When comparing patients with the fibula fixed versus those without fibular fixation, they found no difference in post-operative or final radiographic alignment. They concluded that fibular fixation is not always necessary in treating tibial plafond fractures and leads to higher rates of hardware removal.
Babis et al. reviewed long-term outcomes of 67 tibial plafond fractures treated with open reduction internal fixation, external fixation, or a combination of the two. They found, at average follow up of 8.1 years, good outcomes were associated with lower Reudi-Allgower classification, use of AO technique, and adequacy of fracture reduction. They concluded that these three factors can predict subjective and objective outcomes following tibial plafond fractures.
Williams et al. retrospectively reviewed patients with tibial plafond fractures treated with definitive external fixation with or without fibular fixation. They found no difference in outcomes scores, radiographic arthrosis, or complication rates. They found different types of complications among the two groups with higher rates of fibular wound infection in the group with fibular fixation. They concluded that tibial plafond fractures treated with definitive external fixation can yield good clinical results without fixing the fibula.
Figures A and B show an AP and lateral of the right ankle, respectively, with a comminuted tibial plafond fracture and associated transverse fracture through the fibula
Incorrect Answers:
Answer 1, 2, 3, & 5: Fracture reduction, patient satisfaction scores, final
alignment and rates of radiographic arthrosis are not statistically different when comparing fibular fixation versus no fixation in tibial plafond fractures.
- The structure injured in Figure A occurs most frequently in which of the following injury patterns depicted in figures B though F?
- B
- C
- D
- E
- F
CORRECT ANSWER: 4
The ACL is most commonly injured in Schatzker IV and VI injury patterns; Type IV injuries are also associated with medial meniscus disruption, vascular injury, and compartment syndrome.
Tibial plateau fractures are the result of an axial load with a varus, valgus, or rotational moment. As such, several soft tissue and ligamentous injuries often result, with higher energy mechanisms associated with more soft tissue injuries. Injury to the ACL occurs most commonly in Type IV and VI injuries. Magnetic resonance imaging may be useful at identifying associated soft tissue injuries in tibial plateau fractures. However, the timing and treatment of associated soft tissue injuries in this setting remains a controversial topic.
Gardner et al. evaluated the incidence of associated soft tissue injuries in tibial plateau fractures utilizing MRI in prospective cohort study of 103 patients.
They found that the incidence of soft tissue injury was higher than previously reported; only 1 patient in their study lacked a soft tissue insult. They also found that 77% of patients sustained injury to 1 or more cruciate ligaments, 91% sustained lateral meniscus injury, 44% sustained medial meniscus injury, and 68% of patients injured 1 or more posterolateral corner structures.
Abdel-Hamid et al. retrospectively evaluated soft tissue injuries in 98 tibial plateau fractures that were initially treated with arthroscopy-assisted surgical fixation. They found that meniscal injuries and bony ACL avulsions were most commonly seen in their series. Additionally, they found a statistically significant association between ACL injuries in Type IV and VI injuries.
Wang et al. studied the incidence of soft tissue injuries in tibial plateau fractures and the relationship of articular widening and depression with meniscal and ligamentous injuries in 54 patients. They found that the meniscus was the most commonly injures structure, occurring in 56% of the patients in their series. Finally, they found a correlation between articular widening and depression with a higher incidence of soft tissue injury.
Figure A is a sagittal MRI depicting an ACL tear. Figures B through F are radiographs depicting Schatzker I, II, III, IV, and V injuries, respectively. Illustration A is a diagram showing the Schatzker classification.
Incorrect Answers:
Answer 1: This is a Type I injury
Answer 2: This is a Type II injury, associated with lateral meniscus disruption Answer 3: This is a Type III injury
Answer 5: This is a Type V injury
- A 21-year-old male sustained an injury to his right knee during a football game. Radiographs obtained are shown in figures A and B. Pulses and ABI's are normal, however reports decreased sensation over the dorsum of the foot and the first webspace with weakness in great toe extension. The nerve injury most likely occurred at which of the following locations seen in Figure C?
- A
- B
- C
- D
- E
CORRECT ANSWER: 1
The most common nerve injury in a posterolateral knee dislocation and the symptoms the patient is experiencing suggest an injury at the level of the common peroneal (fibular) nerve.
Knee dislocations are often accompanied by an associated neurovascular injury given the amount of translation with relative tethering of the neurovascular structures above and below the knee. Vascular injury to the popliteal artery represents a limb-threatening injury. Popliteal artery injury can occur in up to 40% of knee dislocations so vascular examination pre and post reduction is essential. The most commonly injured nerve following a posterolateral knee dislocation is the common peroneal nerve, with motor weakness consisting of foot drop and sensory changes in the deep and superficial peroneal nerve distributions.
Woodmass et al. performed a systematic review to identify prognostic factors for recovery after common peroneal nerve (CPN) injuries following knee dislocations. They found full recovery was 87.3% after partial CPN palsies and only 38.4% after completed CPN palsy. They concluded that outcomes following knee dislocations remain poor and the most reliable method for reestablishing antigravity dorsiflexion is posterior tibial tendon transfer.
Poage et al. reviewed peroneal nerve palsies including workup and treatment options. They discuss the nerve anatomy as it branches from the sciatic nerve in the proximal popliteal fossa, then crosses posterior to the lateral gastrocnemius muscle, and finally curves around the head of the fibula. They state the most common etiology of peroneal nerve palsies are compressive in nature, however, traumatic etiology has the worst prognosis overall.
Figure A shows an AP radiograph of a right knee with lateral translation of the tibia relative to the femur. Figure B shows a lateral knee radiograph of a right knee with posterior translation of the tibia relative to the femur. Figure C shows an unlabeled illustration of a selection of nerves of the lower extremity. Illustration A shows the same illustration with appropriate labeling of the selected nerves.
Incorrect Answers:
Answer 2: Choice B represents the tibial nerve after branching off of the sciatic nerve.
Answer 3: Choice C represents the sural nerve seen as a combination of fibers from the common peroneal nerve and the tibial nerve.
Answer 4: Choice D represents the deep peroneal (fibular) nerve distal to the common peroneal nerve
Answer 5: Choice E represents the superficial peroneal (fibular) nerve seen coursing distally and laterally after branching from the common peroneal nerve.
- A 29-year-old male competitive snowboarder presents to your clinic with ankle pain following a fall 5 days prior. He says he saw an orthopedic surgeon following the injury and was told he had an ankle sprain. Figures A and B show his injury on radiograph and CT scan. Which of the following matches the correct diagnosis with the most appropriate treatment option?
- Lateral process of the talus fracture - fragment excision
- Lateral process of the talus fracture - open reduction internal fixation
- Anterior process of the calcaneus fracture - fragment excision
- Anterior process of the calcaneus fracture - open reduction internal fixation
- Posterior process of the talus fracture - fragment excision CORRECT ANSWER: 2
The patient has an intraarticular noncomminuted lateral process of the talus fracture that would be best treated with open reduction internal fixation.
Lateral process of the talus fractures occur commonly in snowboarders following a twisting injury while the foot is strapped to the board. Diagnosis is often missed and confused with a lateral ankle sprain. Close examination of plain films may show a lateral process of the talus fracture or CT scan may be warranted if radiographs are negative and suspicion is high. Once a lateral process of the talus fracture is identified, fracture comminution, displacement, and angulation determine if nonoperative treatment is possible. In cases of noncomminuted intraarticular fractures open reduction internal fixation is advised. In cases of comminuted intraarticular fractures that fail nonoperative management, fragment excision is indicated and is not felt to lead to ankle instability based on cadaveric testing.
von Knoch et al. followed 23 snowboarders with lateral process of the talus fractures treated operatively and nonoperatively. They found 65% of patients returned to preinjury level of sporting and 45% had radiographic evidence of arthritis. They concluded that outcomes are favorable following lateral process of the talus fractures given early identification and treatment.
Langer et al. studied 10 fresh frozen cadavers for instability following excision of a 1cm area of the lateral process of the talus. They found the mean increase in anterior tibial translation (AT), talar tilt (TT), medial talocalcaneal motion, and talocalcaneal tilt (TCT) were all less than the accepted cut-offs for ankle and subtalar instability. They conclude excision of a 1cm fragment of the lateral process of the talus leads to neither ankle or subtalar instability.
Berkowitz et al. reviewed the various tubercle and process fractures of the talus and calcaneus. They state when a fracture fragment is <1cm and <2mm displaced nonoperative treatment in a non-weight bearing short leg cast for 6 weeks is recommended. They recommend large noncomminuted intraarticular fractures be treated with open reduction internal fixation and comminuted intraarticular fractures be primarily excised. Chronic painful nonunions respond best to delayed surgical excision.
Figure A shows a mortise view of the ankle with evidence of a lateral process of the talus fracture. Figure B shows a coronal CT scan with a displaced noncomminuted intraarticular lateral process of the talus fracture.
Incorrect Answers:
Answer 1: The lateral process of the talus is the correct diagnosis but this fragment is amenable to open reduction internal fixation.
Answer 3 & 4: There is no radiographic evidence of an anterior process of the calcaneus fracture
Answer 5: A posterior process of the talus fracture would be seen on a lateral view of the ankle which is not seen.
- A 79-year-old female sustained a slip and fall. Injury films are shown in Figures A, B, and C. She has made limited progress with 3 weeks of physical therapy and continues to endorse severe low back pain and difficulty ambulating. What is the next best step in treatment?
- Examination under anesthesia
- Observation and physical therapy
- Magnetic resonance imaging (MRI) of the pelvis
- Bone scan
- Percutaneous fixation CORRECT ANSWER: 5
This osteoporotic patient has sustained a sacral insufficiency fracture. At this point, given her difficulty ambulating and severe pain percutaneous fixation is the best option.
Sacral fragility fractures encompass both atraumatic sacral fractures as well as sacral fractures that occur as a result of low energy mechanisms in elderly patients with osteopenic bone. Over time, sacral fragility fracture incidence has increased with the aging population. Morbidity of non-op treatment includes pain, loss of independence, need for institutionalization, as well as the ills of recumbency. A number of authors have found that percutaneous iliosacral stabilization of these fractures leads to pain relief and therefore a higher likelihood of early ambulation.
Sanders et al. performed a study to evaluate the effectiveness of transsacral- transiliac screw fixation for the treatment of sacral insufficiency fractures that fail nonoperative treatment. They compared preoperative and postoperative Visual Analog Scale scores and Oswestry Low Back Disability Index scores.
They found that patients experienced statistically significant improvement in both outcome measures after intervention and no complications were encountered. They concluded that transsacral-transiliac screw fixation is a safe and effective treatment for sacral insufficiency fractures recalcitrant to nonoperative management.
Rommens et al. propose a novel classification system for fragility fractures of the pelvic ring. The classification is based on morphological criteria and it corresponds to the degree of instability. They hope that this criterion will help guide treatment based on the degree of instability based on radiological and clinical findings.
Figure A is an AP pelvis radiograph demonstrating no osseous abnormalities. Figures B and C are axial and coronal CT cuts, respectively, demonstrating, fractures of the sacrum. Illustration A is an AP pelvis radiograph following percutaneous skeletal fixation.
Incorrect Answers:
Answer 1: This patient has failed non-operative treatment and therefore requires surgical stabilization. Examination under anesthesia is helpful in diagnosing subtle instability patterns that may not be visible with static imaging.
Answers 2: This patient has failed non-operative treatment and therefore requires surgical stabilization. Sacral fragility fractures may be treated with a trial of weight-bearing and repeat imaging to assess for displacement.
Answers 3 & 4: Additional imaging studies are not necessary in this case as the diagnosis can be made from the CT scan. If no injury is visible on the CT scan and the patient continues to have pain, additional imaging may be warranted.
- Which of the following philosophies in resuscitation of the polytrauma patient utilizes a lactate of < 4.0 mmol/L, a pH >= 7.25, or a base excess of >= -5.5 mmol/L to guide definitive fracture care and is associated with a decreased delay to surgery?
- Early Total Care
- Early Appropriate Care
- Damage Control Orthopaedics
- Early Definitive Care
- Life-Over-Limb CORRECT ANSWER: 2
Early Appropriate Care (EAC) utilizes the above physiologic parameters to proceed with definitive fracture treatment when one of the three are met and is associated with a decreased incidence of ARDS, MOF, mortality, and shorter lengths of stay when polytrauma patients are treated within the first 36 hours of presentation.
The philosophy of Early Total Care (ETC), the concept that all fractures should be fixed in one trip to the operating room as soon as possible, was developed in the 1980s. This approach of ETC exacerbated the second-hit phenomenon especially in patients with severe chest trauma managed acutely with intramedullary fixation of long bone fractures. Because of this Damage Control Orthopedics (DCO) emerged in 2000 and focused on approaching polytrauma patients with a goal of minimizing the impact of the "second-hit" through delayed definitive treatment until physiology is improved (stabilization over fixation). Subsequently, the concept of Early Appropriate Care (EAC) was developed in 2013, aiming to identify major trauma patients and definitively
treat the most time-critical injuries without exacerbating the secondary inflammatory response. Utilizing a lactate of < 4.0 mmol/L, pH ≥ 7.25, or a base excess ≥ -5.5 mmol/L, definitive fracture care can proceed when any of these criteria are met.
Vallier et al. developed a protocol to determine the timing to definitive fracture care based on adequacy of resuscitation at a Level 1 trauma center. In a prospective study, they examined patient outcomes following definitive fixation of pelvis, acetabulum, spine, and femur fractures within 36 hours of injury, with timing based on parameters for acidosis. They found that surgeon preference was the most common reason for delay in definitive treatment with this protocol, but, after 2 years of implementation, only 10% of fractures were definitively treated outside of this protocol.
Vallier et al., in a follow-up study, again advocated for Early Appropriate Care with the goal of standardizing resuscitation assessment and expediting fracture care to reduce length of stay and improve hospital revenue. They found that delayed (> 36 hours) fixation of femur, pelvis, or spine fractures resulted in more complications, prolonged hospital stay, and a mean decrease in facility collections of 5%.
Vallier et al. further describe the development of their protocol for early appropriate care utilizing a statistical model based on a retrospective database of 1,443 adults with pelvis, acetabulum, spine, and femur fractures. They found that an uncomplicated course was associated with the absence of an associated chest injury and definitive fixation within 24-48 hours. They conclude that acidosis on presentation is associated with complications and developed a predictive model based on acidosis, chest injury severity, number of fractures, and timing to definitive fixation.
Incorrect Answers:
Answer 1: Early Total Care is the concept that all fractures should be fixed in one trip to the operating room.
Answer 3: Damage Control Orthopedics is the concept of approaching polytrauma patients with the goal of minimizing the impact of the "second hit", whereas definitive treatment is delayed until physiology is improved.
Answers 4 and 5: Early Definitive Care and Life-Over-Limb are not defined treatment philosophies in the management of polytrauma patients.
- An 86-year-old female sustains the injury shown in Figure A. On admission, she is diagnosed with acute-on-chronic heart failure. She is
subsequently admitted to the medicine service. While waiting for optimization of her medical co-morbidities prior to surgery, which intervention can reduce the patient's overall narcotic use?
- Fascia iliaca block
- Patient-controlled analgesia (PCA) machine
- Distal femoral skeletal traction
- Pain service consult
- Prophylactic haloperidol treatment CORRECT ANSWER: 1
In patients with a hip fracture, a pre-operative fascia iliaca block has been shown to decreased overall narcotic use.
Hip fractures are the most common non-vertebral fracture in patients over the age of 65 and represent a significant source of morbidity and mortality. These fractures are inherently painful, but the use of narcotics in this vulnerable patient population has significant risk factors, including increased risk of falls, delirium, constipation, and respiratory depression. The use of non-opioid pain management strategies, including blocks, can reduce the risks associated with narcotic pain medications.
Candal-Couto et al. assessed the efficacy of a pre-operative fascia iliaca block in 30 elderly patients with hip fractures using an objective sitting score,
assessing the degree of passive hip flexion, and through the visual analog scale. At one hour following the block, there was a significant improvement in all three measured outcomes. The authors concluded that a fascia iliaca block can provide significant benefit and allow patients to sit more comfortably while awaiting surgery.
Callear et al. performed a prospective study of 82 patients over the age of 65 with a hip fracture to evaluate the efficacy of pre-operative fascia iliaca blocks. They found that patients who received a pre-operative block received significantly less post-operative and total opioids, has significantly less delirium, and had shorter inpatient stays compared to patients who did not receive a fascia iliaca block. The authors' work demonstrating the efficacy of the block led to the introduction of a new hip fracture care pathway which incorporated pre-operative fascia iliaca blocks.
Figure A shows a radiograph of a right hip with a displaced femoral neck fracture. Illustration A shows a depiction of a fascia iliaca block.
Incorrect Answers:
Answers #2, 3, and 5: None of these interventions has been shown to decrease overall narcotic use in geriatric patients with hip fractures.
Answer #4: While a pain service consult may potentially lead to a fascia iliaca block, pain service consults by themselves have not been shown to lead to decreased overall narcotic use in this patient population.
- Which of the following techniques after surgical stabilization of patella fractures has the highest risk of hardware migration?
- Figure A
- Figure B
- Figure C
- Figure D
- Figure E
CORRECT ANSWER: 1
Fixation of patella fractures with a tension-band construct consisting of K-wires that are only bent proximally has the highest risk of undergoing hardware migration (Figure A).
There are many techniques described in operative fixation of patella fractures. Historically, tension band wiring has been the most commonly utilized fixation strategy. The classic technique consists of 2 K-wires and a figure of eight cerclage wiring with bending of the wires proximally. This has been associated with a risk of hardware migration and subsequently high incidence of hardware removal. Though migration of hardware may certainly be associated with other fixation strategies, it is the highest for tension band wiring with K-wires.
Eggink et al. performed a study to evaluate the clinical and radiological results of internal fixation of patella fractures, comparing tension band wiring with the proximal wire bent and both the proximal and distal wires bent. Of 60 patients, they found migration of the K-wires in 3 patients. All of these migrations occurred in the group with the proximally bent wires. They recommend bending of K-wires both proximally and distally to avoid this potential complication.
Smith et al. performed a study to identify and review early complications in the operative treatment of patella fractures. They used modified tension band wire fixation in forty-nine fractures, whereas two fractures were treated with tension band wires threaded through cannulated screws. They found that 22% of patients treated with tension band wiring had fracture displacement within the early postoperative period and 9 patients required hardware removal. They conclude that the incidence of early complications in patients treated with tension band wiring is higher than previously reported.
Figure A is an AP radiograph of the knee demonstrating fixation with a tension band wiring technique utilizing K-wires that are only bent proximally. Figure B is an AP radiograph of the knee demonstrating fixation with an anteriorly based plate. Figure C is a lateral radiograph of the knee demonstrating fixation with multiple small plates. Figure D is a lateral radiograph of the knee demonstrating fixation with cannulated screws and a cerclage wire. Figure E is a radiograph demonstrating fixation with a tension band wiring technique utilizing K-wires that are bent both proximally and distally.
Incorrect Answers:
Answers 2, 3, 4, & 5: These fixation strategies have not been associated with hardware migration.
- A primarily sedentary 76-year-old female falls and complains of left shoulder pain. Radiographs and CT scan are shown in Figures A-C. She is vascularly intact but has decreased sensation over the lateral arm and ulnar hand with the inability to cross her fingers. Which of the following is the best definitive management?
- Sling and serial examination for resolution of neuropraxia
- Closed reduction under conscious sedation in the emergency department
- Open reduction internal fixation
- Closed reduction and percutaneous pinning
- Arthroplasty CORRECT ANSWER: 5
An elderly patient with low functional demands and a comminuted proximal humerus fracture-dislocation with poor bone quality is best treated with a hemiarthroplasty or reverse total shoulder arthroplasty.
Proximal humeral fracture-dislocations in the elderly occur following low energy falls resulting in significant comminution due to poor bone quality. Treatment of these fractures with open reduction internal fixation accepts a significant risk of loss of fixation, osteonecrosis, or screw penetration into the joint.
Hemiarthroplasty can be considered, however, functional status largely depends on the healing of the tuberosities which may be difficult in patients with poor bone quality. Acute reverse total shoulder arthroplasty in this setting of a low demand elderly patient is associated with predictable functional outcomes with less reliance on the healing of the tuberosities.
Robinson et al. present their series of 58 patients with proximal humerus fracture-dislocations treated with either open reduction or cemented hemiarthroplasty. They found those patients with complete detachment of capsular tissue and no back bleeding from the humeral head had a higher risk of going on to osteonecrosis. They conclude that patients <60 years of age
with intact capsular attachment and humeral head bleeding can be treated with open reduction internal fixation, while those >60 years of age and no humeral head bleeding or capsular attachment are best treated with hemiarthroplasty.
Demirhan et al. present 6 cases (average age 52) of iatrogenic fracture- dislocations treated with primary arthroplasty. On average, that found that patients achieved 124 degrees of forward flexion, 29 degrees of external rotation, and internal rotation to L2. They conclude that primary arthroplasty is the ideal treatment in the setting of iatrogenic fracture dislocations given the high risk of avascular necrosis and humeral head collapse with open reduction internal fixation.
Solberg et al. reviewed 70 patients >55 years of age with Neer 3- and 4-part proximal humerus fractures treated with locked plating. They found complication rates were as high as 79% in patients with initial varus deformity compared to 19% in patients with initial valgus deformity. They concluded that elderly patients with initial valgus angulation did better functionally and had lower complication rates compared to elderly patients with initial varus deformity.
Figure A shows a radiograph of a left proximal humerus fracture dislocation. Figures B and C show a CT scan of a proximal humerus fracture dislocation with anterior dislocation of the humeral head. Illustration A shows a postoperative radiograph of a reverse total shoulder replacement in the same patient.
Incorrect Answers:
Answer 1: Treating the patient definitively in a sling would leave the humeral head dislocated and likely result in chronic pain, osteonecrosis, and loss of function.
Answer 2: Closed reduction in the ER is not recommended, as gaining control of the humeral head in a fracture dislocation is often unsuccessful.
Answer 3: Open reduction internal fixation would not be recommended in this scenario given the expected humeral head necrosis and nonunion.
Answer 4: Closed reduction and percutaneous pinning is associated with high complication rates and would be unlikely to lead to early return of function.
- A 72-year-old female sustains a displaced proximal humerus fracture after a fall on ice. The patient subsequently undergoes open reduction and internal fixation of her fracture. Of the following answer choices, which correctly orders the predictive accuracy of humeral head ischemia from most to least predictive?
- calcar length less than 8 mm > humeral head angulation more than 45 degrees > head-split fracture > disrupted medial hinge
- head-split fracture > calcar length less than 8 mm > disrupted medial hinge
> humeral head angulation more than 45 degrees
- disrupted medial hinge > humeral head angulation more than 45 degrees > calcar length less than 8 mm > head-split fracture
- calcar length less than 8 mm > disrupted medial hinge > humeral head angulation more than 45 degrees > head-split fracture
- head-split fracture > disrupted medial hinge > calcar length less than 8 mm
> humeral head angulation more than 45 degrees CORRECT ANSWER: 4
The predictive accuracy of characteristics of humeral head ischemia, from most predictive to least predictive, are: calcar length less than 8 mm, disrupted medial hinge, humeral head angulation more than 45 degrees, and head-split fracture.
Proximal humerus fractures are the third-most common non-vertebral fracture in the elderly population and represent a treatment challenge for orthopaedic surgeons. Fracture displacement can lead to disruption of the anterior and posterior humeral circumflex arteries, causing humeral head ischemia. Recent evidence has demonstrated that the posterior humeral circumflex artery is likely the main blood supply to the humeral head. The humeral head does have the possibility to revascularize after injury, however, and the presence of humeral head ischemia in the acute injury setting is not a predictor of subsequent avascular necrosis.
Hertel et al. evaluated predictors of humeral head ischemia at the time of surgery in a prospective study of 100 intracapsular proximal humerus fractures. They found the most accurate predictive measures of humeral head ischemia, from most accurate to least accurate, were: a calcar length <8 mm, disruption of the medial hinge, basic fracture pattern, displacement of the humeral head >45 degrees, displacement of the tuberosities >10 mm, glenohumeral dislocation and head-split fractures (equally predictive). The authors concluded that the most relevant predictors of ischemia were the length of the posteromedial calcar, the integrity of the medial hinge, and the basic fracture type.
In a follow-up study, Bastian and Hertel evaluated the occurrence of avascular necrosis in 51 patients whose humeral head vascularity was analyzed at the time of surgery. They found there was no association between acute humeral head ischemia at the time of surgery and the development of avascular necrosis, but the patients who developed collapse of their head had significantly worse clinical outcome scores. The authors concluded that osteosynthesis with preservation of the humeral head is worth considering when adequate reduction can be obtained.
Xu et al. performed a systematic review and meta-analysis of 7 studies with a total of 291 patients to evaluate the risk of avascular necrosis among patients with proximal humerus fractures undergoing operative fixation compared to those who underwent nonoperative management. The authors found no significant differences in the incidence of AVN between the two groups; however, subgroup analysis of the fixation group found that plate fixation was associated with a higher rate of AVN than conservative treatment. The authors concluded that plate fixation was associated with a higher risk of AVN development than conservative treatment.
Illustration A demonstrates the vascular anatomy of the proximal humerus. Illustration B demonstrates differences in posteromedial calcar lengths in
proximal humerus fractures. Illustration C shows the differences between an intact and disrupted medial hinge. Illustration D is an illustration of a head-split fracture.
Answers 1,2,3 and 5: These answers do not correctly order the predictors of humeral head ischemia from most to least accurate.
- A 35-year-old male presents 6 weeks following open reduction and internal fixation of the injury shown in Figures A and B performed through a sinus tarsi approach. He is a 1/2-pack-per-day smoker and his body mass index (BMI) is 25. He last presented 3 weeks ago at which time his wound appeared healed, and the sutures were removed. Physical therapy was also initiated for gentle range of motion exercises. He reports that his main surgical incision "opened up" a few days ago and has been draining. He denies fevers or chills at this time. On physical exam, he has a small, 1 cm area of wound dehiscence with no exposed hardware. His current radiograph is
shown in Figure C. Which of the following combines the risk factors this patient has for wound dehiscence with the next best step in treatment?
- BMI, smoking; irrigation and debridement with removal of hardware
- Smoking, surgical approach; irrigation and debridement with retention of hardware
- BMI, surgical approach; cessation of physical therapy, immobilization, and nonsurgical wound management
- Surgical approach; irrigation and debridement with retention of hardware
- Smoking; cessation of physical therapy, immobilization, and nonsurgical wound management
CORRECT ANSWER: 5
Smoking is a risk factor for wound dehiscence after open reduction and internal fixation (ORIF) of calcaneal fractures. Superficial wound dehiscence should initially be treated with cessation of physical therapy, immobilization, and nonsurgical wound management.
The most common complication associated with ORIF of calcaneus fractures is delayed wound healing. The extensile lateral approach is the most commonly used approach for treating these injuries and has been associated with wound breakdown in nearly 25% of patients. The sinus tarsi approach utilizes a much smaller incision and studies have shown fewer wound healing difficulties. Pre- operative risk factors that predispose to wound breakdown include smoking, diabetes, open fractures, high BMI, and closure of the wound in a single layer. In the event of superficial wound breakdown, range of motion exercises should be stopped and a prophylactic course of antibiotics should be started with nonsurgical wound management. If purulence is encountered or the wound is found to be deep, hospitalization with serial debridements and antibiotics is required.
Clare et al. performed a review of how to manage complications of calcaneal fractures. They report that immobilization after wound breakdown should be performed with casting with a window over the wound to allow for wound care and wet-to-dry dressing changes. Once the wound is healed, range-of-motion exercises may be resumed.
Folk et al. performed a review of early wound complications of operative treatment of calcaneus fractures through the lateral extensile approach. They found that smoking, open fractures, and diabetes were risks for a wound complication. They recommend that patients with identifiable risk factors should be counseled of this potential complication.
Figure A is a lateral ankle radiograph demonstrating a displaced, intraarticular calcaneal fracture. Figure B is an axial CT image demonstrating the fracture and the sustentaculum tali. Figure C is a lateral ankle radiograph demonstrating fixation of the fracture with a plate and independent screws.
This fixation strategy is utilized along with a sinus tarsi approach. Illustration A is a radiograph illustrating the two approaches used to fix calcaneus fractures. The letter (a) represents the lateral extensile incision while (b) represents the sinus tarsi incision. The number (1) correlates to the location of the sural nerve, while (2) correlates with the superficial peroneal nerve.
Incorrect Answers:
Answer 1: This patient has a normal BMI and irrigation and debridement with removal of hardware is not indicated at this point as the fracture is not healed. Answer 2: The sinus tarsi approach is more forgiving of the soft tissues compared with the lateral extensile approach. Irrigation and debridement with retention of hardware is not indicated at this point.
Answer 3: The sinus tarsi approach is more forgiving of the soft tissues compared with the lateral extensile approach.
Answer 4: The sinus tarsi approach is more forgiving of the soft tissues compared with the lateral extensile approach. Irrigation and debridement with retention of hardware is not indicated at this point.
- An 85-year-old woman is found to have an isolated left hip fracture after a fall from standing. All of the following have been demonstrated to be a benefit of an orthopaedic geriatric comanagement service EXCEPT?
- Decreased time to surgery
- Decreased post-operative mortality
- Decreased post-operative complication rate
- Decreased need for post-discharge rehab facility
- Increased post-operative mobility CORRECT ANSWER: 4
All of the answers listed have been demonstrated to be a benefit of orthopaedic geriatric comanagement service EXCEPT for decreased need for a post- discharge rehab facility.
Many institutions have initiated an orthopaedic geriatric comanagement inpatient service, particularly for orthopaedic trauma patients. Such service assists with perioperative medical management, focusing especially on pre- operative optimization, post-operative management of comorbidities,
prevention of delirium, and management of dementia. Several studies have demonstrated improved post-operative complication and mortality rates though conflicting evidence exists.
Fisher et al. analyzed a prospective group of 951 patients older than 60 years of age with hip fractures managed on a comanagement service, comparing them to a historical control group. They report a reduction in postoperative medical complications (50% vs 71%), mortality (4.7% vs 7.7%), and re- admission rate to a medical service. They did not find a change in the length of stay or post-discharge disposition. They conclude that combined orthopaedic and geriatric care of elderly patients with hip fractures leads to decreased morbidity and mortality while improving postoperative care.
Friedman et al. analyzed a Geriatric Fracture Center with patients 60 years of age or older with hip fracture and report a shorter time to surgery (24 vs 37 hours), shorter length of stay (4.6 vs 8.3 days), fewer post-operative infections (2% vs 20%), fewer complications overall (31% vs 46%), and lower use of physical restraints (0% vs 14%). They conclude that comanagement of elderly patients with hip fractures leads to improved processes and outcomes.
Prestmo et al. conducted a randomized control trial of usual care vs orthogeriatric comanagement for home-dwelling patients older than 70 years of age with hip fracture. They found that geriatric comanagement patients were significantly more mobile at 4 months following surgery. They concluded that the treatment of geriatric patients with hip fractures should be coordinated through orthogeriatric care.
Incorrect Answers:
Answer 1, 2, 3, 5 - Decreased time to surgery, postoperative mortality, complication rates, and improved postoperative mobility have all been shown in the literature to be benefits of an orthogeriatric comanagement service.
- For which of the following fracture patterns would a sliding hip compression screw device be an appropriate treatment option?
- Figures A and C
- Figures A and D
- Figures A, E, and F
- Figures B and E
- Figures C and D CORRECT ANSWER: 3
A sliding hip compression screw device (SHS) would be appropriate for a standard obliquity intertrochanteric hip fracture with a stable lateral wall. Femoral neck stress fractures may also be treated with a SHS.
A SHS is indicated for stable intertrochanteric fractures and some basilar neck and vertical fracture patterns. A lesser trochanteric fracture is not a
contraindication to a SHS as long as the medial buttress lacks comminution and subtrochanteric extension. A contraindication to a SHS is a reverse obliquity intertrochanteric fracture in which the lateral buttress is compromised, as they are associated with SHS failure and cut-out. In these circumstances, an intramedullary nail (IMN) would be more appropriate.
Palm et al. evaluated the integrity of the lateral femoral wall as a predictor of reoperation in intertrochanteric hip fractures. They looked at 214 intertrochanteric fractures treated with a 135° SHS. They found that a postoperative fracture of the lateral femoral wall was the main predictor for reoperation and therefore concluded that patients with lateral femoral wall fractures should not be treated with SHS.
Sadowski et al. performed a prospective randomized study evaluating the treatment of reverse obliquity and transverse intertrochanteric fractures with an IMN versus a 95° screw-plate. 39 elderly patients were included in the study, with 19 patients treated with fixed-angle devices and 20 patients treated with IMN. They concluded that IMN was the better option for treating reverse obliquity and transverse IT fractures in elderly patients.
Baumgaertner et al. evaluated the value of the tip-apex distance in predicting failure of fixation of peritrochanteric hip fractures with SHS. They looked at 198 peritrochanteric fractures and reported 19 failures over an average follow- up time of 13 months. They determined that fractures with a tip-apex distance of 25 mm or less did not experience cut-out, but a strong relationship existed between an increasing tip-apex distance and cut-out rate.
Figures A is a radiograph of a standard obliquity intertrochanteric hip fracture with an associated lesser trochanteric fracture. Figure B is a reverse obliquity intertrochanteric fracture. Figure C is a subcapital femoral neck fracture. Figure D is a subtrochanteric femur fracture. Figure E is a standard obliquity intertrochanteric fracture. Figure F is a tension-sided femoral neck stress fracture. Illustration A shows an intertrochanteric fracture treated with a SHS. Illustration B demonstrates how to measure the tip-apex distance, which should be less than 25 mm to prevent screw cut-out.
Incorrect Answers:
Answer 1: Figure C is a subcapital femoral neck fracture that should be treated with arthroplasty versus ORIF in younger patients.
Answer 2: Figure D is a subtrochanteric femur fracture that is typically treated with IMN.
Answer 4: Figrue B is an reverse obliquity intertrochanteric fracture that is best treated with a cephalomedullary nail.
Answer 5: See Answers 1 and 2.
- Which of the following is an advantage of sliding hip screws compared to cephalomedullary nails for the treatment of appropriate
intertrochanteric femur fractures?
- Decreased risk of deep venous thrombosis
- Biomechanically advantageous under physiologic loading
- Decreased blood loss
- Decreased risk of nonunion
- None of the above CORRECT ANSWER: 5
There is no difference in rates of DVT or nonunion in comparing sliding hip screws(SHS) with cephalomedullary nails (CMN). SHS use is associated with more or equal blood loss and the implant design by being extramedullary is at a disadvantage compared to the intramedullary position of a CMN.
Many studies have looked at the pros/cons of sliding hip screws compared to cephalomedullary nails. The majority find few differences in the treatment of stable intertrochanteric hip fractures (AO 31A1.1 - 31A2.1). The advantages of CMNs include a biomechanically advantageous position and similar to reduced blood loss. In addition, CMNs are superior in fractures extending into the subtrochanteric region or reverse obliquity fractures. For unstable intertrochanteric patterns, most evidence supports CMN. In intertrochanteric patterns, there appears to be minimal to no increased risk of periprosthetic fracture for current generation short cephalomedullary nails compared to long cephalomedullary nails.
Socci et al. reviewed evidence surrounding implant choice for various intertrochanteric hip fractures. They review the AO classification and recommend SHS for 31A1 fractures and CMN for all other fracture patterns.
Roberts et al. reviewed the AAOS clinical practice guidelines for hip fractures in the elderly. They support the use of SHS or CMN for stable fractures with moderate evidence. Strong evidence supports CMN for reverse obliquity and subtrochanteric fractures. Moderate evidence supports CMN for unstable intertrochanteric fracture patterns.
Illustration A taken from Socci et al. demonstrates the AO classification of intertrochanteric hip fractures. 31A1 - 31A2.1 are typically stable patterns while 31A2.2 to 31A3.3 are considered unstable patterns.
Incorrect Answers
Answer 1: There has not been a difference shown in DVT rates between implants.
Answer 2: Cephalomedullary nails are biomechanically advantageous given
their intramedullary location within the bone being closer to the center of rotation of the hip. This reduces the moment arm and overall forces seen by the implant compared to the extramedullary sliding hip screw.
Answer 3: Most studies favor CMN or show no difference in blood loss. Answer 4: In appropriate fracture patterns, there has not been a consistent difference seen in risk of nonunion. Quality of reduction and screw placement is more important than implant type.
- A 26-year-old female presents to the emergency department with right knee pain, swelling, and inability to ambulate for 3 days. She was previously very active and notes no trauma to the knee. Knee range of motion is limited to a 20-degree arc of motion with an obvious large effusion. Radiographs are shown in figure A. Right knee synovial aspiration demonstrates 65,000 WBC with 92% PMNs and no crystals. Upon further questioning, she endorses new vaginal discharge following a new sexual encounter. Given this clinical scenario, which of the following accurately describes the expected causative pathogen?
- Gram-positive cocci in chains
- Gram-positive cocci in grape-like clusters
- A combination of intracellular and extracellular gram-negative diplococci
- Extracellular, facultative anaerobe, gram-negative bacilli
- Extracellular, aerobic, gram-negative bacilli CORRECT ANSWER: 3
This young otherwise healthy female patient has a history concerning for Neisseria gonorrhea septic arthritis of the right knee. The expected morphology is a combination of intracellular and extracellular gram-negative diplococci.
Septic arthritis of the knee is diagnosed with a combination of history, physical examination, and aspiration results. In young otherwise healthy patients with a presentation consistent with acute monoarticular septic arthritis, sexual history should be obtained as suspicion should be very high for a diagnosis of gonococcal septic arthritis. Accurate diagnosis of gonococcal septic arthritis is important as it directs antibiotic treatment to include ceftriaxone (or cefotaxime, or ceftizoxime). Treatment typically includes some form of arthroscopic drainage most commonly through arthroscopic or open irrigation and debridement or less commonly serially aspirations.
Favero et al. reviewed 42 patients diagnosed with septic arthritis from their rheumatology clinic. They found patients under 60 years of age were more frequently affected by joint disease and had lower synovial white blood cell count than patients >60. They conclude the recent increased therapeutic
aggressiveness in rheumatology patients may have increased the frequency of joint disease and lower synovial white blood cell counts in patients younger than 60.
Frazee et al. reviewed 109 synovial fluid samples sent from the emergency department over a 15 month period. They found 12 positive cases of septic arthritis 50% were from MRSA with 33% from MSSA. They concluded that in this emergency department population MRSA was the most common cause of community-acquired adult septic arthritis.
Figure A shows a lateral non-weight bearing x-ray of the right knee with a large knee effusion. Illustration A shows a histologic slide with a combination of intracellular and extracellular gram-negative diplococci consistent with Neisseria gonorrhea.
Incorrect Answers:
Answer 1: Gram-positive cocci in chains most accurately describes streptococcal morphology.
Answer 2: Gram-positive cocci in grape-like clusters describes staphylococcal morphology.
Answer 4: Extracellular, facultative anaerobe, gram-negative bacilli all describe the characteristics of Escherichia or E. coli morphology.
Answer 5: Extracellular, aerobic, gram-negative bacilli all describe the characteristics of pseudomonal morphology.
- A 31-year-old male presents with persistent foot pain 4 months after falling from a ladder. He was treated with an ankle brace for a suspected ankle sprain at the time of injury. On exam there is tenderness about the lateral hindfoot, no laxity on anterior drawer
testing, and full eversion strength. Imaging demonstrates a nonunion of the anterior process of the calcaneus (Figure 1). Which of the following mechanisms and ligaments are involved?
- Inversion and dorsiflexion, calcaneofibular ligament
- Inversion and plantar flexion, bifurcate ligament
- Inversion and dorsiflexion, anterior talofibular ligament
- Inversion and plantar flexion, anterior talofibular ligament
- Eversion with ankle neutral, tibiocalcaneal ligament CORRECT ANSWER: 2
The anterior process of the calcaneus is an attachment of the bifurcate ligament, which can be avulsed with excessive inversion and plantar flexion.
Calcaneal anterior process fractures are uncommon injuries that are easily missed at the initial presentation. They are prone to nonunion and may lead to long-term pain and disability. Three types have been described: Type I) nondisplaced avulsion, Type II) displaced avulsion not involving the calcaneocuboid articulation, and Type III) displaced involving the calcaneocuboid joint. The dorsal aspect of the process serves as an origin for the extensor digitorum brevis and the bifurcate ligament. The bifurcate ligament inserts on the dorsal navicular and cuboid and resists plantar flexion- inversion forces. This fracture is best identified using oblique radiographs of the foot or CT. Treatment is based on the degree of calcaneocuboid articular involvement and chronicity. Immobilization and non-weight bearing is the treatment of choice for acute nondisplaced fractures.
Trnka et al. reviewed fractures of the anterior superior calcaneal process. The authors noted that this injury is often missed and may lead to persistent pain distal to the sinus tarsi at the calcaneocuboid joint. Oblique foot radiographs should be carefully reviewed when patients present with pain about the lateral hindfoot. The mechanism most often implicated is an avulsion of the bifurcate
ligament during excessive inversion and plantar flexion. The authors conclude operative fixation should be considered earlier for type III fractures due to intraarticular involvement and risk of degenerative changes.
Berkowitz et al. reviewed process and tubercle fractures of the hindfoot. The authors highlight two potential mechanisms – avulsion from plantarflexion/inversion and shearing from forced dorsiflexion/eversion. On exam, tenderness and swelling will be approximately 2cm anterior and 1cm inferior to the ATFL. The authors noted successful treatment when approaching acute nondisplaced fractures with immobilization and non-weight bearing.
However, painful nonunions typically require either excision, ORIF, or arthrodesis. Surgical management should be considered acutely for fragments
>1cm in size and those with > 2mm intraarticular displacement. The authors conclude making the correct initial diagnosis is key to providing the best opportunity for complete recovery.
Figure 1 is a sagittal CT image of the hindfoot demonstrating a nonunion of the anterior process of the calcaneus.
Illustration A demonstrates the anterior process of the calcaneus, the origin of the bifurcate ligament.
Incorrect Answers
Answer 1: Inversion and dorsiflexion results places tension on the CFL. The CFL attaches to the lateral calcaneal body, not the anterior process.
Answer 3: Inversion and dorsiflexion injuries are associated with CFL tears. Additionally, as the name implies, the ATFL attaches to the dorsal talus, not the calcaneus.
Answer 4: Although the mechanism of an ATFL injury is an inversion and plantar flexion force, the ATFL attaches to the dorsal talus, not the calcaneus. Answer 5: The tibiocalcaneal ligament is one of four comprising the deltoid ligament, which can be injured with excessive eversion. However, these ligaments do not attach to the anterior process of the calcaneus.
- A 28-year-old male sustains the injury seen in Figure A. After discussing the risks and benefits of surgery, he elects to pursue nonoperative treatment. Of the following possible complications from nonoperative treatment, which is the most likely?
- Skin necrosis
- Nonunion
- Complex regional pain syndrome (CRPS)
- Sternoclavicular joint arthritis
- Acromioclavicular joint arthrosis CORRECT ANSWER: 2
In a patient with a displaced and shortened middle third clavicle fracture, nonunion would be expected to occur more often than any of the other complications listed.
The risk of nonunion following mid-shaft clavicle fractures is increased with
advanced age, female gender, displacement, and comminution ("Z- deformity"). Nonoperative management of mid-shaft clavicle fractures has also been associated with decreased shoulder strength and endurance.
Furthermore, range of motion and shoulder strength have not been shown to be sufficiently different between operative and nonoperative management.
Hoogervorst et al. reviewed the treatment of mid-shaft clavicle fractures. They cite that the rate of nonunion for fractures treated nonoperatively is about 15%. The authors note that those with >2cm of shortening and displacement
>100% of the shaft width are at a greater risk for nonunion and that nearly 66% of those who go on to nonunion eventually undergo surgical repair.
Robinson et al. performed a prospective observational cohort study to evaluate the prevalence of and risk factors for nonunion of clavicle fractures treated nonoperatively. They found that the risk of nonunion was significantly increased by advancing age, female gender, displacement of the fracture, and the presence of comminution.
Figure A shows a radiograph demonstrating a middle third clavicle shaft fracture with >100% displacement and >2cm shortening.
Incorrect Answers:
Answer 1: The rate of skin necrosis following nonoperative treatment has not been shown to be higher than nonunion.
Answer 3: The rate of complex regional pain syndrome (CRPS) was shown to be around 2% in nonoperatively treated patients.
Answer 4 ad 5: Sternoclavicular arthritis and acromioclavicular arthritis or abnormalities have been shown to be somewhere between 4-6%
- What is the most cost-effective implant indicated for the injury shown in Figures A and B, assuming the hospital purchases the implants at-cost from the manufacturer?
- Long cephalomedullary nail
- Short cephelomedullary nail
- Sliding hip screw
- Hemiarthroplasty
- Cannulated screws CORRECT ANSWER: 3
In the intertrochanteric hip fracture displayed, a sliding hip screw (SHS) or cephalomedullary nail (CMN) would be indicated; the SHS is the cheaper implant.
In a standard obliquity intertrochanteric fracture without lateral wall comminution, an SHS is a cost-effective option. In reverse obliquity fractures or those with lateral wall comminution (unstable), there is an increased failure rate with this implant and an CMN should be considered. There is some controversy in terms of short and intermediate nail indications, which are attractive because they do not require reaming. SHS implants are contraindicated in subtrochanteric fractures. Although some hospitals receive discounts on implants, in general, intramedullary nail options are more expensive than SHS.
Swart et al. examined the cost-effectiveness of different fixation options for intertrochanteric hip fractures. They compared SHS and CMN implants using an expected-value decision-analysis model, taking into account fracture patterns, failure rates, and revision costs. In their study, the average cost of a SHS was
$2,000 compared to $3,200 for a standard CMN. They concluded that the SHS was likely more cost-effective for stable and questionably stable fractures compared to CMN fixation; CMN was more cost-effective for reverse obliquity patterns.
Kaplan et al. provided a review article on the surgical management of intertrochanteric fractures. Factors to determine treatment should include the patient's medical co-morbidities, pre-existing arthritis, bone quality, and fracture morphology. Their study did not find an appreciable difference in patient outcomes using either CMN or SHS for stable intertrochanteric fractures; surgeon experience and implant costs should, therefore, drive the decision making process for implant selection.
Figures A and B are AP and lateral right hip radiographs, respectively, demonstrating a standard obliquity intertrochanteric hip fracture. Illustration A is an AP post-operative radiograph showing a hip fracture treated with a SHS.
Incorrect Answers:
Answers 1 and 2: Both of these options are appropriate for this fracture but more costly than a SHS.
Answer 4: Arthroplasty is not indicated for this patient with an intertrochanteric fracture and preserved joint space.
Answer 5: Cannulated screws are not indicated for an intertrochanteric fracture.
- While snowboarding on the steep slopes in New England, a 56- year-old active right-hand-dominant man falls on his right shoulder and sustains a right proximal humerus fracture. Which of the following fracture characteristics would most directly result in altered rotator cuff biomechanics after closed treatment?
- Comminuted medial hinge
- Intact calcar length of 6 mm
- Greater tuberosity displacement 6 mm
- Anatomical neck displacement 8 mm
- Surgical neck displacement 9 mm CORRECT ANSWER: 3
After closed treatment, residual displacement of the greater tuberosity (GT) of
>5 mm will most likely result in altered rotator cuff biomechanics.
The GT is draped by the confluence of the postero-superior rotator cuff tendons, with the anterior half of the GT insertion composed of supraspinatus (SS) fibers and the posterior half composed of overlapping SS and
infraspinatus (IS) fibers. Greater tuberosity fractures with residual displacement less than 5 mm after closed treatment tend to yield great outcomes with nonoperative management. When displacement is >5 mm (or
>3 mm in active patients), surgical treatment is warranted as the resultant malunion of the rotator cuff attachment site can cause limitation in active abduction and external rotation.
Bissell et al. investigated the epidemiology and risk factors of humerus among skiers and snowboarders. They reported that the incidence of humerus fractures among snowboarders was significantly higher (50%) than that of skiers, and that GT displacement >5 mm predicted the need for operative fixation.
George reviewed GT fractures. They reported that nondisplaced and minimally displaced fractures may be successfully treated nonoperatively, with surgical fixation recommended for fractures with >5 mm of displacement in the general population or >3 mm of displacement in active patients. They recommended close follow-up and supervised rehabilitation to optimize outcomes after both nonoperative and operative management.
Incorrect Answers:
Answer 1 and 2: While a disrupted medial hinge would increase the likelihood of osteonecrosis of the humeral head, it would not directly alter rotator cuff biomechanics.
Answer 2: While an intact calcar length of 6 mm (<8 mm) would increase the likelihood of osteonecrosis of the humeral head, it would not directly alter rotator cuff biomechanics.
Answer 4: Anatomical neck displacement >10 mm would be considered a part in the Neer classification. Displacement of <10 mm has not been correlated with altered rotator cuff biomechanics.
Answer 5: Surgical neck displacement >10 mm would each be considered a part in the Neer classification. Displacement of <10 mm has not been correlated with altered rotator cuff biomechanics.
- A 36-year-old male sustains the closed injury shown in Figure A after falling from a ladder. He is treated nonoperatively. Two years following the injury he presents to the clinic complaining of laterally based hindfoot pain which is worsened when walking on uneven surfaces. His tibiotalar motion remains pain-free. He obtains good pain relief with a steroid injection into the sinus tarsi. What additional treatment modality is appropriate at this time?
- ASO/Lace-up ankle brace
- Arizona/Gauntlet ankle brace
- Custom orthotic with arch support and medial wedging
- UCBL ankle brace
- Distraction bone block arthrodesis CORRECT ANSWER: 2
The gauntlet ankle brace, commercially known as an Arizona brace, or a solid ankle-foot orthosis and continued nonoperative treatment is appropriate for early subtalar arthritis.
Subtalar arthritis is a common complication following surgical and nonsurgical treatment of intra-articular calcaneus fractures. Nonoperative treatment modalities for the condition include ankle bracing which limits hindfoot inversion-eversion, corticosteroid injections, NSAIDs, and physical therapy.
This is a successful mitigation strategy for many patients. Ultimately if surgical intervention is warranted, with failed nonoperative treatment, a subtalar arthrodesis with or without a bone block can be considered.
Griffin et al. performed a randomized control trial comparing operative and non-operative treatment of intra-articular calcaneus fractures. They found equivalent pain and function results at two years post-injury but did find a higher complication rate following surgical intervention. It should be noted that all operative patients in this cohort were treated with an extensile lateral approach.
Hsu et al. reviewed the recent surgical advances in the treatment of calcaneus fractures. They note that the less invasive techniques, including the sinus tarsi approach, have resulted in an overall decreased complication rate.
Jackson et al. evaluated the use of distraction bone block arthrodesis for malunion correction following calcaneus fractures. In addition to the standard requirement of lateral wall exostectomy, a bone block is required to correct the loss of height and restore the normal talar declination. This technique can aid in symptom relief in patients who are having anterior ankle impingement secondary to loss of calcaneal height.
Figure A is a lateral radiograph showing and intra-articular calcaneus fracture. Illustration A is a schematic of a bone block arthrodesis. Illustration B is a clinical photograph of a gauntlet ankle brace.
Incorrect Answers:
Answer 1: An ASO or lace-up ankle brace may be attempted, but typically does not provide the inversion/eversion stability required for pain relief.
Answer 3: An orthotic with arch support and medial wedging is typically used for pes planovalgus.
Answer 4: The University of California Biomechanics Lab orthotic does not sufficiently limit subtalar motion.
Answer 5: This patient should undergo further non-operative treatment before surgical considerations.
- A 32-year-old male sustained a right grade IIIB open tibial shaft fracture 10 months ago when he fell down a ledge while hiking. Due to the location of the injury, it took EMS 15 hours to transport the patient to the ED, where IV antibiotics were promptly started. Subsequently,
the patient underwent external fixation with serial debridements followed by definitive flap coverage and unreamed intramedullary nailing six days after the injury. The patient continues to have pain in the leg with weight-bearing but denies any fevers or chills. His surgical wounds appear well-healed with a small sinus tract over the open fracture site. Figures A and B are the current radiographs. Recent labs reveal an ESR, CRP and 25-hydroxyvitamin D2 of 32 mm/hr (reference 0-20 mm/hr), 15 mg/dL (reference 0-3 mg/dL), and 50 ng/mL (reference 20-100 ng/mL). What factor is most likely associated with this patient's current condition?
- Delay in definitive wound coverage
- Definitive treatment with an unreamed intramedullary nail
- Prolonged time to antibiotic administration
- Hypovitaminosis D
- Low-pressure irrigation during debridements CORRECT ANSWER: 3
The patient is presenting with a septic nonunion following an open tibial shaft fracture. Of the given answer choices, the timely administration of antibiotics, ideally within 3 hours of injury, is associated with decreased wound infection
rates and subsequent septic nonunion.
Tibial shaft fractures are the most common long bone fracture, with up to 24% of fractures presenting as an open injury. Appropriate initial management of these injuries is crucial to optimizing outcomes, as they are often fraught with high wound complication and infection rates. Literature has demonstrated that the prompt delivery of appropriate IV antibiotic coverage is one of the most important factors in minimizing wound infection rates and subsequent septic nonunions. The risk of infection is decreased substantially when antibiotics are started within three hours of the injury, compared to later treatment. Ideal coverage includes 1st generation cephalosporins for grades I and II, sometimes with the addition of gentamicin for grade III injuries. During the evaluation for septic nonunion, elevated CRP and ESR levels are suggestive of an infection.
Babhulkar et al. retrospectively reviewed 113 patients with long bone fracture nonunions of various etiologies, including 16 tibial shaft nonunions. The authors reported that the initial treatment of these injuries consisted of six intramedullary nails, four platings, two external fixators, and one plaster cast, which resulted in nine aseptic and four septic nonunions. Following appropriate treatment, all patients eventually attained fracture union, with 16% of tibial nonunions resulting in an acceptable malunion, 13% resulting in limb shortening, and two patients with a persistent septic nonunion that was successfully treated with Ilizarov frame. The authors concluded that successful treatment of long bone fracture nonunions requires consideration of the pathophysiology of the nonunion and appropriately addressing stability, infection control, and provision of an osteogenic environment.
Mundi et al. reviewed the management of open tibial shaft fractures. The authors supported the use of either bacitracin or castile soap irrigation, with no difference in infection outcomes, while low-pressure irrigation delivery was associated with lower reoperation rates due to infections, nonunion, and wound healing problems. They also supported that definitive closure of wounds within seven days of the initial injury was associated with a reduced rate of complications, including infection. The authors concluded that prompt administration of appropriate antibiotic prophylaxis was the single-most important determinant in minimizing wound infections.
The investigators of the SPRINT (The Study to Prospectively Evaluate Reamed Intramedullary Nails in Patients with Tibial Fractures) study performed a multicenter blinded randomized trial of 1319 patients treated with either reamed or unreamed intramedullary nailing for tibial shaft fractures. In the 406 patients with open fractures, of which 137 were grade III, there was no
treatment benefit between reamed and unreamed nails. However, the investigators reported a significantly increased union rate with reamed intramedullary nails for closed tibial shaft fractures. They concluded that reamed intramedullary nailing appears to have a treatment benefit only for closed tibial shaft fractures.
Figures A and B are the AP and lateral radiographs of the right tibia and fibula demonstrating a comminuted fracture of the tibia with minimal healing after intramedullary nail fixation.
Incorrect answers:
Answer 1: Definitive wound coverage should be performed within seven days of the injury in wounds requiring delayed closure in order to minimize the risk of infection. Wound coverage for the patient in this scenario was delayed < 7 days.
Answer 2: Union rates are comparable between unreamed and reamed intramedullary nailing for open tibial shaft fractures.
Answer 4: The patient has a normal vitamin D level, and an elevated CRP suggests an infectious rather than a nutritional etiology of the nonunion. Answer 5: Low-pressure irrigation is associated with lower reoperation rates due to infection, wound complications, and nonunion.
- A 25-year-old female was involved in a high-speed motor vehicle accident and sustained the injuries shown in Figures A-C. Which of the following provides the most stable fixation construct?
- Triangular osteosynthesis
- Bilateral iliosacral screws
- Anterior pelvic ring plating with bilateral iliosacral screw fixation
- Transsacral bar fixation
- Posterior tension band fixation CORRECT ANSWER: 1
This patient has sustained a U-type sacral fracture. The most stable fixation is triangular osteosynthesis, also referred to as lumbopelvic fixation.
This patient has sustained a spinopelvic dissociation, which occurs when there are multiplanar fracture lines in both the horizontal and vertical planes. This creates an unstable situation with the upper part of the sacrum attached to the lumbar spine and the lower part of the sacrum remaining attached to the pelvis. There are many treatment options for this condition including open or percutaneous iliosacral screw osteosynthesis, tension band transiliac plate osteosynthesis, transiliac bars, and local plate osteosynthesis. If a neurologic defect is present, a sacral decompression with laminectomies is recommended.
Schildhauer et al. performed a retrospective study to report the results of sacral decompression and lumbopelvic fixation in neurologically impaired patients with highly displaced, comminuted sacral fracture-dislocations resulting in spinopelvic dissociation. They found 19 patients who were treated with open reduction, sacral decompression, and lumbopelvic fixation. They concluded that lumbopelvic fixation provided reliable fracture stability, and that
neurological outcomes were influenced by the completeness of the injury and the presence of sacral root disruption.
Schildhauer et al. performed a biomechanical comparison of triangular osteosynthesis and the standard iliosacral screw osteosynthesis for unstable transforaminal sacral fractures in both the immediate postoperative situation and in the early postoperative weight-bearing period. They concluded that triangular osteosynthesis for unstable sacral fractures provided significantly greater stability than iliosacral screw fixation under in vitro cyclic loading conditions.
Figure A is an AP radiograph of the pelvis demonstrating a paradoxical inlet that occurs due to the kyphotic deformity of the sacrum. Figure B is an axial CT demonstrating a U-type sacral fracture. Figure C is a sagittal CT demonstrating the same U-type sacral fracture. The kyphotic deformity is well visualized on this image. Illustration A is an AP radiograph of the same patient after surgical treatment with triangular osteosynthesis. Illustrations B & C are coronal and sagittal CTs demonstrating the same. Illustration D is a volume- rendered 3D image of the same patient demonstrating a U-type sacral fracture. Illustration E demonstrates the various multiplanar sacral fractures: A: H-Type, B: Y-Type, C: T-Type, D: U-Type.
Incorrect Answers:
Answers 2, 3, 4, & 5: Though all of these may be treatment options for treating this injury, triangular osteosynthesis is the most stable fixation construct.
- You have a 25-year-old male patient who fell from a 20-foot wall and is brought in by EMS. His only complaint is severe left heel pain. After seeing the patient, you make your diagnosis and decided that non-operative treatment is the best option. When counseling your patient, what should you tell him is the most common complication of non-operative treatment for this injury?
- Nonunion
- Subtalar arthritis
- Foot compartment syndrome
- Avascular necrosis
- Valgus hindfoot malalignment CORRECT ANSWER: 2
Calcaneal fractures that are treated non-operatively most commonly will have associated subtalar arthritis.
Calcaneal fractures comprise 65% of tarsal bone fractures and require a significant amount of force to produce. The force of the calcaneus being pushed into the talus often results in post-traumatic arthritis. 75% of calcaneal fractures have an intra-articular extension, with the posterior facet being most
commonly affected. The treatment for severe post-traumatic arthritis of the subtalar joint is subtalar fusion.
Hsu et al. reviewed current methods of calcaneus surgical fixation. Previous L- shaped lateral approaches had high complication and infection rates postoperatively at 37% and 20% respectively. New methods including limited sinus tarsi, percutaneous fixation, and arthroscopic-assisted reduction. With these methods, there is decreased complication rates and less soft tissue injury.
Jackson et al. reviewed distraction subtalar arthrodesis. Calcaneal fractures often have a loss of height, which results in anterior impingement of the tibiotalar joint. Restoration of subtalar height can be done via open approach, use of a distractor to regain height and placement of graft to fill the void. With this method, there have been significant improvements in functional outcome scores and >90% patient satisfaction.
Buckley and Tough reviewed displaced intraarticular calcaneal fractures and factors that affect surgical outcomes. Smoking and workers compensation are risk factors for poor outcomes, while female gender and age less than 60 have better outcomes with surgical intervention. Heavy laborers have quicker return to work and lower incidence of late fusion. A Bohler's angle >15 predicts better fuctional outcomes in both surgical and nonsurgical patients. While Sanders II and III patients had better outcomes with surgery.
Figures A-C show the AP, oblique and lateral of the foot. On the lateral, you can see the intraarticular calcaneal fracture with depression of the posterior facet and loss of calcaneal pitch.
Incorrect answers:
Answer 1: Nonunion of calcaneal fractures is exceedingly rare.
Answer 3: Although associated with severely comminuted calcaneal fractures, foot compartment syndrome occurs in about 10% of patients.
Answer 4: Avascular necrosis is more associated with talar neck fractures than calcaneal fractures.
Answer 5: Calcaneal fractures typically develop a varus deformity due to the tuberosity fragment being pulled into varus.